immunology nclex-combined sets

Ace your homework & exams now with Quizwiz!

When preparing a client with acquired immunodeficiency syndrome (AIDS) for discharge to the home, the nurse should be sure to include which instruction?

"Avoid sharing such articles as toothbrushes and razors."

A patient who is diagnosed with AIDS tells the nurse, "I have lots of thoughts about dying. Do you think I am just being morbid?" Which response by the nurse is best?

"Can you tell me more about the kind of thoughts that you are having?"

The nurse is caring for a patient undergoing plasmapheresis. The nurse should assess the patient for which clinical manifestation? a. Shortness of breath b. High blood pressure c. Transfusion reaction d. Numbness and tingling

ANS: D Numbness and tingling may occur as the result of the hypocalcemia caused by the citrate used to prevent coagulation. The other clinical manifestations are not associated with plasmapheresis

Which nursing action will be most useful in assisting a college student to adhere to a newly prescribed antiretroviral therapy (ART) regimen? a. Give the patient detailed information about possible medication side effects. b. Remind the patient of the importance of taking the medications as scheduled. c. Encourage the patient to join a support group for students who are HIV positive. d. Check the patient's class schedule to help decide when the drugs should be taken.

ANS: D The best approach to improve adherence is to learn about important activities in the patient's life and adjust the ART around those activities. The other actions also are useful, but they will not improve adherence as much as individualizing the ART to the patient's schedule

Which of the following blood types would the nurse identify as the rarest?

AB

A patient is admitted to the hospital with acute rejection of a kidney transplant. Which intervention will the nurse prepare for this patient? a. Administration of immunosuppressant medications b. Insertion of an arteriovenous graft for hemodialysis c. Placement of the patient on the transplant waiting list d. A blood draw for human leukocyte antigen (HLA) matching

ANS: A Acute rejection is treated with the administration of additional immunosuppressant drugs such as corticosteroids. Because acute rejection is potentially reversible, there is no indication that the patient will require another transplant or hemodialysis. There is no indication for repeat HLA testing

A patient treated for human immunodeficiency virus (HIV) infection for 6 years has developed fat redistribution to the trunk, with wasting of the arms, legs, and face. What instructions will the nurse give to the patient? a. Review foods that are higher in protein. b. Teach about the benefits of daily exercise. c. Discuss a change in antiretroviral therapy. d. Talk about treatment with antifungal agents.

ANS: C A frequent first intervention for metabolic disorders is a change in antiretroviral therapy (ART). Treatment with antifungal agents would not be appropriate because there is no indication of fungal infection. Changes in diet or exercise have not proven helpful for this problem

A patient who has a positive test for human immunodeficiency virus (HIV) antibodies is admitted to the hospital with Pneumocystis jiroveci pneumonia (PCP) and a CD4+ T-cell count of less than 200 cells/mL. Based on diagnostic criteria established by the Centers for Disease Control and Prevention (CDC), which statement by the nurse is correct? a. "The patient meets the criteria for a diagnosis of an acute HIV infection." b. "The patient will be diagnosed with asymptomatic chronic HIV infection." c. "The patient has developed acquired immunodeficiency syndrome (AIDS)." d. "The patient will develop symptomatic chronic HIV infection in less than a year."

ANS: C Development of PCP meets the diagnostic criterion for AIDS. The other responses indicate earlier stages of HIV infection than is indicated by the PCP infection

The charge nurse is assigning rooms for new admissions. Which patient would be the most appropriate roommate for a patient who has acute rejection of an organ transplant? a. A patient who has viral pneumonia b. A patient with second-degree burns c. A patient who is recovering from an anaphylactic reaction to a bee sting d. A patient with graft-versus-host disease after a recent bone marrow transplant

ANS: C Treatment for a patient with acute rejection includes administration of additional immunosuppressants, and the patient should not be exposed to increased risk for infection as would occur from patients with viral pneumonia, graft-versus-host disease, and burns. There is no increased exposure to infection from a patient who had an anaphylactic reaction

A patient who is receiving immunotherapy has just received an allergen injection. Which assessment finding is most important to communicate to the health care provider? a. The patient's IgG level is increased. b. The injection site is red and swollen. c. The patient's allergy symptoms have not improved. d. There is a 2-cm wheal at the site of the allergen injection.

ANS: D A local reaction larger than quarter size may indicate that a decrease in the allergen dose is needed. An increase in IgG indicates that the therapy is effective. Redness and swelling at the site are not unusual. Because immunotherapy usually takes 1 to 2 years to achieve an effect, an improvement in the patient's symptoms is not expected after a few months

A complete blood count is commonly performed before a client goes into surgery. What does this test seek to identify?

Abnormally low hematocrit (HCT) and hemoglobin (Hb) levels

ANTIBIOTICS Aminoglycoside common adverse

Aminoglycosides Streptomycin & Neomycin common adverse: - ototoxicity - palpitations

Anti-infective Medication 1) name the drug 2) indication

Anti-infective Medication 1) Metronidazole (Flagyl) 2) pneumonia/ opportunistic infections

Anti-inflammatory Drugs 1) name the drug 2) adverse

Anti-inflammatory 1) Sulfasalazine (Azulfidine) 2) adverse: GI irritation

d

Association between HLA antigens and disease is most commonly found in what disease condition? A. malignancies B. infectious disease C. neurologic diseases D. autoimmune disorders

The nurse is assisting in developing a plan of care for a client with acquired immunodeficiency syndrome (AIDS) who is experiencing night fever and night sweats. Which nursing intervention should the nurse suggest including in the plan of care to manage this symptom?

Administer an antipyretic at bedtime

antigen presentation triggers

B cell production of an antibody specific for the antigen cytotoxic t cells able to lyse cells infected with the pathogen

The nurse is providing information to a client with systemic lupus erythematosus (SLE) about dietary alterations. The nurse should remind the client to avoid which foods? Select all that apply.

Beef Cheese

A client with suspected lymphoma is scheduled for lymphangiography. The nurse should inform the client that this procedure may cause which harmless, temporary change?

Bluish urine

A 19-year-old male being tested for multiple allergies develops localized redness and swelling in reaction to a patch skin test. Which intervention by the nurse would have the highest priority? A. Notify the primary care provider B. Apply a topical anti-inflammatory cream C. Remove the patch and extract from the skin D. Administer oral diphenhydramine (Benadryl)

C. Remove the patch and extract from the skin If a severe reaction to a patch skin test occurs, the nurse should immediately remove the patch and the extract from the skin. Next the nurse should apply a topical anti-inflammatory cream to the site. A subcutaneous injection of epinephrine may also be necessary but would need a health care provider's order

The health care provider prescribes fluconazole (Diflucan) for a client. When administering this medication the nurse should explain to the client that it is used to treat which opportunistic infection?

Candidiasis

C3 convertase

Converts C3 into C3b and C3a

When caring for a clinic patient who is experiencing an allergic reaction to an unknown allergen, which nursing activity is most appropriate for the RN to delegate to an LPN/LVN?

Administer skin testing by the cutaneous scratch method.

The 3 C's

Coryza, cough and conjunctivitis ; found in rubeola (measles)

When assessing an individual who has been diagnosed with early chronic HIV infection and has a normal CD4+ count, the nurse will a. ask about problems with diarrhea. b. examine the oral mucosa for lesions. c. check neurologic orientation. d. palpate the regional lymph nodes.

D Rationale: Persistent generalized lymphadenopathy is common in the early stage of chronic infection. Diarrhea, oral lesions, and gait abnormalities would occur in the later stages of HIV infection. Cognitive Level: Application Text Reference: p. 252 Nursing Process: Assessment NCLEX: Physiological Integrity

PCR of germline DNA from B cells

DNA fragments of all the same length after PCR

How antibody diversity is generated

Each variable region has different codons and each dif codon has a different antibody it is then spliced into smaller pieces randomly and this gives almost infinite combinations

Adaptive immunity

Educated immune response - memory of a pathogen, formed by antibodies

Which are risk factors for systemic lupus erythematous (SLE)? Select all that apply.

Female gender African-American origin Being in the childbearing years

what is a common adverse effect for all ANTIBIOTIC medications what is the action of antibiotics

GI upset, renal dysfunction, superinfection inhibit bacterial growth

d

If a person is heterozygous for a given gene, it means that the person: A. is a carrier for a genetic disorder B. is affected by the genetic disorder C. has two identical allels for the gene D.has two different allels for the gene

Which immunoglobulin is specific to an allergic response?

IgE

Adjuvants

Ingredient added to vaccines to create a stronger immune response

How can a nurse best ensure the safety of a client who has a latex allergy?

Make sure that the latex allergy is properly documented.

Lymphoid lineage

NK cell, Th Cell, Tc Cell, Bcell, plasma cell

When the nurse is caring for a patient whose HIV status in unknown, which of these patient exposures is most likely to require postexposure prophylaxis?

Needle stick with a needle and syringe used to draw blood

Which of the following is the most numerous type of white blood cell (WBC)?

Neutrophil

Myeloid lineage

Neutrophil, eosinophil, basophil, mast cell, macrophage, monocyte

Which medications should the nurse administer to reduce nasal edema and rhinorrhea (thin watery discharge from the nose)? Select all that apply.

Oxymetazole (Dristan) Pseudoephedrine (Sudafed)

Which factor is most important when planning care for a client with a bleeding disorder?

Prioritization

A client with acquired immunodeficiency syndrome (AIDS) is experiencing shortness of breath related to Pneumocystis jiroveci pneumonia. Which measure should the nurse suggest to assist the client in performing activities of daily living?

Provide supportive care with hygiene needs

After an extensive diagnostic workup, a client is diagnosed with systemic lupus erythematosus (SLE). Which statement about the incidence of SLE is true?

SLE tends to occur in families.

Intrinsic immunity

Series of factors that are capable of preventing virus replication - switched on by interferons

A client receiving ferrous sulfate (Fer-Iron) therapy to treat an iron deficiency reports taking an antacid frequently to relieve heartburn. Which instruction should the nurse provide?

Take ferrous sulfate and the antacid at least 2 hours apart."

The nurse is explaining about antigens and antibodies when the client asks where antibodies come from. Which is an appropriate response? Select all that apply.

Tears Spleen Saliva Blood serum Lymph nodes

Isotypic variation

The heavy chain ( gamma, delta etc.) The light chain ( Lambda etc.)

a

The most common cause of secondary immunodeficiencies is: A. drugs B. stress C. malnutrition D. human immunodeficiency virus

A 20-year-old female patient who is HIV-positive has a new prescription for efavirenz (Sustiva). Which information about the patient is most important to communicate to the prescribing physician before administering the efavirenz?

The patient is sexually active and does not use any contraception.

Which information about an HIV-positive patient who is taking antiretroviral medications is most important for the nurse to address when planning care?

The patient states "sometimes I miss a dose of zidovudine (AZT)."

IgM (immunoglobulin M)

The primary response to the antigen pentatonic strusture

a

The reason newborns are protected for the first 6 months of life from bacterial infection is because of the maternal transmission of: A. IgG B. IgA C. IgM D. IgE

Which information about a patient who is receiving immunotherapy and has just received an allergen injection is most important to communicate to the health care provider?

There is a 3-cm wheal at the site of the allergen injection.

The nurse collects data on a client shortly after kidney transplant surgery. Which postoperative finding must the nurse report to the physician immediately?

Urine output of 20 ml/hour

Idotypic variation

Variation of the antigen binding domain

primary response

Virgin B cell with immunoglobulins on the surface recept the antigen and form plasma cells which secrete the appropriate antibody or they become memory cells

Macrophage

White blood cell responsible for the destruction of microbes

a patient with PJP usually presents with which symptom? A. dyspnea, tachypnea, persistent dry cough, fever b. cough with copious thick sputum, fever, and dyspnea c. chest pain and difficulty swallowing D. fever, persistant cough and vomiting

a

an IV drug user who regularly shares needles is in the ER. what information does the nurse provide to decrease he patients risk of HIV through shared needles after each use? A. fill and flush syringe with clear water, fill with bleach and shake for 30-60 seconds and rinse with clear water B. fill and flush with water then soap and hot water, shake for 2 minutes and flush with cold water C. rinse needles with bleach and water solution and allow to air dry D. rinse needles after each use with rubbing alcohol and water, then rinse with water

a

Plasma cells

a form of b cell Produce the antibodies

The mother of a child with newly diagnosed Duchenne's muscular dystrophy asks how her child developed the disease. The nurse gives a response incorporating which statement about its transmission? a) "It is a disorder usually carried by females and transmitted to male children." b) "It is a disorder primarily transmitted by males in the family." c) "It is an autosomal recessive genetic disorder." d) "It is a genetic disorder carried by males and transmitted to male children

a) "It is a disorder usually carried by females and transmitted to male children." The gene for Duchenne's muscular dystrophy is carried by women and transmitted to their male children. It involves an X-linked inheritance pattern. About one-third of new cases involve mutations.

what methods or agents are used to treat kaposi's sarcoma (Select all that apply) a. radiotherapy b. chemo c. antibiotics d. cryotherapy e. surgery

a, b, d

which actions can the nurse delegate to the UAP who will be giving mouth care to a patient with HIV/AIDS (Select all that apply) a. offer mouth rinses with sodium bicarb and sterile water several times a day b. assess mouth for increased presence of lesions c. encourage the patient to drink plenty of fluids d. provide a soft bristled toothbrush e. administer oral analgesic gel

a, c, d

which lab resluts will the nurse expect to decrease (Select all that apply) a. cd4+ b. cd8+ c. WBC d. lymphocytes e. HIV antibodies

a, c, d

which practices are recommended to prevent transmission of HIV? (Select all that apply) A. latex condoms for genital and anal intercourse B. natural membrane condoms for genital and anal intercourse C. topical contraceptives D. antiviral meds E. latex barrier for genital and anal intercourse

a, e

Immediately after the nurse administers an intracutaneous injection of an allergen on the forearm, a patient complains of itching at the site and of weakness and dizziness. The nurse should first ______________

apply a tourniquet above the injection site.

The mother does not want her child to have any extra immunizations for diseases that no longer occur. What teaching about immunization should the nurse provide this mother? a) There is currently no need for those older vaccines. b) There is a reemergence of some of the infections, such as pertussis. c) There is no longer an immunization available for some of those diseases. d) The only way to protect your child is to have the federally required vaccines.

b) There is a reemergence of some of the infections, such as pertussis. Teaching the mother that some of the diseases are reemerging and the damage they can do to her child gives the mother the information to make an informed decision. The immunizations still exist and do protect individuals.

how does HSV manifest itself in patients with HIV/AIDS (Select all that apply) a. maculopapular lesions that can spread b. chronic ulceration after vesicles rupture c. vesicles ocated in the perirectal, oral, and genital area d. numbness and tingling before vesicle forms e. itching localized to perianal area

b, c, d

where can candidiasis occur in the body (Select all that apply) a. nose b. esophagus c. vagina d. mouth e. ears

b, c, d

HIV is most commonly transmitted by which routes? (Select all that apply) A. oral B. sexual C. parenteral D. airborne E. perinatal

b, c, e

which statements are true about immunodeficiency? (Select all that apply) A. it causes a decrease in the patients risk for infection B. it may be acquired or congential C. it occurs when a persons body cannot recognize antigens D. it is the same as autoimmunity E. it may cause varied reactions from mild, localized health problems to total immune system failure

b, c, e

A client with rheumatoid arthritis is about to begin aspirin therapy to reduce inflammation. When teaching the client about aspirin, the nurse discusses adverse reactions to prolonged aspirin therapy. These include:

bilateral hearing loss.

IgY

birds reptiles and amphibia combined functions of IgE and IgG

shingles results from VZV leaving the body by which route? a. mucous membrane b. pulmonary space c. body fluids and other tissues d. bone marrow

c

the HCP prescribes an integrase inhibitor for an HIV patient. the patient asks the nurse how this drug works. what is the nurses best response? A. it reduces how well HIV genetic material can be converted into human genetic material B. it reinforces the immune systems ability to fight off an infection C. it prevents viral DNA from integrating into hosts DNA D. prevent HIV infection from progressing to AIDS

c

Which patient is most likely to have a multifactorial genetic disorder? a) A 20-year-old woman with cystic fibrosis b) A 50-year-old man with sickle cell disease c) A 40-year-old man with coronary artery disease d) A 30-year-old woman with polycystic kidney disease

c) A 40-year-old man with coronary artery disease Coronary artery disease may be caused by a combination of genetic factors and health behaviors. Cystic fibrosis, sickle cell disease, and polycystic kidney disease are examples of single gene disorders, which are relatively rare.

C3a and C5a

cause inflammation and chemotaxis

Innate functions of the macrophage

microbial activity modulation of the immune response lymphocyte activation tissue reorganisation inflammation and fever

In an individual with Sjögren's syndrome, nursing care should focus on:

moisture replacement.

Antibodies structure

protein part- specific and joins to the pathogen constant region - involved to communicate the immune response

To evaluate the effectiveness of ART, the nurse will schedule the patient for ______________

viral load testing.

The registered nurse (RN) caring for an HIV-positive patient admitted with tuberculosis can delegate which action to unlicensed assistive personnel (UAP)? a. Teach the patient about how to use tissues to dispose of respiratory secretions. b. Stock the patient's room with all the necessary personal protective equipment. c. Interview the patient to obtain the names of family members and close contacts. d. Tell the patient's family members the reason for the use of airborne precautions.

ANS: B A patient diagnosed with tuberculosis would be placed on airborne precautions. Because all health care workers are taught about the various types of infection precautions used in the hospital, the UAP can safely stock the room with personal protective equipment. Obtaining contact information and patient teaching are higher-level skills that require RN education and scope of practice

The nurse prepares to administer the following medications to a hospitalized patient with human immunodeficiency (HIV). Which medication is most important to administer at the right time? a. Oral acyclovir (Zovirax) b. Oral saquinavir (Invirase) c. Nystatin (Mycostatin) tablet d. Aerosolized pentamidine (NebuPent)

ANS: B It is important that antiretrovirals be taken at the prescribed time every day to avoid developing drug-resistant HIV. The other medications should also be given as close as possible to the correct time, but they are not as essential to receive at the same time every day

The nurse is assigned to a client with polymyositis. Which expected outcome in the plan of care relates to a potential problem associated with polymyositis?

"Client will exhibit no signs or symptoms of aspiration."

A client diagnosed with human immunodeficiency virus (HIV) infection states, "I'm afraid of gaining weight, so I always supplement my diet with vitamins." Which response by the nurse is appropriate?

"Eating a variety of healthy foods is the best source of vitamins."

The nurse is teaching the parents of a child with hemophilia about how to provide a safe home environment throughout the child's life. Which nursing instruction is most appropriate?

"Establish a written emergency plan that includes what to do in specific situations and the names and phone numbers of emergency contacts."

While the nurse is obtaining an assessment and health history from a patient, which statement by the patient will alert the nurse to a possible immunodeficiency disorder?

"I had my spleen removed many years ago after a car accident."

The nurse is caring for a client with possible immune deficiency. Which subjective data would be most indicative?

"Just as I get over a virus, it seems that I get another." this is the Correct to this question Explanation: Immune deficiencies make it harder for the body to fight infection. With a low resistance, the client is susceptible to obtaining more circulating viruses. Having morning stuffiness and a sore throat is indicative of sinus congestion. Having a leg sore is indicative of cardiovascular insufficiency or diabetes. Sneezing with watery eyes is indicates seasonal allergies.

Which instruction will be included when teaching a patient with possible allergies about intradermal skin testing?

"Plan to wait in the clinic for 20 to 30 minutes after the testing."

A client with acquired immunodeficiency syndrome (AIDS) is prescribed zidovudine (azidothymidine, AZT [Retrovir]), 200 mg by mouth every 4 hours. When teaching the client about this drug, the nurse should provide which instruction?

"Take zidovudine every 4 hours around the clock."

A 40-year-old client with mild dementia related to end-stage acquired immunodeficiency syndrome (AIDS) is preparing for discharge. She has decided against further curative treatment. Before discharge, she develops ocular cytomegalovirus (CMV). Her physician recommends treatment with a ganciclovir-impregnated implant (Vitrasert), which requires a surgical procedure. The client's husband feels the implant won't help the client and asks the nurse if the implant will cure CMV. Which response best answers the husband's question while promoting client advocacy?

"The implant won't cure the virus, but it may help preserve her vision. If she can't see you or her surroundings, it may worsen her dementia and make caring for her at home more difficult."

(SELECT ALL THAT APPLY) The nurse is preparing a client with systemic lupus erythematosus (SLE) for discharge. Which instructions should the nurse include in the teaching plan?

(1) Stay out of direct sunlight., (3) Monitor body temperature., (4) Taper the corticosteroid dosage as ordered by the physician when symptoms are under control.

The nurse is admitting a child with a a dx. of mumps. What should the nurse do to prevent transmission?

*Airborne droplet and contact precautions *It is transmitted by droplet by infected person or direct contact * Negative pressure room; at least 12 exchanges per hr. *N95 respirator mask, gown, gloves

Passive immunity

-occurs from antibody transmission and occurs rapidly but it's temporary. -may be transferred by mother to neonate.

Which nursing diagnosis should the nurse expect to see in a plan of care for a client in sickle cell crisis?

...

In counseling a couple in which the man has an autosomal recessive disorder, and the woman has no gene for the disorder, the nurse uses Punnett squares to show the couple that the probability of their having a child with the disorder is ________________

0%.

Drag and Drop question - Click and drag the following steps to place them in the correct order. Question: A client is experiencing an allergic response. The nurse should perform the actions in which order from first to last? All options must be used. 1 Activate the rapid response team. 2 Assess the airway and breathing pattern. 3 Notify the health care provider (HCP). 4 Assess for urticaria.

1) Assess the airway and breathing pattern. 2) Assess for urticaria. 3) Activate the rapid response team. 4) Notify the health care provider (HCP). Explanation: If a client is experiencing an allergic response, the nurse's initial (1st) action is to assess the client for signs/symptoms of anaphylaxis, first checking the airway, breathing pattern, and vital signs, with particular attention to signs of increasing edema and respiratory distress. (2nd) The nurse should then assess for other indications of anaphylaxis, such as urticaria, feelings of impending doom or fright, weakness, sweating (because a severe systemic response to an allergen can result in massive vasodilation), increased capillary permeability, decreased perfusion, decreased venous return, and subsequent decreased cardiac output. (3rd) The nurse should call the rapid response team and (4th) then notify the HCP.

A nurse is identifying clients in the community at risk for latex allergy. Which client population is most at risk for developing this type of allergy?

1. Children in day care centers 2. Individuals with spina bifida 3. Individuals with cardiac disease 4. Individuals living in a group home Answer: 2 Rationale: Individuals at risk for developing a latex allergy include health care workers; individuals who work with manufacturing latex products; individuals with spina bifida; individuals who wear gloves frequently such as food handlers, hairdressers, and auto mechanics; and individuals allergic to kiwis, bananas, pineapples, passion fruit, avocados, and chestnuts.

A health care provider aspirates synovial fluid from a knee joint of a client with rheumatoid arthritis. The nurse reviews the laboratory analysis of the specimen and would expect the results to indicate which finding?

1. Cloudy synovial fluid 2. Presence of organisms 3. Bloody synovial fluid 4. Presence of irate crystals Answer: 1 Rationale: Cloudy synovial fluid is diagnostic of rheumatoid arthritis. Organisms present in the synovial fluid are characteristic of a septic joint condition. Bloody synovial fluid is seen with trauma. Urate crystals are found in gout.

A clinic nurse periodically cares for a client diagnosed with acquired immunodeficiency syndrome. The nurse assesses for an early manifestation of Pneumocystis jiroveci infection by monitoring for which of the following at each client visit?

1. Fever 2. Cough 3. Dyspnea on exertion 4. Dyspnea at rest Answer: 2 Rationale: The client with P. jiroveci infection usually has a cough as the first symptom, which begins as nonproductive and then progresses to productive. Later signs include fever, dyspnea on exertion, and finally dyspnea at rest.

A nurse is assisting in developing a plan of care for a pregnant client with acquired immunodeficiency syndrome (AIDS). The nurse determines that which of the following is the priority concern for this client?

1. Inability to care for self at home 2. Development of an infection 3. Lack of available support services 4. Isolation Answer: 2 Rationale: Acquired immunodeficiency syndrome decreases the body's immune response, making the infected person susceptible to infections. AIDS affects helper T lymphocytes, which are vital to the body's defense system. Opportunistic infections are a primary cause of death in people affected with AIDS. Therefore preventing infection is a priority of nursing care. Although the concerns in options 1, 3, and 4 may need to be addressed at some point in the care of the client, these are not the priority.

A nurse is assisting in developing a plan of care for a client with immunodeficiency. The nurse understands that which problem is a priority for the client?

1. Infection

A nurse is doing discharge teaching with a client who has sickle cell disease. The nurse instructs the client to avoid which factor that could precipitate a sickle cell crisis?

1. Infection

A nurse is doing discharge teaching with a client who has sickle cell disease. The nurse instructs the client to avoid which factor that could precipitate a sickle cell crisis?

1. Infection 2. Mild exercise 3. Fluid overload 4. Warm weather Answer: 1 Rationale: The client should avoid infections, which can increase metabolic demand and cause dehydration, precipitating a sickle cell crisis. The client should also avoid dehydration from other causes. Warm weather and mild exercise do not need to be avoided, but the client should take measures to avoid dehydration during these conditions. Fluids are important to prevent dehydration. Finally, the client should avoid being in areas of high altitude, or flying in a nonpressurized aircraft because of lesser oxygen tension in these areas.

A client calls the health care clinic and tells the nurse that he was bitten by a tick. The client is concerned and asks the nurse about the first signs of Lyme disease. The nurse informs the client that stage 1 of Lyme disease is characterized by:

1. Skin rash

A nurse is collecting data on a client with rheumatoid arthritis. The nurse looks at the client's hands and notes these characteristic deformities. The nurse identifies this deformity as: Refer to figure.

1. Ulnar drift

A kid with rubeola (measles) is being admitted to hosp. The nurse should plan for which precaution 1. enteric 2. airborne 3. protective 4. neutropenic

2 * rubeola is transmitted via airborne particles and dirsct contact with the infectious drops. People involved with the kid should wear a mask, private room, door remains closed,

A client with acquired immunodeficiency syndrome (AIDS) reports nausea, vomiting, and abdominal pain after beginning didanosine (Videx) therapy. The clinic nurse emphasizes what instruction to this client?

2. Come to the health care clinic to be seen by the health care provider.

A client is diagnosed with stage I of Lyme disease. In addition to the rash, the nurse would check the client for which manifestation?

2. Flulike symptoms

A client who is prescribed zidovudine (Retrovir) has been diagnosed with severe neutropenia. The nurse anticipates which intervention will be implemented?

2. The medication will be temporarily discontinued.

The nurse prepares to give a bath and change the bed linens on a client with cutaneous Kaposi's sarcoma lesions. The lesions are open and draining a scant amount of serous fluid. Which of the following would the nurse incorporate in the plan during the bathing of this client?

2. Wearing a gown and gloves

A child is scheduled to receive inactivitated Polio vaccine (IPV) and the nurse preparing the vaccine reviews the kids chart. The nurse should question this if what is in the kids chart? 1. Recent recovery from cold 2. A history of frequent respiratory infections 3. A history of an anaphyalactic reaction to neomycin 4. A local reaction at the site if injection of a previous IPV

3 *This contains neomycin

Indinavir (Crixivan) is prescribed for a client with human immunodeficiency virus (HIV). The nurse has provided instructions to the client regarding ways to maximize absorption of the medication. Which of the following, if stated by the client, indicates an adequate understanding of the use of this medication?

3. "I need to take the medication with water but on an empty stomach."

The home care nurse is collecting data from a client who has been diagnosed with an allergy to latex. In determining the client's risk factors associated with the allergy, the nurse questions the client about an allergy to which food item?

4. Bananas

A client who is human immunodeficiency virus (HIV) positive has had a Mantoux skin test. The results show a 7-mm area of induration. The nurse evaluates that this result is:

4. Positive

A client is diagnosed with an immune deficiency. The nurse focuses on which of the following as the highest priority when providing care to this client?

4. Protecting the client from infection

A client is diagnosed with stage 1 Lyme disease. The nurse checks the client for which hallmark characteristic of this stage?

4. Skin rash

c

A father who has an X-linked recessive disorder and a wife with a normal genotype will: A. pass the carrier state to his make child B. pass the carrier state to all of his children C. pass the carrier state to his female child D. not pass on the genetic mutation to any of is children

A female client with human immunodeficiency virus (HIV) receives family-planning counseling. Which statement about safe sex practices for persons with HIV is accurate?

A latex condom with spermicide provides the best protection against HIV transmission during sexual intercourse.

Which of these patients will the nurse working in an HIV testing and treatment clinic anticipate teaching about antiretroviral therapy (ART)?

A patient who tested positive for HIV 2 years ago and has cytomegalovirus (CMV) retinitis

b. Transfusion reactions are characterized as a type II (cytotoxic) reaction in which agglutination and cytolysis occur.

A patient's low hemoglobin and hematocrit have necessitated a transfusion of packed red blood cells (PRBCs). Shortly after the first unit of PRBCs is hung, the patient develops signs and symptoms of a transfusion reaction. Which of the following hypersensitivity reactions has the patient experienced? A. Type I B. Type II C. Type III D. Type IV

After undergoing testing, a client comes to a physician's office for a follow-up appointment. During the appointment, the physician informs the client that she has systemic lupus erythematosus (SLE) . Which resource might be helpful for a nurse to recommend to this client?

A support group for clients with SLE

A healthy 65-year-old man who lives at home is at the clinic requesting a "flu shot." When assessing the patient, what other vaccinations should the nurse ask the patient about receiving (select all that apply)? A. Shingles B. Pneumonia C. Meningococcal D. Haemophilus influenzae type b (Hib) E. Measles, mumps, and rubella (MMR)

A. Shingles B. Pneumonia The patient should receive the shingles (heres zoster) vaccine, Pneumovax, and influenza. The other options do not apply to this patient. Meningococcal vaccination is recommended for adults at risk (e.g., adults with anatomic or functional asplenia or persistent complement component deficiencies). Adults born before 1957 are generally considered immune to measles and mumps. Haemophilus influenzae type b (Hib) vaccination is only considered for adults with selected conditions (e.g., sickle cell disease, leukemia, HIV infection or for those who have anatomic or functional asplenia) if they have not been previously vaccinated.

A patient who is receiving an IV antibiotic develops wheezes and dyspnea. In which order should the nurse implement these prescribed actions? (Put a comma and a space between each answer choice [A, B, C, D, E]). a. Discontinue the antibiotic infusion. b. Give diphenhydramine (Benadryl) IV. c. Inject epinephrine (Adrenalin) IM or IV. d. Prepare an infusion of dopamine (Intropin). e. Start 100% oxygen using a nonrebreather mask

ANS: A, E, C, B, D The nurse should initially discontinue the antibiotic because it is the likely cause of the allergic reaction. Next, oxygen delivery should be maximized, followed by treatment of bronchoconstriction with epinephrine administered IM or IV. Diphenhydramine will work more slowly than epinephrine, but will help prevent progression of the reaction. Because the patient currently does not have evidence of hypotension, the dopamine infusion can be prepared last

After being admitted to the hospital with sickle cell crisis, a client asks a nurse how he can prevent another crisis. Which response by the nurse is best?

"Make sure that you drink plenty of fluids."

A patient whose mother has been diagnosed with BRCA gene-related breast cancer asks the nurse, "Do you think I should be tested for the gene?" Which response by the nurse is most appropriate?

"There are many things to consider before deciding to have genetic testing."

Ten days after receiving a bone marrow transplant, a patient develops a skin rash. What would the nurse suspect is the cause of this patient's skin rash? a. The donor T cells are attacking the patient's skin cells. b. The patient's antibodies are rejecting the donor bone marrow. c. The patient is experiencing a delayed hypersensitivity reaction. d. The patient will need treatment to prevent hyperacute rejection.

ANS: A The patient's history and symptoms indicate that the patient is experiencing graft-versus-host disease, in which the donated T cells attack the patient's tissues. The history and symptoms are not consistent with rejection or delayed hypersensitivity

A client is admitted to the facility with an exacerbation of her chronic systemic lupus erythematosus (SLE). She gets angry when her call bell isn't answered immediately. The nurse's most appropriate response to her would be:

"You seem angry. Would you like to talk about it?"

A man with mild hemophilia asks the nurse, "Will my children be hemophiliacs?" Which response by the nurse is appropriate?

"Your female children will be carriers for hemophilia."

The nurse is assessing a client who has small groups of vesicles over his chest and upper abdominal area. They are located only on the right side of his body. The client states his pain level is 8/10, and describes the pain as burning in nature. Which question is most appropriate to include in the data collection?

1. "Did you have chicken pox as a child?"

A nurse determines that the neutropenic client needs further discharge teaching if which of the following statements is made by the client?

1. "I will include plenty of fresh fruits in my diet."

A nurse is assisting in developing a plan of care for a client with acquired immunodeficiency syndrome (AIDS) who is experiencing night fever and night sweats. Which nursing intervention should the nurse suggest including in the plan of care to manage this symptom?

1. Keep the call bell within reach for the client. 2. Administer a sedative at bedtime. 3. Administer an antipyretic at bedtime.. Provide a back

The camp nurse prepares to instruct a group of children about Lyme disease. Which of the following information would the nurse include in the instructions?

1. Lyme disease is caused by a tick carried by deer.

The nurse is assisting in planning care for a client with a diagnosis of immune deficiency. The nurse would incorporate which of the following as a priority in the plan of care?

1. Protecting the client from infection

A client with acquired immunodeficiency syndrome (AIDS) is experiencing shortness of breath related to Pneumocystis jiroveci pneumonia. Which measure should the nurse suggest to assist the client in performing activities of daily living?

1. Provide supportive care with hygiene needs.

A client arrives at the health care clinic requesting to be tested for Lyme disease. The client tells the nurse that he removed the tick and flushed it down the toilet. Which nursing action is appropriate?

1. Refer the client for a blood test immediately. 2. Inform the client that the tick is needed to perform a test. 3. Arrange for the client to return in 4 to 6 weeks to be tested. 4. Ask the client to describe the size, shape, and color of the tick. Answer: 3 Rationale: There is a blood test available to detect Lyme disease; however, it is not a reliable test if performed before 4 to 6 weeks following the tick bite. Options 1, 2, and 4 are inaccurate.

The client with acquired immunodeficiency syndrome has raised, dark purplish lesions on the trunk of the body. The nurse anticipates that which of the following procedures will be done to confirm whether these lesions are due to Kaposi's sarcoma?

1. Skin biopsy

A nurse is providing dietary instructions to a client with systemic lupus erythematosus. Which of the following dietary items would the nurse instruct the client to avoid?

1. Steak

The client with acquired immunodeficiency syndrome is diagnosed with cutaneous Kaposi's sarcoma. Based on this diagnosis, the nurse understands that this has been confirmed by which of the following?

1. Swelling in the genital area 2. Swelling in the lower extremities 3. Punch biopsy of the cutaneous lesions 4. Appearance of reddish-blue lesions on the skin Answer: 3 Rationale: Kaposi's sarcoma lesions begin as red, dark blue, or purple macules on the lower legs that change into plaques. These large plaques ulcerate or open and drain. The lesions spread by metastasis through the upper body and then to the face and oral mucosa. They can move to the lymphatic system, lungs, and gastrointestinal tract. Late disease results in swelling and pain in the lower extremities, penis, scrotum, or face. Diagnosis is made by punch biopsy of cutaneous lesions and biopsy of pulmonary and gastrointestinal lesions.

A client with acquired immunodeficiency syndrome (AIDS) is taking didanosine (Videx). The client calls the nurse at the health care provider's office and reports nausea, vomiting, and abdominal pain. Which of the following instructions would the nurse provide to the client?

1. This is an expected side effect of the medication. 2. Come to the office to be seen by the health care provider. 3. Take crackers and milk with the administration of the medication. 4. Decrease the dose of the medication until the next health care provider's visit. Answer: 2 Rationale: Pancreatitis, which can be fatal, is the major dose-limiting toxicity associated with the administration of didanosine. Clients should be monitored for indications of developing pancreatitis, which include increased serum amylase in association with increased serum triglycerides, decreased serum calcium, and nausea, vomiting, or abdominal pain. If evolving pancreatitis is diagnosed, the medication should be discontinued. The client should be seen by the health care provider.

A client is positively diagnosed with stage 1 Lyme disease. The client asks the nurse about the treatment for the disease. The nurse responds to the client, anticipating that which of the following will be part of the treatment plan?

1. Ultraviolet light therapy 2. No treatment unless symptoms develop 3. Treatment with intravenous (IV) penicillin G 4. A 3- to 4-week course of oral antibiotic therapy Answer: 4 Rationale: A 3- to 4-week course of oral antibiotic therapy is recommended during stage 1. Later stages of Lyme disease may require therapy with intravenous antibiotics, such as penicillin G. Ultraviolet light therapy is not a component of the treatment plan for Lyme disease.

19. Which question is least useful in the assessment of a client with AIDS? a. Are you a drug user? b. Do you have many sex partners? c. What is your method of birth control? d. How old were you when you became sexually active?

19. Answer D. Drug use is a risk factor for AIDS. Multiple sex partners is a risk factor for AIDS. Birth control methods are important to prevent a baby from being born with the AIDS virus. The age at which sexual activity began it not relevant as it does not usually provide information that identifies the presence of risk factors for AIDS.

A client arrives at the ambulatory care center complaining of flulike symptoms. On data collection, the client tells the nurse that he was bitten by a tick and is concerned that the bite is causing the sick feelings. The client requests a blood test to determine the presence of Lyme disease. Which of the following questions should the nurse ask next?

2. "When were you bitten by the tick?"

A client with pemphigus is being seen in the clinic regularly. The nurse plans care based on which of the following descriptions of this condition?

2. An autoimmune disease that causes blistering in the epidermis

The client is diagnosed with stage I of Lyme disease. The nurse assesses the client for which characteristic of this stage?

2. Flu-like symptoms

A nurse is identifying clients in the community at risk for latex allergy. Which client population is most at risk for developing this type of allergy?

2. Individuals with spina bifida Answer: 2 Rationale: Individuals at risk for developing a latex allergy include health care workers; individuals who work with manufacturing latex products; individuals with spina bifida; individuals who wear gloves frequently such as food handlers, hairdressers, and auto mechanics; and individuals allergic to kiwis, bananas, pineapples, passion fruit, avocados, and chestnuts.

A nurse is providing instructions to a client with acquired immunodeficiency syndrome (AIDS) who is experiencing night fever and night sweats. The nurse advises the client to do which of the following to increase comfort while minimizing symptoms?

2. Keep liquids on the nightstand at home.

A nurse prepares to give a MMR to a 5 yr old. What route should this be given? 1. Sub Q in the gluteal muscle 2. IM in the deltoid 3. subq in the outer aspect of the upper arm 4. IM in the anterolateral aspect of the thigh

3 MMR is given subq in the outer aspect of the upper arm

A Cub Scout leader who is a nurse is preparing a group of Cub Scouts for an overnight camping trip and instructs the scouts about the methods to prevent Lyme disease. Which statement by one of the Cub Scouts indicates a need for further instructions?

3. "I should not use insect repellent because it will attract the ticks."

A client diagnosed with Lyme disease says to the nurse, "I heard this disease can affect the heart. Is this true?" The nurse should make which response to the client?

3. "It can, but you will be monitored closely for cardiac complications."

The nurse is assigned to care for a client with systemic lupus erythematosus (SLE). The nurse plans care knowing that this disorder is:

3. An inflammatory disease of collagen contained in connective tissue

A client arrives at the health care clinic requesting to be tested for Lyme disease. The client tells the nurse that he removed the tick and flushed it down the toilet. Which nursing action is appropriate?

3. Arrange for the client to return in 4 to 6 weeks to be tested.

The nurse is assigned to care for a client admitted to the hospital with a diagnosis of systemic lupus erythematosus (SLE). The nurse reviews the health care provider's prescriptions. Which of the following medications would the nurse expect to be prescribed?

3. Corticosteroid. Rationale: Treatment of SLE is based on the systems involved and symptoms. Treatment normally consists of anti-inflammatory drugs, corticosteroids, and immunosuppressants. The incorrect options are not standard components of medication therapy for this disorder.

A client weighing 158 lb is ordered to receive 5 mg/kg of cyclosporine (Sandimmune) daily. How many milligrams should the client receive?

360

A client with human immunodeficiency virus (HIV) who has contracted tuberculosis (TB) asks the nurse how long the medication therapy lasts. The nurse responds that the duration of therapy would likely be for at least:

4. 9 total months and at least 6 months after cultures convert to negative

A client is positively diagnosed with stage 1 Lyme disease. The client asks the nurse about the treatment for the disease. The nurse responds to the client, anticipating that which of the following will be part of the treatment plan?

4. A 3- to 4-week course of oral antibiotic therapy

The nurse is assisting in administering immunizations at a health care clinic. The nurse understands that immunization provides which of the following?

4. Acquired immunity from disease

A nurse is providing general information to a group of high school students about preventing human immunodeficiency virus (HIV) transmission. The nurse would inform the students that which of the following is an unsafe behavior?

4. Use of natural skin condoms

A client is positively diagnosed with stage 1 Lyme disease. The client asks the nurse about the treatment for the disease. The nurse responds to the client, anticipating that which treatment will be included in the care plan?

A 3- to 4- week course of A 3- to 4- week course of oral antibiotic therapy

Which nursing intervention is appropriate for a client diagnosed with idiopathic thrombocytopenia purpura (ITP)?

Administering stool softeners, as ordered, to prevent straining during defecation

Which action takes priority for a client who is experiencing a hypersensitivity reaction to latex?

Administering supplemental oxygen

The physician orders tests to determine if a client has systemic lupus erythematosus (SLE). Which test result confirms SLE?

An above-normal anti-deoxyribonucleic acid (DNA) test

Allergy

An over reaction of the immune system to a harmless antigen

The nurse is caring for a client with systemic lupus erythematosus (SLE) that is affecting the hematopoietic system. Based on this, which signs and symptoms should the nurse anticipate and collect data on? Select all that apply.

Anemia Splenomegaly Lymphadenopathy

3. Several clients are admitted to an adult medical unit. The nurse would ensure airborne precautions for a client with which of the following medical conditions? a. A diagnosis of AIDS and cytomegalovirus b. A positive PPD with an abnormal chest x-ray c. A tentative diagnosis of viral pneumonia d. Advanced carcinoma of the lung

Answer B. The client who must be placed in airborne precautions is the client with a positive PPD (purified protein derivative) who has a positive x-ray for a suspicious tuberculin lesion.

Anti-fungal Medication 1) Name the drug 2) indication 3) Adverse

Antifungal Medication 1) Fluconazole (Diflucan) 2) candidiasis 3) Adverse: - hearing loss - peripheral neuritis

Antiviral Medication 1) Name the drug 2) indication

Antiviral Medication 1) Acyclovir (Zovirax) 2) herpes

A client's blood studies reveal a deficiency in all of the blood's formed elements. The physician suspects that the client's bone marrow is failing to generate enough new cells. Which disorder is most likely affecting this client?

Aplastic anemia

For early detection of an anaphylactic reaction in a patient who has received allergen testing using the cutaneous scratch method, which action should the nurse take first?

Assess the arm at the site of the skin testing.

Which nursing intervention takes priority for a client infected with Pneumocystis carinii pneumonia?

Auscultating breath sounds

A client is diagnosed with rheumatoid arthritis, an autoimmune disorder. When teaching the client and family about autoimmune disorders, the nurse should provide which information?

Autoimmune disorders include connective tissue (collagen) disorders.

How can a nurse best protect herself after she experiences a minor allergic reaction to latex?

Avoid use of all latex products.

A patient's low hemoglobin and hematocrit have necessitated a transfusion of packed red blood cells (RBCs). Shortly after the first unit of RBCs starts to infuse, the patient develops signs and symptoms of a transfusion reaction. Which type of hypersensitivity reaction has the patient experienced? A. Type I B. Type II C. Type III D. Type IV

B. Type II Transfusion reactions are characterized as a type II (cytotoxic) reaction in which agglutination and cytolysis occur. Type I hypersensitivity reactions are IgE-mediated reactions to specific allergens (e.g., exogenous pollen, food, drugs, or dust). Type III reactions are immune-complex reactions that occur secondary to antigen-antibody complexes. Type IV reactions are delayed cell-mediated immune response reactions.

The nurse is observing a client receiving antiplatelet therapy for adverse reactions. Antiplatelet drugs most commonly produce which hypersensitivity reaction?

Bronchospasm

A 21-year-old student had taken amoxicillin once as a child for an ear infection. She is given an injection of Penicillin V and develops a systemic anaphylactic reaction. What manifestations would be seen first? A. Dyspnea B. Dilated pupils C. Itching and edema D. Wheal-and-flare reaction

C. Itching and edema A systemic anaphylactic reaction starts with edema and itching at the site of exposure to the antigen. Shock can rapidly develop with rapid, weak pulse; hypotension; dilated pupils; dyspnea, and possible cyanosis. The wheal-and-flare reaction occurs with a localized anaphylactic reaction such as a mosquito bite.

The patient with diabetes mellitus has been ill for some time with a severe lung infection needing corticosteroids and antibiotics. The patient does not feel like eating. The nurse understands that this patient is likely to develop A. major histoincompatibility. B. primary immunodeficiency. C. secondary immunodeficiency. D. acute hypersensitivity reaction.

C. secondary immunodeficiency. Secondary immunodeficiency is most commonly caused by immunosuppressive drugs, such as corticosteroids. It can also be caused by diabetes mellitus, severe infection, malnutrition, and chronic stress, all of which are present in this patient. The other options are not possible for this patient. Histoincompatibility occurs when the human leukocyte antigen (HLA) system of the donor is not compatible with the recipient's HLA genes. Primary immunodeficiency is rare and includes phagocytic defects, B cell deficiency, T cell deficiency, or a combination of B cell and T cell deficiency. Acute hypersensitivity reaction is an anaphylactic-type allergic reaction to an antigen.

The patient with diabetes mellitus has been ill for some time with a severe lung infection needing corticosteroids and antibiotics. The patient does not feel like eating. The nurse understands that this patient is likely to develop A. major histoincompatibility. B. primary immunodeficiency. C. secondary immunodeficiency. D. acute hypersensitivity reaction.

C. secondary immunodeficiency. Secondary immunodeficiency is most commonly caused by immunosuppressive drugs, such as corticosteroids. It can also be caused by diabetes mellitus, severe infection, malnutrition, and chronic stress, all of which are present in this patient. The other options are not possible for this patient. Histoincompatibility occurs when the human leukocyte antigen (HLA) system of the donor is not compatible with the recipient's HLA genes. Primary immunodeficiency is rare and includes phagocytic defects, B cell deficiency, T cell deficiency, or a combination of B cell and T cell deficiency. Acute hypersensitivity reaction is an anaphylactic-type allergic reaction to an antigen.

ANTIBIOTICS Cephalosporins common adverse

Cephalosporins Cefaclor & Cephalexin Common Adverse: - pseudomembranous colitis - superinfection

A client who agreed to become an organ donor is pronounced dead. What is the most important factor in selecting a transplant recipient?

Compatible blood and tissue types

A client is admitted to the medical-surgical floor with a suspected diagnosis of acute myeloid leukemia. A nurse discusses the client's condition in the hallway. This action by the nurse jeopardizes which of the following principles?

Confidentiality

The home care nurse is prescribing dressing supplies for a client who has an allergy to latex. The nurse asks the medical supply personnel to deliver which?

Cotton pads and silk tape

A clinic nurse periodically cares for a client diagnosed with acquired immunodeficiency syndrome. The nurse assesses for an early manifestation of Pneumocystis jiroveci infection by monitoring for which sign/symptom at each client visit?

Cough

Oxygen independent killing

Damage to microbial membranes destruction to the bacterial cell wall chelation of iron digestion of ingested organism

A complete blood cell count is performed on a client with systemic lupus erythematosus (SLE). The nurse should suspect that which finding will be reported from this blood test?

Decrease of all cell types

A client with anemia has been admitted to the medical-surgical unit. Which data collection findings are characteristic of iron-deficiency anemia?

Dyspnea, tachycardia, and pallor

Ten years after seroconversion, an HIV-infected patient has a CD4+ cell count of 800/µl and an undetectable viral load. What is the priority nursing intervention at this time?

Encourage adequate nutrition, exercise, and sleep.

Which factor is most important when providing care for a client with hemophilia?

Ensuring client safety

A client with human immunodeficiency virus (HIV) infection is preparing for discharge from the hospital when he complains to a nurse that he continually feels weak. How should the nurse intervene?

Explain to the client that he should schedule periods of rest throughout the day.

ANTIBIOTICS Fluoroquinalones Common Adverse

Fluoroquinalones Ciprofloxacin & Levofloxacin Common Adverse - bone marrow depression - crystaluria

The nurse is working in a support group for clients with acquired immunodeficiency syndrome (AIDS). Which point is most important for the nurse to stress?

Following safe-sex practices

b, e

In a person having an acute rejection of a transplanted kidney, which of the following would help the nurse understand the course of events (select all that apply): A. a new transplant could be considered B. acute rejection can be treated with OKT3 C. acute rejection usually leads to chronic rejection D. corticosteroids are the most successful drug used to treat acute rejection E. Acute rejection is common after a transplant and can be treated with drug therapy

d

In a type 1 hypersensitivity reaction, the primary immunologic disorder appears to be: A. binding of IgG to an antigen on the cell surface B. deposit of antigen-antibody complexes in small vessels C. release of cytokines to interact with specific antigens D. release of chemical mediators from IgE-bound mast cells and basophils

For a client with an exacerbation of rheumatoid arthritis, the physician prescribes the corticosteroid prednisone (Deltasone). When caring for this client, the nurse should monitor for which adverse drug reactions?

Increased weight, hypertension, and insomnia

The nurse is teaching a client who will be discharged soon how to change a sterile dressing on the right leg. During the teaching session, the nurse notices redness, swelling, and induration at the wound site. What do these signs suggest?

Infection

Phagocytosis

Kill microbes by oxygen dependent and independent mechanisms

Which iron-rich foods should the nurse encourage an anemic client requiring iron therapy to eat?

Lamb and peaches

ANTIBIOTICS Macrolides Common Adverse

Macrolides Azithromycin & Clarithromycin Common Adverse: - pseudomembranous colitis - superinfection

C5b, C6, C7 etc

Membrane attack complex

The nurse is preparing a client with systemic lupus erythematosus (SLE) for discharge. Which instruction should the nurse include in the teaching plan?

Monitor body temperature.

A licensed practical nurse (LPN) is coassigned with a registered nurse (RN) for the care of a client with hemophilia. The physician prescribes a blood transfusion for this client. Which task associated with blood transfusion is the responsibility of the LPN?

Monitoring the client during the transfusion

ANTIBIOTICS Monobactam Common Adverse

Monobactam Aztreonam Common Adverse - allergic reaction

A pregnant woman with a history of early chronic HIV infection is seen at the clinic. Which information will the nurse include when teaching the patient?

Most infants born to HIV-positive mothers are not infected with the virus.

rheumatoid arthritis

Non organ specific auto immunity immune complexes are deposited on the synovial membrane which leads to complement fixation and inflammation with cartilage degradation and bone demineralisation

c

One function of cell-mediated immunity is: A. formation of antibodies B. activation of the complement system C. surveillance for malignant cell changes D. opsonization of antigens to allow phagocytosis by neutrophils

The nurse is preparing to give the following medications to an HIV-positive patient who is hospitalized with Pneumocystis jiroveci pneumonia (PCP). Which is most important to administer at the right time?

Oral saquinavir (Inverase) to suppress HIV infection

A client diagnosed with systemic lupus erythematosus (SLE) comes to the emergency department with severe back pain. She reports that she first felt pain after manually opening her garage door and that she is taking prednisone (Deltasone) daily. What adverse effect of long-term corticosteroid therapy is most likely responsible for the pain?

Osteoporosis

The nurse is caring for a 32-year-old client admitted with pernicious anemia. Which set of findings should the nurse expect when collecting data on the client?

Pallor, tachycardia, and a sore tongue

Antiretroviral therapy (ART) is being considered for an HIV-infected patient who has a CD4+ cell count of 400/µl. Which factor is most important to consider when determining whether ART will be started for this patient?

Patient ability to comply with ART schedule

ANTIBIOTICS Penicillins Common Adverse

Penicillins Amoxicillin & Penicillin Common Adverse - anaphalactic reaction - superinfections

The physician prescribes didanosine (ddI [Videx]), 200 mg by mouth every 12 hours, for a client with acquired immunodeficiency syndrome (AIDS) who is intolerant to zidovudine (azidothymidine, AZT [Retrovir]). Which condition in the client's history warrants cautious use of this drug?

Peripheral neuropathy

A client who is receiving cyclosporine (Sandimmune) must practice good oral hygiene, including regular brushing and flossing of the teeth, to minimize gingival hyperplasia. Good oral hygiene also is essential to minimize gingival hyperplasia during long-term therapy with certain drugs. Which of the following drugs falls into this category?

Phenytoin (Dilantin)

A nurse administers etanercept (Enbrel) by subcutaneous injection to a client with ankylosing spondylitis. Which action should the nurse take to prevent a needle-stick injury?

Place the uncapped needle in the designated puncture-resistant container.

Which nursing intervention takes priority for a client with human immunodeficiency virus (HIV) infection?

Protecting the client from infection

A patient seen at the clinic with atopic dermatitis has a history of multiple allergies and several previous anaphylactic reactions. Which type of testing for allergens will the nurse anticipate for this patient?

Radioallergosorbent test (RAST)

Which action should the nurse take when a client diagnosed with human immunodeficiency virus (HIV) infection refuses treatment?

Recognize that the client might not be ready to make treatment decisions.

A client has been taking a decongestant for allergic rhinitis. During a follow-up visit, which of the following suggests that the decongestant has been effective?

Reduced sneezing

Two days after a client undergoes splenectomy, a nurse changes his abdominal dressings according to the physician's order. How should the nurse proceed with the dressing change?

Remove the soiled dressings using clean gloves.

A nurse is preparing a teaching plan for a client with sickle cell disease. She includes periods of rest in her plan. Why is this point important to include?

Rest relieves stress, which may precipitate sickle cell crisis.

Which information about patient and donor tissue typing results for a patient who needs a kidney transplant is most important for the nurse to communicate to the health care provider?

Results of patient-donor cross matching are positive.

A client is admitted to an acute care facility with a myocardial infarction. During the admission history, the nurse learns that the client also has hypertension and progressive systemic sclerosis. For a client with this disease, the nurse is most likely to assist in formulating which nursing diagnosis?

Risk for impaired skin integrity

A client in a late stage of acquired immunodeficiency syndrome (AIDS) shows signs of AIDS-related dementia. Which nursing diagnosis takes highest priority?

Risk for injury

Type 1 hypersensitivity

Soluble antigen causes mast cell degranulation granules release histamine and proteases histamine causes increased permeability proteases cause the breakdown of extracellular matrix asthma, sweet itch and heaves in horses

When taking a dietary history from a newly admitted client, the nurse should remember that which of the following foods is a common allergen?

Strawberries

ANTIBIOTICS Sulfonamides Common Adverse

Sulfonamides Sulfamethoxazole & Trimethoprim Common Adverse - photosensitivity - bone marrow depression

T cell tolerance

T cells which are unresponsive endothelial barrier splits up T cells from self antigens so self antigens are present in insufficient quantities to trigger an immune response lack of second signal suppression of the T cell responses by T cells

The client with rheumatoid arthritis reports GI irritation after taking piroxicam (Feldene). To prevent GI upset, the nurse should provide which instruction?

Take piroxicam with food or an antacid.

ANTIBIOTICS Tetracyclines Common adverse

Tetracyclines Doxycycline & Tetracycline Common Adverse: - teeth staining - superinfection

IgG (immunoglobulin gamma)

The antibody you produce when you are immune to something neutralises the antigen, fixes compliment high affinity due to maturation cant be secreted as it will be broken down by enzymes

d

The function of monocytes in immunity is related to their ability to: A. stimulate the production of T and B lymphocytes B. produce antibodies on exposure to foreign substances C. bind antigens and stimulate natural killer cell activation D. capture antigens by phagocytosis and present them to lymphocytes

Antigen

The infectious agent

d

The nurse advises a friend who asks him to administer his allergy shots that: A. it is illegal for nurses to administer injections outside of a medical setting B. he is qualified to do it if the friend has epinephrine in an injectible syringe provided with his extract C. avoiding the allergens is a much more effective way of controlling allergens, and allergy shots are not usually effective D. immunotherapy should only be administered in a setting where emergency equipment and drugs are available

a

The nurse is altered to possible anaphylactic shock immediately after a patient has received intramuscular penicillin by the development of: A. edema and itching at the injection site B. sneezing and itching of the nose and eyes C. a wheal-and-flare reaction at the injection site D. chest tightness and production of thick sputum

A nurse delegates the task of obtaining a blood sample to a nursing assistant trained in venipuncture. When delegating this task, the nurse should understand which delegation principle?

The nurse may delegate the task but she remains accountable for the delegated task.

A patient has a new prescription for cyclosporine after having a kidney transplant. Which information in the patient's health history has the most implications for planning patient teaching about the medication at this time?

The patient has a glass of grapefruit juice every day for breakfast.

Virgin B cells

They are antigen specific- when the leave the BM they produce antibodies which move to the cell surface and stick out

indirect Coombs' test

To detect maternal antibodies against fetal Rh-positive factor

A client just diagnosed with toxoplasmosis asks the nurse, "What is toxoplasmosis? How did I get it, and what do I have to do to get rid of it"? Which information should the nurse include in the reply? Select all that apply.

Toxoplasmosis is Toxoplasmosis is treated with sulfadiazine Pregnant people should not empty litter boxes Toxoplasmosis is an organisms found in rare pork Toxoplasmosis may cause a severe inflammatory response

A client with blood type B needs a blood transfusion. Which type of blood can this client receive?

Type B or type O blood

A client has had heavy menstrual bleeding for 6 months. Her gynecologist diagnoses microcytic hypochromic anemia and prescribes ferrous sulfate (Feosol), 300 mg by mouth daily. Before initiating iron therapy, the nurse reviews the client's medical history. Which condition would contraindicate the use of ferrous sulfate?

Ulcerative colitis

The nurse is providing general information to a group of high school students about preventing human immunodeficiency virus (HIV) transmission. The nurse should inform the students that which behavior is unsafe?

Use of natural skin condom

The nurse is caring for a client with thrombocytopenia. What is the best way to protect this client?

Use the smallest needle possible for injections.

The nurse provides care for a client with deep partial-thickness burns. What could cause a reduced hematocrit (HCT) in this client?

Volume overload

The nurse prepares to give a bath and change the bed linens on a client with cutaneous Kaposi's sarcoma lesions. The lesions are open and draining a scant amount of serous fluid. Which should the nurse incorporate in the plan during the bathing of this client?

Wearing a gown and gloves

A client seeks medical evaluation for fatigue, night sweats, and a 20-lb weight loss in 6 weeks. To confirm that the client has been infected with the human immunodeficiency virus (HIV), the nurse expects the physician to order:

Western blot test with ELISA.

Complement cascade trigger

When antibodies join to the organism

a, c, d, e

When caring for a patient with a known latex allergy, the nurse would monitor the patient closely for a cross-sensitivity to which of the following foods (select all that apply)? A. Grapes B. Oranges C. Bananas D. Potatoes E. Tomatoes

The role of the lymph node

Where an antigen goes to be caught by a dendritic cell

d

Which of the following accurately describes rejection following transplantation? A. hyperacute rejection can be treated with OKT3 B. acute rejections can be treated with sirolimus or tacrolimus C. chronic rejection can be treated with tacrolimus or cyclosporine D. hyperacute rejection can usually be avoided is crossmatching is done before the transplantation

class switching

a B cell that has undergone a class switch from IgM to IgG this cen then go to IgE or IgA however a switch from IgA cannot go back to IgG or IgE

The nurse reminds the staff that standard precautions should be used when providing care for which type of patient? a) All patients regardless of diagnosis b) Pediatric and gerontologic patients c) Patients who are immunocompromised d) Patients with a history of infectious diseases

a) All patients regardless of diagnosis Standard precautions are designed for all care of all patients in hospitals and health care facilities.

Choice Multiple question - Select all answer choices that apply. A 42-year-old client comes to the clinic and is diagnosed with shingles. Which findings confirm this diagnosis? Select all that apply. a) Fever b) Malaise c) Severe, deep pain around the thorax d) Diarrhea e) Red, nodular skin lesions around the thorax

a) Fever b) Malaise c) Severe, deep pain around the thorax e) Red, nodular skin lesions around the thorax Shingles, also called herpes zoster, is an acute unilateral and segmental inflammation of the dorsal root ganglia. It is caused by infection with the herpes virus varicella-zoster, the same virus that causes chickenpox. It commonly causes severe, deep pain along a peripheral nerve on the trunk of the body and red, nodular skin lesions. Fever and malaise typically accompany these findings. Diarrhea does not commonly occur with shingles.

An 11-year-old child contracted severe acute respiratory syndrome (SARS) when traveling abroad with her parents. The nurse knows she must put on personal protective equipment to protect herself while providing care. Based on the mode of SARS transmission, which personal protective equipment should the nurse wear? a) Gown, gloves, mask, and eye goggles or eye shield b) Gown, gloves, and mask c) Gloves d) Gown and gloves

a) Gown, gloves, mask, and eye goggles or eye shield The transmission of SARS isn't fully understood. Therefore, all modes of transmission must be considered possible, including airborne, droplet, and direct contact with the virus. For protection from contracting SARS, any health care worker providing care for a person with SARS should wear a gown, gloves, mask, and eye goggles or an eye shield.

A client develops a facial rash and urticaria after receiving penicillin. Which laboratory value does the nurse expect to be elevated? a) IgE b) IgG c) IgA d) IgB

a) IgE Immunoglobulin E (IgE) is involved with an allergic reaction. IgA combines with antigens and activates the complement system. IgB coats the surface of B lymphocytes. IgG is the principal immunoglobulin formed in response to most infectious agents.

which conditions cause severe pain in HIV and AIDS (Select all that apply) a. enlarged organs b. peripheral neuropathy c. tumors d. high fever e. dry skin

a, b, c

which actions are useful in helping orient a patient (Select all that apply) a. repeating person, place, time b. using clocks and calendars c. using MMSE screening test d. having familiar items present e. providing uninterrupted time

a, b, d

the nurse assesses a patient diagnosed with advanced AIDS for malnutirition. which findings does the nurse most likely assess (Select all that apply) a. pain b. anorexia c. urinary incontinence d. diarrhea e. vomiting

a, b, d, e

which opportunistic infections can be observed in AIDS (Select all that apply) A. toxicoplasmosis B. gastroenteritis C. TB D. candidiasis E. cytomegalovirus

a, c, d, e

Immunodeficiency

absence of an effective immune response`

A patient is admitted to the hospital with acute rejection of a kidney transplant. The nurse will anticipate ________________

administration of immunosuppressant medications.

Type 2 hypersensitivity

altered cell surface component IgG to bind to the novel antigen this triggers complement cascade igG binds to the cell surface receptor causes haemolytic anemia and autoimmunity

A client who is hospitalized with scleroderma signs a document that provides instructions concerning the provision of care if he is unable to make his own treatment decisions. The document is known as:

an advance directive.

After being stung by a wasp, a patient is brought to the clinic by a co-worker. Upon arrival the patient is anxious and having difficulty breathing. The first action that the nurse should take is to ______________

assess the patient's airway.

A client with autoimmune thrombocytopenia and a platelet count of 8,000/μl develops epistaxis and melena. Treatment with corticosteroids and immunoglobulins has been unsuccessful, and the physician recommends a splenectomy. The client states, "I don't need surgery — this will go away on its own." In considering her response to the client, the nurse must depend on the ethical principle of:

autonomy.

HAART causes what effects? A. reversal of a patients antibody status B. decrease of the viral load C. increase of the viral load D. moe delectable HIV

b

What type of precautions should the nurse take for a patient suspected of having TB as a result of HIV? A. universal b. airborne c. enteric d. protective isolation

b

the nurse is teaching a patient about preventing infection through sex. which statement indicates effective teaching? a. latex condom with spermicide proves the best protection b. mutually monogamous sex with a non infected partner will best prevent HIV c. contraceptive methods like implants and injections are recommended to prevent HIV transmission d. if my partner and i are both HIV positive, unprotected sex is permitted

b

Choice Multiple question - Select all answer choices that apply. Which vaccine should a nurse encourage a client with chronic obstructive pulmonary disease (COPD) to receive? Select all that apply. a) Varicella b) Pneumonia c) Hepatitis B d) Pertussis e) Influenza

b) Pneumonia d) Pertussis e) Influenza Clients with COPD are more susceptible to respiratory infections, so they should be encouraged to receive the influenza, pneumococcal, and pertussis vaccines. Clients with COPD are not at high risk for varicella or hepatitis B.

A nurse is caring for a client with the following laboratory values: white blood cell count (WBC) 4,500/mm3, neutrophils 15%, and bands 1%. Based on the client's absolute neutrophil count (ANC), the nurse knows that the clients risk for infection is: a) No increased risk b) Significant risk c) low risk d) intermediate risk

b) Significant risk In practical clinical terms, a normal ANC is 1.5 or higher; a "safe" ANC is 500-1500; a low ANC is less than 500. A safe ANC means that the patient's activities do not need to be restricted (on the basis of the ANC).

which statements about HIV are accurate? (Select all that apply) A. may be acquired or congenital B. it is retrovirus C. it always progresses to AIDS D. it is a virus that attacks the immune system E. it is a parasite that forces cells to make copies of itself

b, d, e

which immune function abnormalities are a result of HIV infection? (Select all that apply) A. lymphocytosis B. CD4+ depletion C. increased CD8+ activity D. long macrophage life span E. lymphocytopenia

b, e

The nurse was accidently stuck with a needle used on an HIV-positive patient. After reporting this, what care should this nurse first receive? b) Personal protective equipment c) Combination antiretroviral therapy d) Counseling to report blood exposures e) A negative evaluation by the manager

c) Combination antiretroviral therapy Postexposure prophylaxis with combination antiretroviral therapy can significantly decrease the risk of infection. Personal protective equipment should be available although it may not have stopped this needle stick. The needle stick has been reported. The negative evaluation may or may not be needed but would not occur first.

A nurse practicing in a nurse-managed clinic suspects that an 8-year-old child's chronic sinusitis and upper respiratory tract infections may result from allergies. Which laboratory test would the nurse most likely order? Select all that apply. a) Metabolic panel b) Rheumatoid factor c) Immunoglobulin assay (IgE) d) Liver function studies e) Complete blood count

c) Immunoglobulin assay (IgE) e) Complete blood count The nurse would order a complete blood count, which may indicate elevate white blood cells and eosinophils, as well as an immunoglobulin assay to look specifically for IgE elevations. Rheumatoid factor would be ordered for rheumatic disorders and Sjögren's syndrome; metabolic panel and liver function studies would not provide information about allergies.

A 5-year-old girl was diagnosed with type 1 diabetes mellitus. The mother says that no one else in her family has had diabetes and asks why her daughter would get it. How should the nurse explain this complex disease? a) It is a congenital disorder that she was born with. b) It is a single gene disorder, meaning only one gene mutation caused the disease. c) It is a multifactorial genetic disorder caused by one or more genes and environmental factors. d) It was an acquired genetic mutation, meaning she developed it, but her children will not have it.

c) It is a multifactorial genetic disorder caused by one or more genes and environmental factors. Type 1 diabetes mellitus is a multifactorial genetic disorder related to one or more gene mutations and potentially various environmental factors that alter the way the gene(s) work. Type 1 diabetes is not a single gene disorder nor an acquired genetic mutation.

a patient with HIV is receiving meds to reduce viral load and improve cd4+ counts. which term accurately describes this HIV drug regimen a. interferon treatment b. antiviremia c. ELISA administration d. HAART

d

cryptosporidiosis is a form of intestinal infection in which diarrhea can amount to a loss of how many liters of fluid per day? A. 1-2 B. 3-5 c. 5-8 d. 15-20

d

which is the most common route for HCP to contract HIV: A. blood B. bodily fluids C. mucous membranes D. needle sticks

d

which malignancy is most common in patients with HIV/AIDS a. non-hodgkins B cell lymphoma b. anal cancer c. primary brain cancer d. kaposi's sarcoma

d

which statements regarding HIV/AIDS among older adults are true? A. the risk for HIV infection after exposure is minimal for older adults B. older men are more susceptible to HIV C. it is not necessary to assess an older adult for history of drug use D. older adults who participate in high-risk behaviors are susceptible to HIV

d

which treatments are intended to boost the immune system? a. protease inhibitors b. hematopoietic growth factors c. lymphocyte transfusion d. interleukin-2 infusion

d

The nurse is caring for a client who is receiving antibiotics to treat a gram-negative bacterial infection. Because antibiotics destroy the body's normal flora, the nurse must monitor the client for:

diarrhea

While obtaining a health history, the nurse learns that the client is allergic to bee stings. When obtaining this client's medication history, the nurse should determine if the client keeps which medication on hand?

diphenhydramine hydrochloride (Benadryl)

In a client who has human immunodeficiency virus (HIV) infection, CD4+ levels are measured to determine the:

extent of immune system damage.

A client with end-stage acquired immunodeficiency syndrome (AIDS) has profound manifestations of Cryptosporidium infection caused by the protozoa. In planning the client's care, the nurse should focus on his need for:

fluid replacement.

A client takes prednisone (Deltasone), as prescribed, for rheumatoid arthritis. The nurse should tell the client to look for common adverse reactions to this drug, such as:

fluid retention and weight gain.

A patient seeks medical care after sharing needles for heroin injection with a friend who has hepatitis B. To provide immediate protection from infection, the nurse expects to administer _____________

gamma globulin.

Allergen

harmless antigen

camelids

have the different IgG forms

During the admission process, the nurse evaluates a client with rheumatoid arthritis. To assess for the most obvious disease manifestations first, the nurse checks for:

joint abnormalities.

A patient with HIV infection has developed Mycobacterium avium complex infection. An appropriate outcome for the patient is that the patient will ____________

maintain intact perineal skin.

A client with neutropenia has an absolute neutrophil count (ANC) of 900 (0.9 X 109/L). What is the client's risk of infection?

moderate risk Explanation: A client is at moderate risk when the ANC is less than 1,000 (1 × 109/L). The ANC decreases proportionately to the increased risk for infection. Normal risk for infection is when the ANC is 1,500 (1.5 × 109/L) or greater. High risk for infection is when the ANC is less than 500 (0.5 × 109/L). An ANC of 100 (0.1 × 109/L) or less is life threatening.

The nurse will monitor a patient who is undergoing plasmapheresis for ___________

numbness and tingling.

Canine Hyperthyroidism

organ specific immunity thyroid being destroyed

Antibodies

originate from the B cells in the foetal liver and bone marrow

Respiratory Burst

oxidative burst (rapid production of ROS

IgD

produced by B cells

Oxidative burst

rapid production of ROS

A client with thrombocytopenia, secondary to leukemia, develops epistaxis. The nurse should instruct the client to:

sit upright, leaning slightly forward.

A patient who has vague symptoms of fatigue and headaches is found to have a positive enzyme immunoassay (EIA) for human immunodeficiency virus (HIV) antibodies. In discussing the test results with the patient, the nurse informs the patient that ____________

the EIA test will need to be repeated to verify the results.

Epitope

the bit an antibody binds to can be linear or conformational

Oxygen dependent killing

the defense mechanism used by phagocytes to produce H2O2 and so kill infection

PCR of leukemic cells

they have a smaller fragment size that is more common

Tolerance

unresponsiveness that is specific to a particular antigen which is induced by previous expsure

Allotypic variation

variation in the amino acid sequence of the heavy and light chain genes

Choice Multiple question - Select all answer choices that apply. After teaching the parents of a child with celiac disease about diet, the nurse understands the teaching has been effective if the parents state that which foods should be avoided? Select all that apply. a) soy milk b) corn tortillas c) hot dog buns d) bologna on rye sandwich e) white rice

• hot dog buns • bologna on rye sandwich Explanation: Children with celiac disease should avoid foods containing the protein gluten, which is found in wheat, oats, rye, and barley grains. Hot dog buns, unless otherwise labeled, contain wheat.

1. A patient who seeks health care for vague symptoms of fatigue and headaches has HIV testing and is found to have a positive enzyme immunoassay (EIA) for HIV antibodies. In discussing the test results with the patient, the nurse informs the patient that a. the enzyme immunoassay test will need to be repeated to verify the results. b. a viral culture will be done to determine the progress of the disease. c. it will probably be 10 or more years before the patient develops AIDS. d. the Western blot test will need to be done to determine whether AIDS has developed.

a. the enzyme immunoassay test will need to be repeated to verify the results. b. a viral culture will be done to determine the progress of the disease. c. it will probably be 10 or more years before the patient develops AIDS. d. the Western blot test will need to be done to determine whether AIDS has developed. A Rationale: After an initial positive EIA test, the EIA is repeated before more specific testing such as the Western blot is done. Viral cultures are not part of HIV testing. Because the nurse does not know how recently the patient was infected, it is not appropriate to predict the time frame for AIDS development. The Western blot tests for HIV antibodies, not for AIDS. Cognitive Level: Application Text Reference: p. 256 Nursing Process: Implementation NCLEX: Physiological Integrity

A patient who is receiving an IV antibiotic develops wheezes and dyspnea. In which order should the nurse implement these prescribed actions?

1) Discontinue the antibiotic infusion. 2) Start 100 % oxygen using a nonrebreather mask. 3) Inject epinephrine (Adrenalin) IM or IV. 4) Give diphenhydramine (Benadryl) IV. 5) Prepare an infusion of dopamine (Intropin).

PCR of antibody producing B cells

all the frangments of DNA from the B cell are differnet

The nurse is assessing a client who has small groups of vesicles over his chest and upper abdominal area. They are located only on the right side of his body. The client states his pain level is 8/10, and describes the pain as burning in nature. Which question is most appropriate to include in the data collection?

1. "Did you have chicken pox as a child?" 2. "How many sexual partners have you had?" 3. "Did you use an electric blanket on your side?" 4. "Why don't you try docosanol cream (Abreva) on your lesions?" Answer: 1 Rationale: The client has the symptoms of herpes zoster, or shingles, which is caused by the same organism as chicken pox. Asking about sexual partners is inappropriate for this disorder. An electric blanket use does not cause this type of lesions. Abreva is used on herpes simplex I (cold sores).

The nurse provides home care instructions to a client with systemic lupus erythematosus and tells the client about methods to manage fatigue. Which statement by the client indicates a need for further instructions?

1. "I should take hot baths because they are relaxing."

A nurse determines that the neutropenic client needs further discharge teaching if which of the following statements is made by the client?

1. "I will include plenty of fresh fruits in my diet." 2. "If I develop a fever over 100° F, I will call my doctor." 3. "Petting my dog is fine as long as I wash my hands after doing so." 4. "My husband will just have to take over cleaning the cat's litter box." Answer: 1 Rationale: Fresh fruits and vegetables are eliminated from the diet to avoid the introduction of pathogens. Fever of 100.4° F or greater should be reported immediately. Feeding and petting cats and dogs are fine as long as handwashing follows. Handling pet excrement must be avoided to avoid exposure to pathogens.

A client arrives at the ambulatory care center complaining of flulike symptoms. On data collection, the client tells the nurse that he was bitten by a tick and is concerned that the bite is causing the sick feelings. The client requests a blood test to determine the presence of Lyme disease. Which of the following questions should the nurse ask next?

1. "Was the tick small or large?" 2. "When were you bitten by the tick?" 3. "Did you save the tick for inspection?" 4. "Did the tick bite anyone else in the family?" Answer: 2 Rationale: There is a blood test available to detect Lyme disease; however, it is not a reliable test if performed before 4 to 6 weeks following the tick bite. The appropriate question by the nurse should elicit information related to when the tick bite occurred.

A nurse is providing information to a client with systemic lupus erythematosus (SLE) about dietary alterations. The nurse should remind the client to avoid which of the following foods?

1. Chicken 2. Beef 3. Melons 4. Cauliflower Answer: 2 Rationale: The client with SLE is at risk for cardiovascular disorders, such as coronary artery disease and hypertension. The client is advised of lifestyle changes to reduce these risks, which include smoking cessation and prevention of obesity and hyperlipidemia. The client is advised to reduce intake of salt, fat, and cholesterol.

A health care provider aspirates synovial fluid from a knee joint of a client with rheumatoid arthritis. The nurse reviews the laboratory analysis of the specimen and would expect the results to indicate which finding?

1. Cloudy synovial fluid

The community health nurse is conducting a research study and is identifying clients in the community who are at risk for latex allergy. Which client population is at most risk for developing this type of allergy?

1. Hairdressers and individuals allergic to kiwis, bananas, pineapples, tropical fruits, grapes, avocados, potatoes, hazelnuts, and water chestnuts.

The community health nurse is conducting a research study and is identifying clients in the community who are at risk for latex allergy. Which client population is at most risk for developing this type of allergy?

1. Hairdressers 2. The homeless 3. Children in day care centers 4. Individuals living in a group home Answer: 1 Rationale: Individuals at risk for developing a latex allergy include health care workers; individuals who work in the rubber industry; individuals having multiple surgeries; individuals with spina bifida; individuals who wear gloves frequently such as food handlers, hairdressers, and auto mechanics; and individuals allergic to kiwis, bananas, pineapples, tropical fruits, grapes, avocados, potatoes, hazelnuts, and water chestnuts.

The nurse is assisting in administering immunizations at a health care clinic. The nurse understands that immunization provides which of the following?

1. Protection from all diseases 2. Innate immunity from disease 3. Natural immunity from disease 4. Acquired immunity from disease Answer: 4 Rationale: Acquired immunity can occur by receiving an immunization that causes antibodies to a specific pathogen to form. Natural (innate) immunity is present at birth. No immunization protects the client from all diseases.

A client is diagnosed with stage 1 Lyme disease. The nurse checks the client for which hallmark characteristic of this stage?

1. Signs of neurological disorders 2. Enlarged and inflamed joints 3. Headache 4. Skin rash Answer: 4 Rationale: The hallmark of stage 1 is the development of a skin rash that occurs within 2 to 30 days of infection, generally at the site of the tick bite. The remaining options are not initially related to this pathology.

A nurse is providing dietary instructions to a client with systemic lupus erythematosus. Which of the following dietary items would the nurse instruct the client to avoid?

1. Steak 2. Turkey 3. Broccoli 4. Cantaloupe Answer: 1 Rationale: The client with systemic lupus erythematosus is at risk for cardiovascular disorders such as coronary artery disease and hypertension. The client is advised of lifestyle changes to reduce these risks, which include smoking cessation and prevention of obesity and hyperlipidemia. The client is advised to reduce salt, fat, and cholesterol intake.

A client with acquired immunodeficiency syndrome has a respiratory infection from Pneumocystis jiroveci and a nursing diagnosis of Impaired Gas Exchange written in the plan of care. Which of the following indicates that the expected outcome of care has not yet been achieved?

1. The client limits fluid intake.

A client with acquired immunodeficiency syndrome has a respiratory infection from Pneumocystis jiroveci and a nursing diagnosis of Impaired Gas Exchange written in the plan of care. Which of the following indicates that the expected outcome of care has not yet been achieved?

1. The client limits fluid intake. 2. The client has clear breath sounds. 3. The client expectorates secretions easily. 4. The client is free of complaints of shortness of breath. Answer: 1 Rationale: The status of the client with a nursing diagnosis of Impaired Gas Exchange would be evaluated against the standard outcome criteria for this nursing diagnosis. These would include that the client breathes easier, coughs up secretions effectively, and has clear breath sounds. The client should not limit fluid intake because fluids are needed to decrease the viscosity of secretions for expectoration.

A nurse is collecting data on a client with rheumatoid arthritis. The nurse looks at the client's hands and notes these characteristic deformities. The nurse identifies this deformity as: Refer to figure.

1. Ulnar drift 2. Rheumatoid nodules 3. Swan neck deformity 4. Boutonniere deformity Answer: 1 Rationale: All of the conditions identified in the options can occur in rheumatoid arthritis. Ulnar drift occurs when synovitis stretches and damages the tendons, and eventually the tendons become shortened and fixed. This damage causes subluxation (drift) of the joints.

Which interventions would apply in the care of a client at high risk for an allergic response to a latex allergy. Select all that apply.

1. Use non-latex gloves. 2. Use medications from glass ampules. 4. Do not puncture rubber stoppers with needles. 5. Keep a latex-safe supply cart available in the client's area.

17. Jayson, 1 year old child has a staph skin infection. Her brother has also developed the same infection. Which behavior by the children is most likely to have caused the transmission of the organism? a. Bathing together. b. Coughing on each other. c. Sharing pacifiers. d. Eating off the same plate.

17. Answer A. Direct contact is the mode of transmission for staphylococcus. Staph is not spread by coughing. Staph is not spread through oral secretions. Direct contact is required. Staph is not spread through oral secretions.

A nurse is providing information to a client with systemic lupus erythematosus (SLE) about dietary alterations. The nurse should remind the client to avoid which of the following foods?

2. Beef

A client with acquired immunodeficiency syndrome (AIDS) is taking didanosine (Videx). The client calls the nurse at the health care provider's office and reports nausea, vomiting, and abdominal pain. Which of the following instructions would the nurse provide to the client?

2. Come to the office to be seen by the health care provider.

A client in the clinical unit who is allergic to shellfish unknowingly ate a dish brought by a friend that had shellfish as an ingredient. The client quickly develops anaphylaxis. The nurse would focus on which of the following first until additional help arrives?

3. Maintaining a patent airway

The client with acquired immunodeficiency syndrome is diagnosed with cutaneous Kaposi's sarcoma. Based on this diagnosis, the nurse understands that this has been confirmed by which of the following?

3. Punch biopsy of the cutaneous lesions

A 4 month old comes to clinic for shots. The child is up to date. What vaccines should the kid get at this time? 1. varicella, HEP B 2. DTaP, MMR, IPV 3. MMR, Hib, DTaP 4. DTaP, hib, PCV, RV

4 DTap, hib, PVC and RV are given at 4 months

An infant receives a a DTaP immuinization at a well baby clinic. The parent returns home and calls the clinic and reports swelling & redness at the injection site. Which is an appropriate action? 1. Monitor the infant for a fever 2. Bring the infant back to clinic 3. Apply hot pack 4. Apply cold pack

4 * On occasion tenderness, swelling and redness appear at the site of a DTaP injection. Cold packs for the first 24 hrs, followed by cold or warm compresses if inflammation persist.

Which client is at the highest risk for systemic lupus erythematous (SLE)?

4. An African-American female

A 34-year-old female patient who has systemic lupus erythematosus is receiving plasmapheresis to treat an acute attack. What symptoms will the nurse monitor to determine if the patient develops complications related to the procedure? A. Hypotension, paresthesias, and dizziness B. Polyuria, decreased reflexes, and lethargy C. Intense thirst, flushed skin, and weight gain D. Abdominal cramping, diarrhea, and leg weakness

A. Hypotension, paresthesias, and dizziness Common complications associated with plasmapheresis are hypotension and citrate toxicity. Citrate is used as an anticoagulant and may cause hypocalcemia, which may manifest as headache, paresthesias, and dizziness. Polyuria, decreased reflexes, and lethargy are symptoms of hypercalcemia. Abdominal cramping, diarrhea, and leg weakness indicate hyperkalemia. Intense thirst, flushed skin, and weight gain indicate hypernatremia with normal or increased extracellular fluid volume.

A 34-year-old female patient who has systemic lupus erythematosus is receiving plasmapheresis to treat an acute attack. What symptoms will the nurse monitor to determine if the patient develops complications related to the procedure? A. Hypotension, paresthesias, and dizziness B. Polyuria, decreased reflexes, and lethargy C. Intense thirst, flushed skin, and weight gain D. Abdominal cramping, diarrhea, and leg weakness

A. Hypotension, paresthesias, and dizziness Common complications associated with plasmapheresis are hypotension and citrate toxicity. Citrate is used as an anticoagulant and may cause hypocalcemia, which may manifest as headache, paresthesias, and dizziness. Polyuria, decreased reflexes, and lethargy are symptoms of hypercalcemia. Abdominal cramping, diarrhea, and leg weakness indicate hyperkalemia. Intense thirst, flushed skin, and weight gain indicate hypernatremia with normal or increased extracellular fluid volume.

A 62-year-old patient has acquired immunodeficiency syndrome (AIDS), and the viral load is reported as undetectable. What patient teaching should be provided by the nurse related to this laboratory study result? A. The patient has the virus present and can transmit the infection to others. B. The patient is not able to transmit the virus to others through sexual contact. C. The patient will be prescribed lower doses of antiretroviral medications for 2 months. D. The syndrome has been cured, and the patient will be able to discontinue all medications.

A. The patient has the virus present and can transmit the infection to others. In human immunodeficiency virus (HIV) infections, viral loads are reported as real numbers of copies/μL or as undetectable. "Undetectable" indicates that the viral load is lower than the test is able to report. "Undetectable" does not mean that the virus has been eliminated from the body or that the individual can no longer transmit HIV to others.

The nurse teaches a patient about drug therapy after a kidney transplant. Which statement by the patient would indicate a need for further instructions? a. "After a couple of years, it is likely that I will be able to stop taking the cyclosporine." b. "If I develop an acute rejection episode, I will need to have other types of drugs given IV." c. "I need to be monitored closely because I have a greater chance of developing malignant tumors." d. "The drugs are given in combination because they inhibit different ways the kidney can be rejected."

ANS: A Cyclosporine, a calcineurin inhibitor, will need to be continued for life. The other patient statements are accurate and indicate that no further teaching is necessary about those topics

Which information would be most important to help the nurse determine if the patient needs human immunodeficiency virus (HIV) testing? a. Patient age b. Patient lifestyle c. Patient symptoms d. Patient sexual orientation

ANS: A The current Center for Disease Control (CDC) policy is to offer routine testing for HIV to all individuals age 13 to 64. Although lifestyle, symptoms, and sexual orientation may suggest increased risk for HIV infection, the goal is to test all individuals in this age range

To evaluate the effectiveness of antiretroviral therapy (ART), which laboratory test result will the nurse review? a. Viral load testing b. Enzyme immunoassay c. Rapid HIV antibody testing d. Immunofluorescence assay

ANS: A The effectiveness of ART is measured by the decrease in the amount of virus detectable in the blood. The other tests are used to detect HIV antibodies, which remain positive even with effective ART

A patient who has received allergen testing using the cutaneous scratch method has developed itching and swelling at the skin site. Which action should the nurse take first? a. Administer epinephrine. b. Apply topical hydrocortisone. c. Monitor the patient for lower extremity edema. d. Ask the patient about exposure to any new lotions or soaps.

ANS: A The initial symptoms of anaphylaxis are itching and edema at the site of the exposure. Hypotension, tachycardia, dilated pupils, and wheezes occur later. Rapid administration of epinephrine when excessive itching or swelling at the skin site is observed can prevent the progression to anaphylaxis. Topical hydrocortisone would not deter an anaphylactic reaction. Exposure to lotions and soaps does not address the immediate concern of a possible anaphylactic reaction. The nurse should not wait and observe for edema. The nurse should act immediately in order to prevent progression to anaphylaxis

An older adult patient who is having an annual check-up tells the nurse, "I feel fine, and I don't want to pay for all these unnecessary cancer screening tests!" Which information should the nurse plan to teach this patient? a. Consequences of aging on cell-mediated immunity b. Decrease in antibody production associated with aging c. Impact of poor nutrition on immune function in older people d. Incidence of cancer-stimulating infections in older individuals

ANS: A The primary impact of aging on immune function is on T cells, which are important for immune surveillance and tumor immunity. Antibody function is not affected as much by aging. Poor nutrition can also contribute to decreased immunity, but there is no evidence that it is a contributing factor for this patient. Although some types of cancer are associated with specific infections, this patient does not have an active infection

Which statement by a patient would alert the nurse to a possible immunodeficiency disorder? a. "I take one baby aspirin every day to prevent stroke." b. "I usually eat eggs or meat for at least 2 meals a day." c. "I had my spleen removed many years ago after a car accident." d. "I had a chest x-ray 6 months ago when I had walking pneumonia."

ANS: C Splenectomy increases the risk for septicemia from bacterial infections. The patient's protein intake is good and should improve immune function. Daily aspirin use does not affect immune function. A chest x-ray does not have enough radiation to suppress immune function

While obtaining a health history from a patient, the nurse learns that the patient has a history of allergic rhinitis and multiple food allergies. Which action by the nurse is most appropriate? a. Encourage the patient to carry an epinephrine kit in case a type IV allergic reaction to latex develops. b. Advise the patient to use oil-based hand creams to decrease contact with natural proteins in latex gloves. c. Document the patient's allergy history and be alert for any clinical manifestations of a type I latex allergy. d. Recommend that the patient use vinyl gloves instead of latex gloves in preventing blood-borne pathogen contact.

ANS: C The patient's allergy history and occupation indicate a risk of developing a latex allergy. The nurse should be prepared to manage any symptoms that may occur. Epinephrine is not an appropriate treatment for contact dermatitis that is caused by a type IV allergic reaction to latex. Oil-based creams will increase the exposure to latex from latex gloves. Vinyl gloves are appropriate to use when exposure to body fluids is unlikely

A patient who is human immunodeficiency virus (HIV)-infected has a CD4+ cell count of 400/µL. Which factor is most important for the nurse to determine before the initiation of antiretroviral therapy (ART) for this patient? a. HIV genotype and phenotype b. Patient's social support system c. Potential medication side effects d. Patient's ability to comply with ART schedule

ANS: D Drug resistance develops quickly unless the patient takes ART medications on a strict, regular schedule. In addition, drug resistance endangers both the patient and the community. The other information is also important to consider, but patients who are unable to manage and follow a complex drug treatment regimen should not be considered for ART

A patient with human immunodeficiency virus (HIV) infection has developed Mycobacterium avium complex infection. Which outcome would be appropriate for the nurse to include in the plan of care? a. The patient will be free from injury. b. The patient will receive immunizations. c. The patient will have adequate oxygenation. d. The patient will maintain intact perineal skin.

ANS: D The major manifestation of M. avium infection is loose, watery stools, which would increase the risk for perineal skin breakdown. The other outcomes would be appropriate for other complications (pneumonia, dementia, influenza, etc.) associated with HIV infection

2. A client is diagnosed with methicillin resistant staphylococcus aureus pneumonia. What type of isolation is MOST appropriate for this client? a. Reverse isolation b. Respiratory isolation c. Standard precautions d. Contact isolation

Answer D. Contact or Body Substance Isolation (BSI) involves the use of barrier protection (e.g. gloves, mask, gown, or protective eyewear as appropriate) whenever direct contact with any body fluid is expected. When determining the type of isolation to use, one must consider the mode of transmission. The hands of personnel continues to be the principal mode of transmission for methicillin resistant staphylococcus aureus (MRSA). Because the organism is limited to the sputum in this example, precautions are taken if contact with the patient"s sputum is expected. A private room and BSI, along with good hand washing techniques, are the best defense against the spread of MRSA pneumonia

A 62-year-old patient who is having an annual check-up tells the nurse, "I don't understand why I need to have so many cancer screening tests now. I feel just fine!" The nurse will plan to teach the patient about the _______________

consequences of aging on cell-mediated immunity.

A patient who receives weekly immunotherapy at a clinic missed the previous appointment. When the patient comes for the next injection, the nurse should _______________

consult with the health care provider about giving a lower allergen dose.

B cell tolerance

B cells are exposed to high levels of multivalent self antigens and this leads to clonal deletion low level antigen exposure makes the B cells unresponsive by down regulation of the surface IgM expression - the B cells are short lived-clonal anergy

A patient has begun immunotherapy for the treatment of intractable environmental allergies. When administering the patient's immunotherapy, what is the nurse's priority action? A. Monitor the patient's fluid balance. B. Assess the patient's need for analgesia. C. Monitor for signs and symptoms of an adverse reaction. D. Assess the patient for changes in level of consciousness.

C. Monitor for signs and symptoms of an adverse reaction. When administering immunotherapy, it is imperative to closely monitor the patient for any signs of an adverse reaction. The high risk and significant consequence of an adverse reaction supersede the need to assess the patient's fluid balance. Pain and changes in level of consciousness are not likely events when administering immunotherapy.

A 19-year-old male being tested for multiple allergies develops localized redness and swelling in reaction to a patch skin test. Which intervention by the nurse would have the highest priority? A. Notify the primary care provider B. Apply a topical antiinflammatory cream C. Remove the patch and extract from the skin D. Administer oral diphenhydramine (Benadryl)

C. Remove the patch and extract from the skin If a severe reaction to a patch skin test occurs, the nurse should immediately remove the patch and the extract from the skin. Next the nurse should apply a topical antiinflammatory cream to the site. A subcutaneous injection of epinephrine may also be necessary but would need a health care provider's order.

To determine whether a patient's angioedema has responded to prescribed therapies, which action should the nurse take first?

Check for swelling of the lips and tongue.

During posttest counseling for a patient who has positive testing for HIV, the patient is anxious and does not appear to hear what the nurse is saying. At this time, it is most important that the nurse a. inform the patient how to protect sexual and needle-sharing partners. b. teach the patient about the medications available for treatment. c. ask the patient to notify individuals who have had risky contact with the patient. d. remind the patient about the need to return for retesting to verify the results.

D Rationale: After an initial positive antibody test, the next step is retesting to confirm the results. A patient who is anxious is not likely to be able to take in new information or be willing to disclose information about HIV status of other individuals. Cognitive Level: Application Text Reference: pp. 256, 264 Nursing Process: Implementation NCLEX: Psychosocial Integrity

Which patient is at highest risk for developing graft-versus-host disease? A. A 65-year-old man who received an autologous blood transfusion B. A 40-year-old man who received a kidney transplant from a living donor C. A 65-year-old woman who received a pancreas and kidney from a deceased donor D. A 40-year-old woman who received a bone marrow transplant from a close relative

D. A 40-year-old woman who received a bone marrow transplant from a close relative Graft-versus-host disease occurs when an immunoincompetent patient is transfused or transplanted with immunocompetent cells. Examples include blood transfusions or the transplantation of bone marrow, fetal thymus, or fetal liver. An autologous blood transfusion is the collection and reinfusion of the individual's own blood or blood components. There is no risk for graft-versus-host disease in this situation.

Ten days after receiving a bone marrow transplant, a patient develops a skin rash on his palms and soles, jaundice, and diarrhea. What is the most likely etiology of these clinical manifestations? A. The patient is experiencing a type I allergic reaction. B. An atopic reaction is causing the patient's symptoms. C. The patient is experiencing rejection of the bone marrow. D. Cells in the transplanted bone marrow are attacking the host tissue.

D. Cells in the transplanted bone marrow are attacking the host tissue. The patient's symptoms are characteristic of graft-versus-host-disease (GVHD) in which transplanted cells mount an immune response to the host's tissue. GVHD is not a type I allergic response or an atopic reaction, and it differs from transplant rejection in that the graft rejects the host rather than the host rejecting the graft.

A 52-year-old female patient was exposed to human immunodeficiency virus (HIV) 2 weeks ago through sharing needles with other substance users. What symptoms will the nurse teach the patient to report that would indicate the patient has developed an acute HIV infection? A. Cough, diarrhea, headaches, blurred vision, muscle fatigue B. Night sweats, fatigue, fever, and persistent generalized lymphadenopathy C. Oropharyngeal candidiasis or thrush, vaginal candidal infection, or oral or genital herpes D. Flu-like symptoms such as fever, sore throat, swollen lymph glands, nausea, or diarrhea

D. Flu-like symptoms such as fever, sore throat, swollen lymph glands, nausea, or diarrhea Clinical manifestations of an acute infection with HIV include flu-like symptoms between 2 to 4 weeks after exposure. Early chronic HIV infection clinical manifestations are either asymptomatic or include fatigue, headache, low-grade fever, night sweats, and persistent generalized lympadenopathy. Intermediate chronic HIV infection clinical manifestations include candidal infections, shingles, oral or genital herpes, bacterial infections, Kaposi sarcoma, or oral hairy leukoplakia. Late chronic HIV infection or acquired immunodeficiency syndrome (AIDS) includes opportunistic diseases (infections and cancer).

The nurse is providing postoperative care for a 30-year-old female patient after an appendectomy. The patient has tested positive for human immunodeficiency virus (HIV). What type of precautions should the nurse observe to prevent the transmission of this disease? A. Droplet precautions B. Contact precautions C. Airborne precautions D. Standard precautions

D. Standard precautions Standard precautions are indicated for prevention of transmission of HIV to the health care worker. HIV is not transmitted by casual contact or respiratory droplets. HIV may be transmitted through sexual intercourse with an infected partner, exposure to HIV-infected blood or blood products, and perinatal transmission during pregnancy, at delivery, or though breastfeeding.

Which signs/symptoms would indicate to the nurse that a client is experiencing an anaphylactic reaction? Select all that apply.

Hives Stridor Dyspnea Urticaria Wheezing

A 56-year-old client diagnosed with acquired immunodeficiency syndrome (AIDS) is admitted with a closed head injury after being found unconscious on the kitchen floor by her neighbor. The staff suspects domestic abuse, based on information supplied by the neighbor that the client has a restraining order against the husband, who repeatedly tries to visit the client. Which nursing action ensures client safety?

Inform hospital security personnel of the restraining order and formulate an action plan with security that protects the client.

A 33-year-old client who tested positive for the human immunodeficiency virus (HIV) is admitted to the medical unit with pancreatitis. A nurse director from another unit comes into the medical unit nurses' station and begins reading the client's chart. The staff nurse questions the director, who says that the client is her neighbor's son. What should the nurse do to protect the client's right to privacy?

Inform the nurse director that she's violating the client's right to privacy and ask her to return the chart.

ANTIBIOTICS Lincosamides Common Adverse

Lincosamides Clindamycin Common Adverse - bone marrow suppression

A client with acquired immunodeficiency syndrome (AIDS) is admitted with Pneumocystis carinii pneumonia. During a bath, the client begins to cry and says that most friends and relatives have stopped visiting and calling. What should the nurse do?

Listen and show interest as the client expresses feelings.

Which symptoms should the nurse expect and monitor for in clients with Stage 4 human immunodeficiency virus (HIV) infection? Select all that apply.

Lymphoma Kaposi sarcoma Candidiasis of the esophagus

the patient with HIV/AIDS appears emaciated and has diarrhea, anorexia, mouth lesions, and peristent weight loss. what condition does the nurse suspect this patient is developing? a. AIDS dementia B. AIDS wasting syndrome C. AIDS GI opportunistic infection D. AIDS candidiasis opportunistic infection

b

what is the most important means of preventing HIV spread? A. engineering B. education C. isolation D. counseling

b

which definition of immunodeficiency is accurate? A. disease/deficiency acquired as a result of viral infection, contact with toxin, or medical therapy B. deficient immune response as a result of imapired or missing immune components C. chronic infection wih immunodeficiency virus D. disease/deficiency pesent since birth

b

The nurse is reviewing the chart for a patient who is scheduled for an annual physical exam. Which information will be most important in determining whether the patient needs HIV testing?

Patient age

For which individual is genetic carrier screening indicated? a) A patient with a history of type 1 diabetes b) A patient with a family history of sickle cell disease c) A patient whose mother and sister died of breast cancer d) A patient who has a long-standing history of iron-deficiency anemia

b) A patient with a family history of sickle cell disease Genetic carrier screening should be done in families with a history of sickle cell disease. Iron deficiency is not related to genetic status. Although there is a multifactorial genetic basis for diabetes (based on research), there are no genetic markers for testing for the presence of or risk of diabetes. A significant family history of breast cancer may suggest that presymptomatic testing for BRCA 1 and 2 may be indicated.

The patient is admitted to the ED with fever, swollen lymph glands, sore throat, headache, malaise, joint pain, and diarrhea. What nursing measures will help identify the need for further assessment of the cause of this patient's manifestations (select all that apply)? a) Assessment of lung sounds b) Assessment of sexual behavior c) Assessment of living conditions d) Assessment of drug and syringe use e) Assessment of exposure to an ill person

b) Assessment of sexual behavior d) Assessment of drug and syringe use With these symptoms, assessing this patient's sexual behavior and possible exposure to shared drug equipment will identify if further assessment for the HIV virus should be made or the manifestations are from some other illness (e.g., lung sounds and living conditions may indicate further testing for TB).

The nurse is caring for a patient newly diagnosed with HIV. The patient asks what would determine the actual development of AIDS. The nurse's response is based on the knowledge that what is a diagnostic criterion for AIDS? a) Presence of HIV antibodies b) CD4+ T cell count below 200/µL c) Presence of oral hairy leukoplakia d) White blood cell count below 5000/µL

b) CD4+ T cell count below 200/µL Diagnostic criteria for AIDS include a CD4+ T cell count below 200/µL and/or the development of specified opportunistic infections, cancers, wasting syndrome, or dementia. The other options may be found in patients with HIV disease but do not define the advancement of HIV infection to AIDS.

The patient has vancomycin-resistant enterococci (VRE) infection in a surgical wound. What infection precautions should the nurse use to best prevent transmission of the infection to the nurse, other patients, staff, and those outside the hospital? a) Droplet precautions b) Contact precautions c) Airborne precautions d) Standard precautions

b) Contact precautions Contact precautions are used to minimize the spread of pathogens that are acquired from direct or indirect contact. Droplet precautions are used with pathogens that are spread through the air at close contact and that affect the respiratory system or mucous membranes (e.g., influenza, pertussis). Airborne precautions are used if the organism can cause infection over long distances when suspended in the air (e.g., TB, rubeola). Standard precautions are used with all patients and included in the transmission-based precautions above.

A 45-year-old woman asks the nurse, "If my sister has the mutated BRCA gene, what does it mean for me?" What information should the nurse consider for an accurate response to this question? a) Only an identical twin would be at risk for breast cancer. b) Family members are at high risk of developing breast cancer. c) The patient already has breast cancer but does not know it yet. d) Family members will not have any treatment options for breast cancer.

b) Family members are at high risk of developing breast cancer. Inheriting a mutated BRCA1 or BRCA2 gene confers significant risk of developing breast cancer. In addition, it significantly increases family members' risk of developing the disease. Having the BRCA gene does not mean that breast cancer is already present or that a future cancer could not be treated.

A hospital has seen a recent increase in the incidence of hospital care-associated infections (HAIs). Which measure should be prioritized in the response to this trend? a) Use of gloves during patient contact b) Frequent and thorough hand washing c) Prophylactic, broad-spectrum antibiotics d) Fitting and appropriate use of N95 masks

b) Frequent and thorough hand washing Hand washing remains the mainstay of the prevention of HAIs. Gloves, masks, and antibiotics may be appropriate in specific circumstances, but none of these replaces the central role of vigilant, thorough hand washing between patients and when moving from one task to another, even with the same patient.

Type 3 hypersensitivity

Soluble antigen effector mechanisms- immune complex, complement, phagocytes reaction - farmers lung, arthurs reaction

A client with rheumatoid arthritis is being discharged with a prescription for aspirin (Ecotrin), 600 mg by mouth every 6 hours. The nurse should instruct the client to notify the physician if which adverse drug reaction occurs?

Tinnitus

A 25-year-old male patient has been diagnosed with HIV. The patient does not want to take more than one antiretroviral drug. What reasons can the nurse tell the patient about for taking more than one drug? a) Together they will cure HIV. b) Viral replication will be inhibited. c) They will decrease CD4+ T cell counts. d) It will prevent interaction with other drugs.

b) Viral replication will be inhibited. The major advantage of using several classes of antiretroviral drugs is that viral replication can be inhibited in several ways, making it more difficult for the virus to recover and decreasing the likelihood of drug resistance that is a major problem with monotherapy. Combination therapy also delays disease progression and decreases HIV symptoms and opportunistic diseases. HIV cannot be cured. CD4+ T cell counts increase with therapy. There are dangerous interactions with many antiretroviral drugs and other commonly used drugs.

which statement about the transmission of HIV is true? (Select all that apply) A. can only be transmitted during end stage B. those with recent HIV infection and high viral load are very infectious C. those with end stage HIV and no drug therapy are very infectious D. HIV is only transmitted with sexual contact E. all people infected with HIV will quickly progress to AIDS

b, c

Choice Multiple question - Select all answer choices that apply. The nurse is caring for a client who has been newly diagnosed with systemic lupus erythematosus (SLE). Which information would be included in a teaching plan that focuses on home care? Select all that apply. a) Keep exercise to a minimal level. b) Avoid over-the-counter (OTC) medications unless approved by the physician. c) Avoid exposure to sunlight. d) Take rest periods as needed. e) Report development of a butterfly rash on the face.

a) Keep exercise to a minimal level. b) Avoid over-the-counter (OTC) medications unless approved by the physician. c) Avoid exposure to sunlight. d) Take rest periods as needed. Explanation: The client who suffers from SLE has a tendency toward photosensitivity; therefore, the client would avoid exposure to sunlight. The client would also be advised to keep exercise to a minimum, to avoid OTC medications unless directed by the physician, and to rest as needed. Because the butterfly rash associated with lupus is an initial sign, the client would already have the rash and would not be reporting its development after discharge.

The couple is delivering their first child. What newborn genetic screening should the nurse teach them about (select all that apply)? a) Pheylketonuria b) Dienoyl-CoA reductase c) Polycystic kidney disease d) Congenital hypothyroidism e) Hereditary nonpolyposis colorectal cancer syndrome

a) Pheylketonuria d) Congenital hypothyroidism Newborn genetic screening for phenylketonuria (PKU) and congenital hypothyroidism are universally required by law in the United States. Genetic screening for dienoyl-CoA reductase, a fatty acid disorder, is required in some states. Hereditary nonpolyposis colorectal cancer syndrome and polycystic kidney disease would only be tested for if there was family history and the family requested it.

When administering a blood transfusion to a client with multiple traumatic injuries, the nurse monitors closely for evidence of a transfusion reaction. Shortly after the transfusion begins, the client complains of chest pain, nausea, and itching and there is a rise in the client's temperature. The nurse stops the transfusion and notifies the physician. The nurse suspects which type of hypersensitivity reaction with a blood transfusion? a) Type II (cytolytic, cytotoxic) hypersensitivity reaction b) Type IV (cell-mediated, delayed) hypersensitivity reaction c) Type I (immediate, anaphylactic) hypersensitivity reaction d) Type III (immune complex) hypersensitivity reaction

a) Type II (cytolytic, cytotoxic) hypersensitivity reaction ABO (blood type) incompatibility, such as from an incompatible blood transfusion, is a type II hypersensitivity reaction. Transfusions of more than 100 ml of incompatible blood can cause severe and permanent renal damage, circulatory shock, and even death. Drug-induced hemolytic anemia is another example of a type II reaction. A type I hypersensitivity reaction occurs in anaphylaxis, atopic diseases, and skin reactions. A type III hypersensitivity reaction occurs in Arthus reaction, serum sickness, systemic lupus erythematosus, and acute glomerulonephritis. A type IV hypersensitivity reaction occurs in tuberculosis, contact dermatitis, and transplant rejection.

A parent of a child with acute post-streptococcal glomerulonephritis (APSCN) asks how streptococcal infection caused the child to have a kidney problem. What is the nurse's best response? a) "The streptococcal infection spread through the bloodstream to your child's kidneys." b) "By-products of immune complexes that fought the infection are depositing in the kidneys." c) "The strep infection weakened your child's immune system, making him susceptible to a secondary infection." d) "Your child made excessive antibodies to fight the infection that are now attacking the kidneys."

b) "By-products of immune complexes that fought the infection are depositing in the kidneys." Explanation: APSGN is an immune complex disease. Large antigen-antibody complexes are formed that deposit in the glomerular capillary loops leading to obstruction. APSGN is considered an autoimmune disorder, not an infection. Antibodies do not attack the kidneys in this disorder.

A nurse practicing in a nurse-managed clinic suspects that an 8-year-old child's chronic sinusitis and upper respiratory tract infections may result from allergies. She orders an immunoglobulin assay. Which immunoglobulin would the nurse expect to find elevated? a) Immunoglobulin M b) Immunoglobulin E c) Immunoglobulin D d) Immunoglobulin G

b) Immunoglobulin E The nurse would expect elevated immunoglobulin (Ig) E levels because IgE is predominantly found in saliva and tears as well as intestinal and bronchial secretions and, therefore, may be found in allergic disorders. IgD's physiologic function is unknown and constitutes only 1% of the total number of circulating immunoglobulins. IgG is elevated in the presence of viral and bacterial infections. IgM is the first antibody activated after an antigen enters the body, and is especially effective against gram-negative organisms.

A nurse encourages a client with an immunologic disorder to eat a nutritionally balanced diet to promote optimal immunologic function. Which snacks have the greatest probability of stimulating autoimmunity? a) Applesause and dried apricots b) Potato chips and chocolate milk shakes c) Raisins and carrot sticks d) Fruit salad and mineral water

b) Potato chips and chocolate milk shakes A diet containing excessive fat, such as that found in potato chips and milk shakes, seems to contribute to autoimmunity — overreaction of the body against constituents of its own tissues. Raisins, carrot sticks, fruit, mineral water, applesauce, and dried apricots are snacks containing adequate amounts of vitamin A, zinc, and carotene, which are beneficial for the body.

A 34-year-old primigravid client at 39 weeks' gestation admitted to the hospital in active labor has type B Rh-negative blood. The nurse should instruct the client that if the neonate is Rh positive, the client will receive an Rh immune globulin (RHIG) injection for what reason? a) to provide active antibody protection for this pregnancy b) to prevent Rh-positive sensitization with the next pregnancy c) to decrease the amount of Rh-negative sensitization for the next pregnancy d) to destroy fetal Rh-positive cells during the next pregnancy

b) to prevent Rh-positive sensitization with the next pregnancy The purpose of the RhoGAM is to provide passive antibody immunity and prevent Rh-positive sensitization with the next pregnancy. It should be given within 72 hours after birth of an Rh-positive neonate. Clients who are Rh-negative and conceive an Rh-negative fetus do not need antibody protection. Rh-positive cells contribute to sensitization, not Rh-negative cells. The RhoGAM does not cross the placenta and destroy fetal Rh-positive cells.

Which of the following clients are at an increased risk of developing Kaposi's sarcoma skin lesions? Select all that apply. a) Client status post a liver transplant b) Client with acquired immunodeficiency syndrome (AIDS) c) Client with type 1 diabetes mellitus d) Female client of European ancestry e) Male client of Mediterranean/Jewish ancestry

b) • Client with acquired immunodeficiency syndrome (AIDS) a) • Client status post a liver transplant e) • Male client of Mediterranean/Jewish ancestry Explanation: Clients with a compromised immune system (such as transplant clients or those with AIDS) are at an increased risk of developing Kaposi's sarcoma. Kaposi's sarcoma is also more prevalent among males of Mediterranean or Jewish ancestry, although in a less severe form. Clients with type 1 diabetes mellitus or females of European ancestry are not at an increased risk.

The HIV positive patient tells the nurse that his HIV negative partner will be using preexposure drugs (Truvada). which statement indicates the need for additional teaching? A. my partner will need to be tested q3m B. this drug will decrease the chances of my partner becoming positive C. once we start using Truvada I will no longer need a condom D. my partner will need to be monitored for any side effects on this drug

c

An infant is to receive the diphtheria, tetanus, and acellular pertussis (DTaP) and inactivated polio vaccine (IPV) immunizations. The child is recovering from a cold and is afebrile. The child's sibling has cancer and is receiving chemotherapy. Which action is most appropriate? a) Postpone both immunizations until the sibling is in remission. b) Withhold both immunizations until the infant is well. c) Administer the DTaP and IPV immunizations. d) Give the DTaP and withhold the IPV.

c) Administer the DTaP and IPV immunizations. At this time, the infant can be given the vaccines. The fact that the child's sibling is immunosuppressed because of chemotherapy is not a reason to withhold the vaccines. The fact that the child has a cold is not grounds for delaying the immunizations. However, if the child had a high fever, the immunizations would be delayed.

Rho (D) immune globulin (RhoGAM) is prescribed for a client before she is discharged after a spontaneous abortion. The nurse instructs the client that this drug is used to prevent which condition? a) an antibody response to Rh-negative blood b) development of a future Rh-positive fetus c) development of Rh-positive antibodies d) a future pregnancy resulting in abortion

c) development of Rh-positive antibodies Rh sensitization can be prevented by Rho(D) immune globulin, which clears the maternal circulation of Rh-positive cells before sensitization can occur, thereby blocking maternal antibody production to Rh-positive cells. Administration of this drug will not prevent future Rh-positive fetuses, nor will it prevent future abortions. An antibody response will not occur to Rh-negative cells. Rh-negative mothers do not develop sensitivities if the fetus is also Rh negative.

Twenty-four hours after undergoing kidney transplantation, a client develops a hyperacute rejection. To correct this problem, the nurse should prepare the client for: a) intra-abdominal instillation of methylprednisolone sodium succinate. b) bone marrow transplant. c) removal of the transplanted kidney. d) high-dose I.V. cyclosporine therapy.

c) removal of the transplanted kidney. Hyperacute rejection isn't treatable; the only way to stop this reaction is to remove the transplanted organ or tissue. Although cyclosporine is used to treat acute transplant rejection, it doesn't halt hyperacute rejection. Bone marrow transplant isn't effective against hyperacute rejection of a kidney transplant. Methylprednisolone sodium succinate may be given I.V. to treat acute organ rejection, but it's ineffective against hyperacute rejection.

C3b

complement the receptors on phagocytic cells opsonisation -(identifying the particle to the pagocyte) immune complex formation

The nurse encourages a client with an immunologic disorder to eat a nutritionally balanced diet to promote optimal immunologic function. Autoimmunity has been linked to excessive ingestion of:

fat.

Active immunity

results from direct exposure of an antigen by immunization or disease exposure.

Secondary response

the antigen is attached to a dendrite cell and then a memory cell/helper t cell complex attaches to the antigen also and this signals to B cells to produce plasma cells and secrete a specific antibody and also to produce more memory cells

B cell development

the be cell is selected then with the help of t cells there is rapid proliferation and then with t cells there is differentiation and maturation

After teaching a patient on immunosuppressant therapy after a kidney transplant about the posttransplant drug regimen, the nurse determines that additional teaching is needed when the patient says, ____________

"After a couple of years, it is likely that I will be able to stop taking the calcineurin inhibitor."

The nurse discusses the prevention and management of allergic reactions with a beekeeper who has developed a hypersensitivity to bee stings. The nurse identifies a need for additional teaching when the patient states, ________________

"I will plan to take oral antihistamines daily before going to work."

A client with pernicious anemia is receiving parenteral vitamin B12 therapy. Which client statement indicates effective teaching about this therapy?

"I will receive parenteral vitamin B12 therapy for the rest of my life."

A nurse is teaching high school students about transmission of the human immunodeficiency virus (HIV). Which comment by a student warrants clarification by the nurse?

"I won't donate blood because I don't want to get AIDS."

In teaching a female client who is HIV-positive about pregnancy and the human immunodeficiency virus (HIV), the nurse would know more teaching is necessary when the client says:

"I'll need to have a C-section if I become pregnant and have a baby."

How should a nurse respond when asked by a family member of a client with human immunodeficiency virus (HIV) infection why she's performing passive range-of-motion (ROM) exercises on the client?

"These exercises help prevent contractures by keeping his joints mobile."

A 27-year-old client with end-stage acquired immunodeficiency syndrome (AIDS) is being cared for by his wife at home. The hematologist recommends hospice care and the couple agrees. During the initial admission visit, the hospice nurse provides information to the client and his family about advance directives. At the next visit, the client states that since he and his wife filled out the advance directive form he feels abandoned by his physician. Which statement by the hospice nurse best addresses the client 's concerns?

"Your physician will continue to care for you. Advance directives document in writing your wishes regarding your care in case you're unable to communicate them to the physician yourself."

(SELECT ALL THAT APPLY) A client has undergone total gastrectomy due to stomach cancer. Which nursing interventions are necessary for this client immediately after surgery?

(3) Observe the wound for redness, swelling, and warmth, (4) Encourage incentive spirometry use every hour during the client's waking hours., (5) Administer opioid analgesics as prescribed.

(SELECT ALL THAT APPLY) The nurse is planning care for a client with human immunodeficiency virus (HIV). Which statement by the nurse indicates her understanding of HIV transmission?

(4) "I will wear a mask, gown, and gloves when splashing of bodily fluids is likely.", (5) "I will wash my hands after client care."

When caring for a patient who has just been diagnosed with early chronic HIV infection, which prophylactic measures will the nurse anticipate being included in the plan of care (

-Hepatitis B vaccine -Pneumococcal vaccine -Influenza virus vaccine

Drag and Drop question - Click and drag the following steps to place them in the correct order. Question: During a school party a child with a known food allergy has an itchy throat, is wheezing, and reports not feeling "quite right." The nurse should do the following in what order from first to last? All options must be used. 1 Assess vital signs. 2 Position to facilitate breathing. 3 Send someone to activate the Emergency Management Systems (EMS). 4 Administer the child's epinephrine. 5 Notify the parents.

1) Send someone to activate the Emergency Management Systems (EMS). 2) Administer the child's epinephrine. 3) Position to facilitate breathing. 4) Assess vital signs. 5) Notify the parents. Explanation: The child is exhibiting signs of anaphylaxis. The principles of emergency management involve activating EMS when an emergency is first realized. The nurse then follows the priorities of Circulation, Airway, Breathing (C, A, B). The epinephrine should then be given to reduce airway constriction and prevent cardiovascular collapse. The child should be assisted into the most comfortable position to facilitate breathing, usually with the head elevated. Then the nurse can take the child's vital signs to assess the effectiveness of the treatment. Lastly, the nurse should notify the family.

A Cub Scout leader who is a nurse is preparing a group of Cub Scouts for an overnight camping trip and instructs the scouts about the methods to prevent Lyme disease. Which statement by one of the Cub Scouts indicates a need for further instructions?

1. "I need to bring a hat to wear during the trip." 2. "I should wear long-sleeved tops and long pants." 3. "I should not use insect repellent because it will attract the ticks." 4. "I need to wear closed shoes and socks that can be pulled up over my pants." Answer: 3 Rationale: In the prevention of Lyme disease, individuals need to be instructed to use an insect repellent on the skin and clothes when in an area where ticks are likely to be found. Long-sleeved tops and long pants, closed shoes, and a hat or cap should be worn. If possible, one should avoid heavily wooded areas or areas with thick underbrush. Socks can be pulled up and over the pant legs to prevent ticks from entering under clothing.

Indinavir (Crixivan) is prescribed for a client with human immunodeficiency virus (HIV). The nurse has provided instructions to the client regarding ways to maximize absorption of the medication. Which of the following, if stated by the client, indicates an adequate understanding of the use of this medication?

1. "I need to take the medication with my large meal of the day." 2. "I need to store the medication in the refrigerator." 3. "I need to take the medication with water but on an empty stomach." 4. "I need to take the medication with a high-fat snack." Answer: 3 Rationale: To maximize absorption, the medication should be administered with water on an empty stomach. The medication can be taken 1 hour before a meal or 2 hours after a meal, or it can be administered with skim milk, coffee, tea, or a low-fat meal such as cornflakes with skim milk and sugar. It is not to be administered with a large meal. The medication should be stored at room temperature and protected from moisture because moisture can degrade the medication.

The nurse provides home care instructions to a client with systemic lupus erythematosus and tells the client about methods to manage fatigue. Which statement by the client indicates a need for further instructions?

1. "I should take hot baths because they are relaxing." 2. "I should sit whenever possible to conserve my energy." 3. "I should avoid long periods of rest because it causes joint stiffness." 4. "I should do some exercises, such as walking, when I am not fatigued." Answer: 1 Rationale: To help reduce fatigue in the client with systemic lupus erythematosus, the nurse should instruct the client to sit whenever possible, avoid hot baths (because they exacerbate fatigue), schedule moderate low-impact exercises when not fatigued, and maintain a balanced diet. The client is instructed to avoid long periods of rest because it promotes joint stiffness.

A client diagnosed with Lyme disease says to the nurse, "I heard this disease can affect the heart. Is this true?" The nurse should make which response to the client?

1. "Where did you get your information?" 2. "Yes, that's true but it rarely ever occurs." 3. "It can, but you will be monitored closely for cardiac complications." 4. "It primarily affects the joints with the occasional facial paralysis." Answer: 3 Rationale: Stage 2 of Lyme disease develops within 1 to 6 months in the majority of untreated individuals. The serious problems that occur in this stage include cardiac conduction defects and neurological disorders, such as Bell's palsy and paralysis. The remaining options are either untrue or do not effectively address the client's concern.

A client with human immunodeficiency virus (HIV) who has contracted tuberculosis (TB) asks the nurse how long the medication therapy lasts. The nurse responds that the duration of therapy would likely be for at least:

1. 6 total months and at least 1 month after cultures convert to negative 2. 6 total months and at least 3 months after cultures convert to negative 3. 9 total months and at least 3 months after cultures convert to negative 4. 9 total months and at least 6 months after cultures convert to negative Answer: 4 Rationale: The client with tuberculosis who is coinfected with HIV requires that antitubercular therapy last longer than usual. The prescription is usually for a total of 9 months and at least 6 months after sputum cultures convert to negative.

A nurse reads the chart of a client who has been diagnosed with stage 3 Lyme disease. Which clinical manifestation supports this diagnosis?

1. A generalized skin rash 2. A cardiac dysrhythmia 3. Complaints of joint pain 4. Paralysis of a facial muscle Answer: 3 Rationale: Stage 3 develops within a month to several months after initial infection. It is characterized by arthritic symptoms, such as arthralgia and enlarged or inflamed joints, which can persist for several years after the initial infection. Cardiac and neurological dysfunction occurs in stage 2. A rash occurs in stage 1. Paralysis of the extremity where the tick bite occurred is not a characteristic of Lyme disease.

Which of the following individuals is least likely at risk for the development of Kaposi's sarcoma?

1. A kidney transplant client 2. A male with a history of same-sex partners 3. A client receiving antineoplastic medications 4. An individual working in an environment where exposure to asbestos exists Answer: 4 Rationale: Kaposi's sarcoma is a vascular malignancy that presents as a skin disorder and is a common acquired immunodeficiency syndrome indicator. It is seen frequently in men with a history of same-sex partners. Although the cause of Kaposi's sarcoma is not known, it is considered to be the result of an alteration or failure in the immune system. The renal transplant client and the client receiving antineoplastic medications are at risk for immunosuppression. Exposure to asbestos is not related to the development of Kaposi's sarcoma.

The nurse is assigned to care for a client with systemic lupus erythematosus (SLE). The nurse plans care knowing that this disorder is:

1. A local rash that occurs as a result of allergy 2. A disease caused by overexposure to sunlight 3. An inflammatory disease of collagen contained in connective tissue 4. A disease caused by the continuous release of histamine in the body Answer: 3 Rationale: SLE is an inflammatory disease of collagen contained in connective tissue. Options 1, 2, and 4 are not associated with this disease.

A nurse is providing general information to a group of high school students about preventing human immunodeficiency virus (HIV) transmission. The nurse would inform the students that which of the following is an unsafe behavior?

1. Abstinence 2. Mutual monogamy 3. Use of latex condoms 4. Use of natural skin condoms Answer: 4 Rationale: The use of natural skin condoms is not considered safe because the pores in the condom are large enough for the virus to pass through. Abstinence is the safest way to avoid HIV infection. The next most reliable method is participation in a mutually monogamous relationship. The use of latex condoms is considered safe because the latex prevents the transmission of the HIV virus as long as the condom is used properly and remains in place and intact.

A client calls the office of his primary care health care provider and tells the nurse that he was just stung by a bumblebee while gardening. The client is afraid of a severe reaction, because the client's neighbor experienced such a reaction just 1 week ago. The appropriate nursing action is to:

1. Advise the client to soak the site in hydrogen peroxide. 2.Ask the client if he ever sustained a bee sting in the past. 3. Tell the client to call an ambulance for transport to the emergency room. 4. Tell the client not to worry about the sting unless difficulty with breathing occurs. Answer: 2 Rationale: In some types of allergies, a reaction occurs only on second and subsequent contacts with the allergen. Therefore, the appropriate action would be to ask the client if he ever received a bee sting in the past. Option 1 is not appropriate advice. Option 3 is unnecessary. The client should not be told "not to worry."

Which client is at the highest risk for systemic lupus erythematous (SLE)?

1. An Asian male 2. A white female 3. An African-American male 4. An African-American female Answer: 4 Rationale: SLE affects females more commonly than males. It is more common in African-American females than in white females.

The nurse is assigned to care for a client admitted to the hospital with a diagnosis of systemic lupus erythematosus (SLE). The nurse reviews the health care provider's prescriptions. Which of the following medications would the nurse expect to be prescribed?

1. Antibiotic 2. Antidiarrheal 3. Corticosteroid 4. Opioid analgesic Answer: 3 Rationale: Treatment of SLE is based on the systems involved and symptoms. Treatment normally consists of anti-inflammatory drugs, corticosteroids, and immunosuppressants. The incorrect options are not standard components of medication therapy for this disorder.

A nurse is assisting in the care of a client diagnosed with systemic lupus erythematosus (SLE). The nurse should administer which of the following prescribed medications that is needed to manage the condition?

1. Antidiarrheal 2. Corticosteroid 3. Antibiotic 4. Opioid analgesic Answer: 2 Rationale: Treatment of SLE is based on the systems involved and symptoms. Treatment normally consists of anti-inflammatory medications, corticosteroids, and immunosuppressants. The other options are not standard components of medication therapy for this disorder.

The client is diagnosed with stage I of Lyme disease. The nurse assesses the client for which characteristic of this stage?

1. Arthralgias 2. Flu-like symptoms 3. Enlarged and inflamed joints 4. Signs of neurological disorders Answer: 2 Rationale: The hallmark of stage I is the development of a skin rash within 2 to 30 days of infection, generally at the site of the tick bite. The rash develops into a concentric ring, giving it a bullseye appearance. The lesion enlarges up to 50 to 60 cm, and smaller lesions develop farther away from the original tick bite. In stage I, most infected persons develop flu-like symptoms that last 7 to 10 days; these symptoms may reoccur later. Neurological deficits occur in stage II. Arthralgias and joint enlargements are most likely to occur in stage III.

A client is diagnosed with stage I of Lyme disease. In addition to the rash, the nurse would check the client for which manifestation?

1. Arthralgias 2. Flulike symptoms 3. Neurologic deficits 4. Enlarged and inflamed joints Answer: 2 Rationale: The hallmark of stage I is the development of a skin rash at the tick bite site. The rash develops into a concentric ring that has a bullseye appearance. The lesion enlarges up to 50 to 60 cm, and smaller lesions develop farther away from the original tick bite. In stage I, most infected persons develop flulike symptoms that last 7 to 10 days, and these symptoms may recur later. The other options listed occur in stage II (neurological deficits) or stage III (arthralgias and enlarged, inflamed joints).

A nurse is assigned to care for a client who returned home from the emergency department following treatment for a sprained ankle. The nurse notes that the client was sent home with crutches that have rubber axillary pads and needs instructions regarding crutch walking. On data collection, the nurse discovers that the client has an allergy to latex. Before providing instructions regarding crutch walking, the nurse should:

1. Contact the health care provider (HCP). 2. Cover the crutch pads with cloth. 3. Call the local medical supply store, and ask for a cane to be delivered. 4. Tell the client that the crutches must be removed immediately from the house. Answer: 2 Rationale: The rubber pads used on crutches may contain latex. If the client requires the use of crutches, the nurse can cover the pads with a cloth to prevent cutaneous contact. Option 4 is inappropriate and may alarm the client. The nurse cannot prescribe a cane for a client. In addition, this type of assistive device may not be appropriate, considering this client's injury. No reason exists to contact the HCP at this time.

A nurse is assisting in preparing a plan of care for a client with acquired immunodeficiency syndrome (AIDS) who has nausea. Which dietary measure should the nurse include in the plan?

1. Dairy products with each snack and meal 2. Red meat daily 3. Adding spices to food to make the taste more palatable 4. Foods that are at room temperature Answer: 4 Rationale: The client with AIDS experiencing nausea should avoid fatty products, such as dairy products and red meat. Meals should be small and frequent to lessen the chance of vomiting. Spices and odorous foods should be avoided because they aggravate nausea. Foods are best tolerated either cold or at room temperature.

The nurse interprets that the client who is prescribed zalcitabine (Hivid) is experiencing an adverse effect of this medication when which event is reported by the client?

1. Diarrhea 2. Tinnitus 3. Burning with urination 4. Numbness in the legs Answer: 4 Rationale: Peripheral neuropathy is an adverse effect associated with the use of zalcitabine, which manifests initially as numbness and burning sensations in the extremities. They may progress to sharp shooting pains or severe continuous burning if the medication is not withdrawn. The other options are not associated with use of this medication.

The home care nurse is collecting data from a client who has been diagnosed with an allergy to latex. In determining the client's risk factors associated with the allergy, the nurse questions the client about an allergy to which food item?

1. Eggs 2. Milk 3. Yogurt 4. Bananas Answer: 4 Rationale: Individuals who are allergic to kiwis, bananas, pineapples, tropical fruits, grapes, avocados, potatoes, hazelnuts, and water chestnuts are at risk for developing a latex allergy. This is thought to be due to a possible cross-reaction between the food and the latex allergen. The incorrect options are unrelated to latex allergy.

The home care nurse is ordering dressing supplies for a client who has an allergy to latex. The nurse asks the medical supply personnel to deliver which of the following?

1. Elastic bandages 2. Adhesive bandages 3. Brown Ace bandages 4. Cotton pads and silk tape Answer: 4 Rationale: Cotton pads and plastic or silk tape are latex-free products. The items identified in the incorrect options are products that contain latex.

A nurse is collecting data on a client who complains of fatigue, weakness, malaise, muscle pain, joint pain at multiple sites, anorexia, and photosensitivity. Systematic lupus erythematosus (SLE) is suspected. The nurse further checks for which of the following that is also indicative of the presence of SLE?

1. Emboli 2. Ascites 3. Two hemoglobin S genes 4. Butterfly rash on cheeks and bridge of nose Answer: 4 Rationale: SLE is a chronic inflammatory disease that affects multiple body systems. A butterfly rash on the cheeks and on the bridge of the nose is a classic sign of SLE. Option 3 is found in sickle cell anemia. Options 1 and 2 may be found in many conditions but are not associated with SLE.

A client is diagnosed with an immune deficiency. The nurse focuses on which of the following as the highest priority when providing care to this client?

1. Encouraging discussion about emotional impact of the disorder 2. Identifying historical factors that placed the client at risk 3. Providing emotional support to decrease fear 4. Protecting the client from infection Answer: 4 Rationale: The client with immune deficiency has inadequate immune bodies and is at risk for infection. The priority nursing intervention would be to protect the client from infection. The other options are also part of the plan of care but are not the highest priority.

A complete blood cell count is performed on a client with systemic lupus erythematosus (SLE). The nurse would suspect that which of the following findings will be reported from this blood test?

1. Increased red blood cell count 2. Decrease of all cell types 3. Increased white blood cell count 4. Increased neutrophils Answer: 2 Rationale: In the client with SLE, a complete blood count commonly shows pancytopenia, a decrease of all cell types, probably caused by a direct attack of all blood cells or bone marrow by immune complexes. The other options are incorrect.

A nurse is assisting in developing a plan of care for a client with immunodeficiency. The nurse understands that which problem is a priority for the client?

1. Infection 2. Inability to cope 3. Lack of information about the disease 4. Feeling uncomfortable about body changes Answer: 1 Rationale: The client with immunodeficiency has inadequate or an absence of immune bodies and is at risk for infection. The priority problem is infection. The question presents no data indicating that options 2, 3, or 4 are a problem.

A nurse is assisting in developing a plan of care for a client with acquired immunodeficiency syndrome (AIDS) who is experiencing night fever and night sweats. Which nursing intervention should the nurse suggest including in the plan of care to manage this symptom?

1. Keep the call bell within reach for the client. 2. Administer a sedative at bedtime. 3. Administer an antipyretic at bedtime. 4. Provide a back rub and comfort measures before bedtime. Answer: 3 Rationale: For clients with AIDS who experience night fever and night sweats, it is useful to offer an antipyretic at bedtime. It is also helpful to keep a change of bed linens and night clothes nearby for use. The pillow should have a plastic cover, and a towel may be placed over the pillowcase if there is profuse diaphoresis. The client should have liquids at the bedside to drink. Options 1, 2, and 4 are important interventions but they are unrelated to the subject of fever and night sweats.

The camp nurse prepares to instruct a group of children about Lyme disease. Which of the following information would the nurse include in the instructions?

1. Lyme disease is caused by a tick carried by deer. 2. Lyme disease is caused by contamination from cat feces. 3. Lyme disease can be contagious by skin contact with an infected individual. 4. Lyme disease can be caused by the inhalation of spores from bird droppings. Answer: 1 Rationale: Lyme disease is a multisystem infection that results from a bite by a tick carried by several species of deer. Persons bitten by Ixodes ticks can be infected with the spirochete Borrelia burgdorferi. Lyme disease cannot be transmitted from one person to another. Toxoplasmosis is caused from the ingestion of cysts from contaminated cat feces. Histoplasmosis is caused by the inhalation of spores from bat or bird droppings.

A client who is human immunodeficiency virus (HIV) positive has had a Mantoux skin test. The results show a 7-mm area of induration. The nurse evaluates that this result is:

1. Negative 2. Borderline 3. Uncertain 4. Positive Answer: 4 Rationale: The client with HIV is considered to have positive results on Mantoux skin testing with an area of 5 mm of induration or greater. The client without HIV is positive with induration greater than 10 or 15 mm if the client is at low risk. The client with HIV is immunosuppressed, making a smaller area of induration positive for this type of client. It is also possible for the client infected with HIV to have false negative readings because of the immunosuppression factor.

A client in the clinical unit who is allergic to shellfish unknowingly ate a dish brought by a friend that had shellfish as an ingredient. The client quickly develops anaphylaxis. The nurse would focus on which of the following first until additional help arrives?

1. Preparing a dose of epinephrine (Adrenalin) 2. Preparing a dose of a corticosteroid 3. Maintaining a patent airway 4. Telling the client to obtain a Medic-Alert bracelet Answer: 3 Rationale: The initial priority of the nurse would be to maintain a patent airway. Once additional helps arrives, the client would likely receive epinephrine and corticosteroids. The topic of the Medic-Alert bracelet should be deferred until the client is stable.

A nurse is reviewing the medical record of a young female client who is suspected of having systematic lupus erythematosus (SLE). Which of the following would the nurse expect to note documented in the record that is related to this diagnosis?

1. Presence of two hemoglobin S genes in the blood cell report 2. Ascites noted in the abdomen 3. Recurrent emboli 4. Butterfly rash on cheeks and bridge of the nose Answer: 4 Rationale: SLE primarily occurs in females 10 to 35 years of age and is a chronic inflammatory disease that affects multiple body systems. A butterfly rash on the cheeks and the bridge of the nose is a characteristic sign of SLE. Option 1 is found in sickle cell anemia. Options 2 and 3 are found in many conditions but are not usually noted in SLE.

The nurse is assisting in planning care for a client with a diagnosis of immune deficiency. The nurse would incorporate which of the following as a priority in the plan of care?

1. Protecting the client from infection 2. Providing emotional support to decrease fear 3. Encouraging discussion about lifestyle changes 4. Identifying factors that decreased the immune function Answer: 1 Rationale: The client with immune deficiency has inadequate or absent immune bodies and is at risk for infection. The priority nursing intervention would be to protect the client from infection. Options 2, 3, and 4 may be components of care but are not the priority.

A client with acquired immunodeficiency syndrome (AIDS) is experiencing shortness of breath related to Pneumocystis jiroveci pneumonia. Which measure should the nurse suggest to assist the client in performing activities of daily living?

1. Provide supportive care with hygiene needs. 2. Provide meals and snacks with high protein, high calorie, and high nutritional value. 3. Provide small, frequent meals. 4. Offer low microbial food. Answer: 1 Rationale: Providing supportive care with hygiene needs as needed reduces the client's physical and emotional energy demands and conserves energy resources for other functions such as breathing. Options 2, 3, and 4 are important interventions for the client with AIDS but do not address the subject of activities of daily living. Option 2 will assist the client in maintaining appropriate weight and proper nutrition. Option 3 will assist the client in tolerating meals better. Option 4 will decrease the client's risk of infection.

A client with acquired immunodeficiency syndrome (AIDS) has difficulty swallowing. The nurse has given the client suggestions to minimize the problem. The nurse determines that the client has understood the instructions if the client verbalized to increase intake of foods such as:

1. Raw fruits and vegetables 2. Hot soup 3. Peanut butter 4. Puddings Answer: 4 Rationale: The client is instructed to avoid spicy, sticky, or excessively hot or cold foods. The client also is instructed to avoid foods that are rough, such as uncooked fruits or vegetables. The client is encouraged to take in foods that are mild, nonabrasive, and easy to swallow. Examples of these include baked fish, noodle dishes, well-cooked eggs, and desserts such as ice cream or pudding. Dry grain foods such as crackers, bread, or cookies may be softened in milk or another beverage before eating.

A client with acquired immunodeficiency syndrome (AIDS) is taking zidovudine (Retrovir) 200 mg orally three times daily. The client reports to the health care clinic for follow-up blood studies, and the results of the blood studies indicate severe neutropenia. Which of the following would the nurse anticipate to be prescribed for the client?

1. Reduction in the medication dosage 2. Discontinuation of the medication 3. The administration of prednisone concurrent with the therapy 4. Administration of epoetin alfa (Epogen) Answer: 2 Rationale: Hematological monitoring should be done every 2 weeks in the client taking zidovudine. If severe anemia or severe neutropenia develops, treatment should be discontinued until there is evidence of bone marrow recovery. If anemia or neutropenia is mild, a reduction in dosage may be sufficient. The administration of prednisone may further alter the immune function. Epoetin alfa is given to clients experiencing anemia.

A female client arrives at the health care clinic and tells the nurse that she was just bitten by a tick and would like to be tested for Lyme disease. The client tells the nurse that she removed the tick and flushed it down the toilet. Which of the following nursing actions is appropriate?

1. Refer the client for a blood test immediately. 2. Inform the client that there is not a test available for Lyme disease. 3. Tell the client that testing is not necessary unless arthralgia develops. 4. Instruct the client to return in 4 to 6 weeks to be tested, because testing before this time is not reliable. Answer: 4 Rationale: A blood test is available to detect Lyme disease; however, the test is not reliable if performed before 4 to 6 weeks following the tick bite. Antibody formation takes place in the following manner: immunoglobulin M is detected 3 to 4 weeks after Lyme disease onset, peaks at 6 to 8 weeks, and then gradually disappears; immunoglobulin G is detected 2 to 3 months after infection and may remain elevated for years. Options 1, 2, and 3 are incorrect.

A nurse is providing instructions to a client with acquired immunodeficiency syndrome (AIDS) who is experiencing night fever and night sweats. The nurse advises the client to do which of the following to increase comfort while minimizing symptoms?

1. Remove the plastic cover on the pillow. 2. Keep liquids on the nightstand at home. 3. Reduce fluid intake before bedtime. 4. Take an antipyretic after the fever spikes. Answer: 2 Rationale: For clients with AIDS who experience night fever and night sweats, it is useful to keep liquids on the nightstand at home. The client should keep a plastic cover on the pillow and place a towel over the pillowcase if needed also. The client should not decrease fluid intake, and the client should take an antipyretic before going to sleep and before the fever spikes.

The client with acquired immunodeficiency syndrome has raised, dark purplish lesions on the trunk of the body. The nurse anticipates that which of the following procedures will be done to confirm whether these lesions are due to Kaposi's sarcoma?

1. Skin biopsy 2. Lung biopsy 3. Western blot 4. Enzyme-linked immunosorbent assay Answer: 1 Rationale: The skin biopsy is the procedure of choice to diagnose Kaposi's sarcoma, which frequently complicates the clinical picture of the client with acquired immunodeficiency syndrome. Lung biopsy would confirm Pneumocystis jiroveci infection. The enzyme-linked immunosorbent assay and Western blot are tests to diagnose human immunodeficiency virus status.

A client calls the health care clinic and tells the nurse that he was bitten by a tick. The client is concerned and asks the nurse about the first signs of Lyme disease. The nurse informs the client that stage 1 of Lyme disease is characterized by:

1. Skin rash 2. Painful joints 3. Tremors and weakness 4. Headaches and blurred vision Answer: 1 Rationale: The hallmark of stage 1 of Lyme disease is the development of a skin rash within 2 to 30 days of infection, generally at the site of the tick bite. The rash develops into a concentric ring, giving it a bullseye appearance. The lesion enlarges up to 50 to 60 cm, and smaller lesions develop farther away from the original tick bite. In stage 1, most infected people develop flulike symptoms that last 7 to 10 days, and these symptoms may recur later. Options 2, 3, and 4 are not the first symptoms related to Lyme disease.

A client with acquired immunodeficiency syndrome (AIDS) reports nausea, vomiting, and abdominal pain after beginning didanosine (Videx) therapy. The clinic nurse emphasizes what instruction to this client?

1. Take crackers and milk with each dose of the medication. 2. Come to the health care clinic to be seen by the health care provider. 3. Decrease the dose of the medication until the next clinic visit. 4. This is an uncomfortable but expected side effect of the medication. Answer: 2 Rationale: Pancreatitis, which can be fatal, is the major dose-limiting toxicity associated with the administration of didanosine (Videx). The client should be seen by the health care provider and be monitored for indications of developing pancreatitis. The reported symptoms are not the primary subject, and so the options directed toward explaining or managing them are not correct. The nurse should not encourage the client to alter the medication dose without first notifying the health care provider.

A client calls the emergency department and tells the nurse that he received a bee sting to the arm while weeding a garden. The client states that he has received bee stings in the past and is not allergic to bees. The client states that the site is painful and asks the nurse for advice to alleviate the pain. The nurse tells the client to first:

1. Take two acetaminophen (Tylenol). 2. Place a heating pad to the site. 3. Apply ice and elevate the site. 4. Lie down and elevate the arm. Answer: 3 Rationale: When a bee sting occurs and is painful, it is best to treat the site locally rather than systemically. Pain may be alleviated by the application of an ice pack and elevating the site. A heating pad will increase discomfort at the site. Acetaminophen may be taken by the client to assist in alleviating discomfort, but this would not treat the injury at a local level. Lying down and elevating the arm may have some effect on reducing edema at the site but will not directly assist in alleviating the pain at the site of injury.

A client who is prescribed zidovudine (Retrovir) has been diagnosed with severe neutropenia. The nurse anticipates which intervention will be implemented?

1. The medication dose will be reduced. 2. The medication will be temporarily discontinued. 3. Prednisone will be added to the medication regimen. 4. Epoetin alfa (Epogen) will be added to the medication regimen. Answer: 2 Rationale: Hematological monitoring should be done every 2 weeks in the client taking zidovudine. If severe anemia or neutropenia develops, treatment should be interrupted until there is evidence of bone marrow recovery. If anemia or neutropenia is mild, a reduction in dosage may be sufficient. The administration of prednisone may further alter the immune function. Epoetin alfa is given to clients experiencing anemia.

A client with pemphigus is being seen in the clinic regularly. The nurse plans care based on which of the following descriptions of this condition?

1. The presence of tiny red vesicles 2. An autoimmune disease that causes blistering in the epidermis 3. The presence of skin vesicles found along the nerve caused by a virus 4. The presence of red, raised papules and large plaques covered by silvery scales Answer: 2 Rationale: Pemphigus is an autoimmune disease that causes blistering in the epidermis. The client has large flaccid blisters (bullae). Because the blisters are in the epidermis, they have a thin covering of skin and break easily, leaving large denuded areas of skin. On initial examination, clients may have crusting areas instead of intact blisters. Option 1 describes eczema, option 3 describes herpes zoster, and option 4 describes psoriasis.

A client reports to the health care clinic to obtain testing regarding human immunodeficiency virus (HIV) status after being exposed to an individual who is HIV positive. The test results are reported as negative, and the client tells the nurse that he feels so much better knowing that he had not contracted HIV. The nurse explains the test results to the client, telling the client that:

1. There is no further need for testing. 2. A negative HIV test is considered accurate. 3. A negative HIV test is not considered accurate during the first 6 months after exposure. 4. The test should be repeated in 1 week. Answer: 3 Rationale: A test done for HIV should be repeated. There might be a lag period after the infection occurs and before antibodies appear in the blood. Therefore a negative HIV test is not considered accurate during the first 6 months after exposure.

Which interventions would apply in the care of a client at high risk for an allergic response to a latex allergy. Select all that apply.

1. Use non-latex gloves. 2. Use medications from glass ampules. 3. Place the client in a private room only. 4. Do not puncture rubber stoppers with needles. 5. Keep a latex-safe supply cart available in the client's area. 6. Use a blood pressure cuff from an electronic device only to measure the blood pressure. Answer: 1 2 4 5 Rationale: If a client is allergic to latex and is at high risk for an allergic response, the nurse would use non-latex gloves and latex-safe supplies and would keep a latex-safe supply cart available in the client's area. Any supplies or materials that contain latex would be avoided. These include blood pressure cuffs and medication bottles with a rubber stopper that requires puncture with a needle. It is not necessary to place the client in a private room.

The nurse prepares to give a bath and change the bed linens on a client with cutaneous Kaposi's sarcoma lesions. The lesions are open and draining a scant amount of serous fluid. Which of the following would the nurse incorporate in the plan during the bathing of this client?

1. Wearing gloves 2. Wearing a gown and gloves 3. Wearing a gown, gloves, and a mask 4. Wearing a gown and gloves to change the bed linens and gloves only for the bath Answer: 2 Rationale: Gowns and gloves are required if the nurse anticipates contact with soiled items, such as wound drainage, or while caring for a client who is incontinent with diarrhea or a client who has an ileostomy or colostomy. Masks are not required unless droplet or airborne precautions are necessary. Regardless of the amount of wound drainage, a gown and gloves must be worn.

A client is suspected of having systemic lupus erythematous. The nurse monitors the client, knowing that which of the following is one of the initial characteristic sign of systemic lupus erythematous?

1. Weight gain 2. Subnormal temperature 3. Elevated red blood cell count 4. Rash on the face across the bridge of the nose and on the cheeks Answer: 4 Rationale: Skin lesions or rash on the face across the bridge of the nose and on the cheeks is an initial characteristic sign of systemic lupus erythematosus (SLE). Fever and weight loss may also occur. Anemia is most likely to occur later in SLE.

10. The nurse in charge is evaluating the infection control procedures on the unit. Which finding indicates a break in technique and the need for education of staff? a. The nurse aide is not wearing gloves when feeding an elderly client. b. A client with active tuberculosis is asked to wear a mask when he leaves his room to go to another department for testing. c. A nurse with open, weeping lesions of the hands puts on gloves before giving direct client care. d. The nurse puts on a mask, a gown, and gloves before entering the room of a client on strict isolation.

10. Answer C. There is no need to wear gloves when feeding a client. However, universal precautions (treating all blood and body fluids as if they are infectious) should be observed in all situations. A client with active tuberculosis should be on respiratory precautions. Having the client wear a mask when leaving his private room is appropriate. Persons with exudative lesions or weeping dermatitis should not give direct client care or handle client-care equipment until the condition resolves. Strict isolation requires the use of mask, gown, and gloves.

11. The charge nurse observes a new staff nurse who is changing a dressing on a surgical wound. After carefully washing her hands the nurse dons sterile gloves to remove the old dressing. After removing the dirty dressing, the nurse removes the gloves and dons a new pair of sterile gloves in preparation for cleaning and redressing the wound. The most appropriate action for the charge nurse is to: a. interrupt the procedure to inform the staff nurse that sterile gloves are not needed to remove the old dressing. b. congratulate the nurse on the use of good technique. c. discuss dressing change technique with the nurse at a later date. d. interrupt the procedure to inform the nurse of the need to wash her hands after removal of the dirty dressing and gloves.

11. Answer D. Nonsterile gloves are adequate to remove the old dressing. However, the use of sterile gloves does not put the client in danger so discussion of this can wait until later. The staff nurse is doing two things incorrectly. Nonsterile gloves are adequate to remove the old dressing. The nurse should wash her hands after removing the soiled dressing and before donning sterile gloves to clean and dress the wound. The nurse should wash her hands after removing the soiled dressing and before donning the sterile gloves to clean and dress the wound. Not doing this compromises client safety and should be brought to the immediate attention of the nurse. The staff nurse is doing two things incorrectly. Nonsterile gloves are adequate to remove the old dressing. However, the use of sterile gloves does not put the client in danger so discussion of this can wait until later. However, the nurse should wash her hands after removing the soiled dressing and before donning sterile gloves to clean and dress the wound. Not doing this compromises client safety and should be brought to the immediate attention of the nurse.

12. Nurse Jane is visiting a client at home and is assessing him for risk of a fall. The most important factor to consider in this assessment is: a. Correct illumination of the environment. b. amount of regular exercise. c. the resting pulse rate. d. status of salt intake.

12. Answer A. To prevent falls, the environment should be well lighted. Night lights should be used if necessary. Other factors to assess include removing loose scatter rugs, removing spills, and installing handrails and grab bars as appropriate. The amount of regular exercise is not the most important factor to assess. It is only indirectly related. The resting pulse rate is not related to preventing falls. The salt intake is not directly related to preventing falls.

13. Mrs. Jones will have to change the dressing on her injured right leg twice a day. The dressing will be a sterile dressing, using 4 X 4s, normal saline irrigant, and abdominal pads. Which statement best indicates that Mrs. Jones understands the importance of maintaining asepsis? a. "If I drop the 4 X 4s on the floor, I can use them as long as they are not soiled." b. "If I drop the 4 X 4s on the floor, I can use them if I rinse them with sterile normal saline." c. "If I question the sterility of any dressing material, I should not use it." d. "I should put on my sterile gloves, then open the bottle of saline to soak the 4 X 4s."

13. Answer C. Anything dropped on the floor is no longer sterile and should not be used. The statement indicates lack of understanding. Anything dropped on the floor is no longer sterile and should not be used. The statement indicates lack of understanding. If there is ever any doubt about the sterility of an instrument or dressing, it should not be used. The 4 X 4s should be soaked prior to donning the sterile gloves. Once the sterile gloves touch the bottle of normal saline they are no longer sterile. This statement indicates a need for further instruction.

14. A client has been placed in blood and body fluid isolation. The nurse is instructing auxiliary personnel in the correct procedures. Which statement by the nursing assistant indicates the best understanding of the correct protocol for blood and body fluid isolation? a. Masks should be worn with all client contact. b. Gloves should be worn for contact with nonintact skin, mucous membranes, or soiled items. c. Isolation gowns are not needed. d. A private room is always indicated.

14. Answer B. Masks should only be worn during procedures that are likely to cause splashes of blood or body fluid. Gloves should be worn for all contact with blood and body fluids, nonintact skin and mucous membranes; for handling soiled items; and for performing venipuncture. Gowns should be worn during procedures that are likely to cause splashes of blood or body fluids. A private room is only indicated if the client's hygiene is poor.

15. A client has been placed in blood and body fluid isolation. The nurse is instructing auxiliary personnel in the correct procedures. Which statement by the nursing assistant indicates the best understanding of the correct protocol for blood and body fluid isolation? a. Masks should be worn with all client contact. b. Gloves should be worn for contact with nonintact skin, mucous membranes, or soiled items. c. Isolation gowns are not needed. d. A private room is always indicated.

15. Answer B. Masks should only be worn during procedures that are likely to cause splashes of blood or body fluid. Gloves should be worn for all contact with blood and body fluids, nonintact skin and mucous membranes; for handling soiled items; and for performing venipuncture. Gowns should be worn during procedures that are likely to cause splashes of blood or body fluids. A private room is only indicated if the client's hygiene is poor.

16. The nurse is evaluating whether nonprofessional staff understand how to prevent transmission of HIV. Which of the following behaviors indicates correct application of universal precautions? a. A lab technician rests his hand on the desk to steady it while recapping the needle after drawing blood. b. An aide wears gloves to feed a helpless client. c. An assistant puts on a mask and protective eye wear before assisting the nurse to suction a tracheostomy. d. A pregnant worker refuses to care for a client known to have AIDS.

16. Answer C. Needles that have been used to draw blood should not be recapped. If it is necessary to recap them, an instrument such as a hemostat should be used to recap. The hand should never be used. Gloves are not necessary when feeding, since there is no contact with mucus membranes. Although saliva may have small amounts of HIV in it, the virus does not invade through unbroken skin. There is no evidence in the question to indicate broken skin. Masks and protective eye wear are indicated anytime there is great potential for splashing of body fluids that may be contaminated with blood. Suctioning of a tracheostomy almost always stimulates coughing, which is likely to generate droplets that may splash the health care worker. Clients who are suctioned frequently or have had an invasive procedure like a tracheostomy are likely to have blood in the sputum. There is no reason to restrict pregnant workers from caring for persons with AIDS as long as they utilize universal precautions.

18. Jessie, a young man with newly diagnosed acquired immune deficiency syndrome (AIDS) is being discharged from the hospital. The nurse knows that teaching regarding prevention of AIDS transmission has been effective when the client: a. verbalizes the role of sexual activity in spread of the disorder. b. states he will make arrangements to drop his college classes. c. acknowledges the need to avoid all contact sports. d. says he will avoid close contact with his three-year-old niece.

18. Answer A. The AIDS virus is spread through direct contact with body fluids such as blood and through sexual intercourse. Casual contact with other people does not pose a risk of transmission of AIDS. Unless the client is feeling very ill, there is no need for him to drop his college classes. Contact sports are not contraindicated unless there is a significant chance of bleeding and direct contact with others. Casual contact with other people does not pose a risk of transmission of AIDS. There is no need to limit casual contact with children.

A nurse is assessing a kid who is scheduled to receive a live vaccine. What are the general contraindications associated with receiving a live virus vaccine? SELECT ALL 1. The kid has symptoms of a cold 2. The child has a previous anaphylactic reaction to the vaccine 3. mother reports the kid is having intermittent episodes of diarrhea 4. mother reports that the kid has had no appetite and has been fussy 5. The kid has a severely deficient immune system 6. Mother reports that the kid has recently been exposed to infectious disease

2, 5 The contraindications for for this are a previous anaphylactic episode, weakened immune systems, those with a severe sensitivity to gelatin, or pregnant women

A nurse is assisting in the care of a client diagnosed with systemic lupus erythematosus (SLE). The nurse should administer which of the following prescribed medications that is needed to manage the condition?

2. Corticosteroid

A clinic nurse periodically cares for a client diagnosed with acquired immunodeficiency syndrome. The nurse assesses for an early manifestation of Pneumocystis jiroveci infection by monitoring for which of the following at each client visit?

2. Cough

A nurse is assigned to care for a client who returned home from the emergency department following treatment for a sprained ankle. The nurse notes that the client was sent home with crutches that have rubber axillary pads and needs instructions regarding crutch walking. On data collection, the nurse discovers that the client has an allergy to latex. Before providing instructions regarding crutch walking, the nurse should:

2. Cover the crutch pads with cloth.

A complete blood cell count is performed on a client with systemic lupus erythematosus (SLE). The nurse would suspect that which of the following findings will be reported from this blood test?

2. Decrease of all cell types

A nurse is assisting in developing a plan of care for a pregnant client with acquired immunodeficiency syndrome (AIDS). The nurse determines that which of the following is the priority concern for this client?

2. Development of an infection

A client with acquired immunodeficiency syndrome (AIDS) is taking zidovudine (Retrovir) 200 mg orally three times daily. The client reports to the health care clinic for follow-up blood studies, and the results of the blood studies indicate severe neutropenia. Which of the following would the nurse anticipate to be prescribed for the client?

2. Discontinuation of the medication

A client calls the office of his primary care health care provider and tells the nurse that he was just stung by a bumblebee while gardening. The client is afraid of a severe reaction, because the client's neighbor experienced such a reaction just 1 week ago. The appropriate nursing action is to:

2.Ask the client if he ever sustained a bee sting in the past.

20. Mrs. Parker, a 70-year-old woman with severe macular degeneration, is admitted to the hospital the day before scheduled surgery. The nurse's preoperative goals for Mrs. M. would include: a. independently ambulating around the unit. b. reading the routine preoperative education materials. c. maneuvering safely after orientation to the room. d. using a bedpan for elimination needs.

20. Answer C. Independently ambulating around the unit is not appropriate because the unit environment can change and injury could result. Assistance is necessary because of the client's visual deficit. It is unlikely the client can see well enough to read the materials. Maneuvering safely after orientation to the room is a realistic goal for a person with impaired vision. Orienting the client to the room should help the client to move safely. Using the bedpan is an unnecessary restriction on the client as she can be oriented to the bathroom or to call for assistance.

A client reports to the health care clinic to obtain testing regarding human immunodeficiency virus (HIV) status after being exposed to an individual who is HIV positive. The test results are reported as negative, and the client tells the nurse that he feels so much better knowing that he had not contracted HIV. The nurse explains the test results to the client, telling the client that:

3. A negative HIV test is not considered accurate during the first 6 months after exposure.

A client calls the emergency department and tells the nurse that he received a bee sting to the arm while weeding a garden. The client states that he has received bee stings in the past and is not allergic to bees. The client states that the site is painful and asks the nurse for advice to alleviate the pain. The nurse tells the client to first:

3. Apply ice and elevate the site.

A nurse reads the chart of a client who has been diagnosed with stage 3 Lyme disease. Which clinical manifestation supports this diagnosis?

3. Complaints of joint pain

A kid is receiving a series of Hep B vaccines and arrives for a second dose. Before giving the nurse should ask the kid and parent about a history of a severe allergic reaction to what? 1. Eggs 2. PCN 3. Sulfonamides 4. A previous dose of HEP B vaccine or component

4 * A contraindication is a previous reaction to a previous dose of HEP B or a component ( aluminum hydroxide or yeast protein)

A nurse visits a child with Mono and provides care instructions to the parents. Which instruction should the nurse give the parents? 1. maintain bed rest for 2 weeks 2. maintain Resp. precautions for 1 week 3. Notify HCP if kid develops a fever 4. Notify HCP if child develops ABD pain Left shoulder pain

4 * Mono is caused by Epstein- Barr virus. Parents need to report pain in ABD, especially in LUQ or left shoulder pain ; this may indicate that the spleen has ruptured.

The nurse provides home care instructions to the parents of a child hospitalized with pertussis who is in the convalescent stage and is being prepared for d/c. What statement made by parent indicates a NEED for further instruction? 1. We need to encourage our child to drink fluids 2. Coughing spells may be triggered by dust or smoke 3. Vomiting may occur when our child has coughing episodes 4. We need to maintain droplet precautions and a quiet environment for at leat 2 weeks

4 * pertussis is transmitted by direct contact or resp. droplets from coughing

Which of the following individuals is least likely at risk for the development of Kaposi's sarcoma?

4. An individual working in an environment where exposure to asbestos exists

4. Which of the following is the FIRST priority in preventing infections when providing care for a client? a. Handwashing b. Wearing gloves c. Using a barrier between client's furniture and nurse's bag d. Wearing gowns and goggles

4. Answer A. Handwashing remains the most effective way to avoid spreading infection. However, too often nurses do not practice good handwashing techniques and do not teach families to do so. Nurses need to wash their hands before and after touching the client and before entering the nursing bag.

A nurse is collecting data on a client who complains of fatigue, weakness, malaise, muscle pain, joint pain at multiple sites, anorexia, and photosensitivity. Systematic lupus erythematosus (SLE) is suspected. The nurse further checks for which of the following that is also indicative of the presence of SLE?

4. Butterfly rash on cheeks and bridge of nose

A nurse is reviewing the medical record of a young female client who is suspected of having systematic lupus erythematosus (SLE). Which of the following would the nurse expect to note documented in the record that is related to this diagnosis?

4. Butterfly rash on cheeks and bridge of the nose

The home care nurse is ordering dressing supplies for a client who has an allergy to latex. The nurse asks the medical supply personnel to deliver which of the following?

4. Cotton pads and silk tape Answer: 4 Rationale: Cotton pads and plastic or silk tape are latex-free products. The items identified in the incorrect options are products that contain latex.

A nurse is assisting in preparing a plan of care for a client with acquired immunodeficiency syndrome (AIDS) who has nausea. Which dietary measure should the nurse include in the plan?

4. Foods that are at room temperature

A female client arrives at the health care clinic and tells the nurse that she was just bitten by a tick and would like to be tested for Lyme disease. The client tells the nurse that she removed the tick and flushed it down the toilet. Which of the following nursing actions is appropriate?

4. Instruct the client to return in 4 to 6 weeks to be tested, because testing before this time is not reliable.

The nurse interprets that the client who is prescribed zalcitabine (Hivid) is experiencing an adverse effect of this medication when which event is reported by the client?

4. Numbness in the legs

A client with acquired immunodeficiency syndrome (AIDS) has difficulty swallowing. The nurse has given the client suggestions to minimize the problem. The nurse determines that the client has understood the instructions if the client verbalized to increase intake of foods such as:

4. Puddings

A client is suspected of having systemic lupus erythematous. The nurse monitors the client, knowing that which of the following is one of the initial characteristic sign of systemic lupus erythematous?

4. Rash on the face across the bridge of the nose and on the cheeks

5. An adult woman is admitted to an isolation unit in the hospital after tuberculosis was detected during a pre-employment physical. Although frightened about her diagnosis, she is anxious to cooperate with the therapeutic regimen. The teaching plan includes information regarding the most common means of transmitting the tubercle bacillus from one individual to another. Which contamination is usually responsible? a. Hands. b. Droplet nuclei. c. Milk products. d. Eating utensils.

5. Answer B. Hands are the primary method of transmission of the common cold. The most frequent means of transmission of the tubercle bacillus is by droplet nuclei. The bacillus is present in the air as a result of coughing, sneezing, and expectoration of sputum by an infected person. The tubercle bacillus is not transmitted by means of contaminated food. Contact with contaminated food or water could cause outbreaks of salmonella, infectious hepatitis, typhoid, or cholera. The tubercle bacillus is not transmitted by eating utensils. Some exogenous microbes can be transmitted via reservoirs such as linens or eating utensils.

6. A 2 year old is to be admitted in the pediatric unit. He is diagnosed with febrile seizures. In preparing for his admission, which of the following is the most important nursing action? a. Order a stat admission CBC. b. Place a urine collection bag and specimen cup at the bedside. c. Place a cooling mattress on his bed. d. Pad the side rails of his bed.

6. Answer D. Preparing for routine laboratory studies is not as high a priority as preventing injury and promoting safety. Preparing for routine laboratory studies is not as high a priority as preventing injury and promoting safety. A cooling blanket must be ordered by the physician and is usually not used unless other methods for the reduction of fever have not been successful. The child has a diagnosis of febrile seizures. Precautions to prevent injury and promote safety should take precedence.

7. A young adult is being treated for second and third degree burns over 25% of his body and is now ready for discharge. The nurse evaluates his understanding of discharge instructions relating to wound care and is satisfied that he is prepared for home care when he makes which statement? a. "I will need to take sponge baths at home to avoid exposing the wounds to unsterile bath water." b. "If any healed areas break open I should first cover them with a sterile dressing and then report it." c. "I must wear my Jobst elastic garment all day and can only remove it when I'm going to bed." d. "I can expect occasional periods of low-grade fever and can take Tylenol every 4 hours."

7. Answer B. Bathing or showering in the usual manner is permitted, using a mild detergent soap such as Ivory Snow. This cleanses the wounds, especially those that are still open, and removes dead tissue. The client is taught to report changes in wound healing such as blister formation, signs of infection, and opening of a previously healed area. Sterile dressings are applied until the wound is assessed and a plan of care developed. The Jobs garment is designed to place constant pressure on the new healthy tissue that is forming to promote adherence to the underlying structure in order to prevent hypertrophic scarring. In order to be effective, the garment must be worn for 23 hours daily. It is removed for wound assessment and wound care and to permit bathing. The client must be aware that infection of the wound may occur; signs of infection, including fever, redness, pain, warmth in and around the wound and increased or foul smelling drainage must be reported immediately.

8. An eighty five year old man was admitted for surgery for benign prostatic hypertrophy. Preoperatively he was alert, oriented, cooperative, and knowledgeable about his surgery. Several hours after surgery, the evening nurse found him acutely confused, agitated, and trying to climb over the protective side rails on his bed. The most appropriate nursing intervention that will calm an agitated client is a. limit visits by staff. b. encourage family phone calls. c. position in a bright, busy area. d. speak soothingly and provide quiet music.

8. Answer D. The client needs frequent visits by the staff to orient him and to assess his safety. Phone calls from his family will not help a client who is trying to climb over the side rails and may even add to his danger. Putting the client in a bright, busy area would probably add to his confusion. The environment is an important factor in the prevention of injuries. Talking softly and providing quiet music have a calming effect on the agitated client.

9. Ms. Smith is admitted for internal radiation for cancer of the cervix. The nurse knows the client understands the procedure when she makes which of the following remarks the night before the procedure? a. She says to her husband, "Please bring me a hamburger and french fries tomorrow when you come. I hate hospital food." b. "I told my daughter who is pregnant to either come to see me tonight or wait until I go home from the hospital." c. "I understand it will be several weeks before all the radiation leaves my body." d. "I brought several craft projects to do while the radium is inserted."

9. Answer B. The client will be on a clear liquid or very low residue diet. Hamburgers and french fries are not allowed. People who are pregnant should not come in close contact with someone who has internal radiation therapy. The radioactivity could possibly damage the fetus. This statement is not true. As soon as the radiation source is removed (probably 36 to 72 hours after insertion), the client is no longer contaminated with radioactivity. Craft projects usually require the client to sit. The client must remain flat with very little head elevation during the time the rods are in place.

A pregnant woman with a history of asymptomatic HIV infection is seen at the clinic. Which information will the nurse include when teaching the patient? a. Although infants of HIV-infected mothers always test positive for HIV antibodies, most infants are not infected with the virus. b. Because she has not developed AIDS, the infant will not contract HIV during intrauterine life. c. The infant will be started on zidovudine (AZT) after delivery to prevent HIV infection. d. It is likely that her newborn will develop HIV infection unless she takes antiretroviral drugs during the pregnancy.

A Rationale: Because antibodies are transmitted from the mother to the fetus during intrauterine life, all infants of HIV-positive mothers will test positive at birth. Ongoing antibody (or viral) testing is needed to determine whether the infant is infected with HIV. Transmission of the virus can occur during fetal life even if the mother does not have AIDS. Infants of HIV-positive mothers are not routinely started on antiretroviral therapy (ART). Only 25% of infants born to HIV-positive mothers develop HIV infection, even when the mother does not use ART during pregnancy. Cognitive Level: Application Text Reference: p. 250 Nursing Process: Implementation NCLEX: Health Promotion and Maintenance

A 24-year-old woman who uses injectable illegal drugs asks the nurse about preventing AIDS. The nurse informs the patient that the best way to reduce the risk of HIV infection from drug use is to a. participate in a needle-exchange program. b. clean drug injection equipment before use. c. ask those who share equipment to be tested for HIV. d. avoid sexual intercourse when using injectable drugs.

A Rationale: Participation in needle-exchange programs has been shown to control the rate of HIV infection. Cleaning drug equipment before use also reduces risk, but it might not be consistently practiced by individuals in withdrawal. HIV antibodies do not appear for several weeks to months after exposure, so testing drug uses would not be very effective in reducing risk for HIV exposure. It is difficult to make appropriate decisions about sexual activity when under the influence of drugs. Cognitive Level: Comprehension Text Reference: pp. 262-263 Nursing Process: Planning NCLEX: Health Promotion and Maintenance

When designing a program to decrease the incidence of HIV infection in the community, the nurse will prioritize education about a. how to prevent transmission between sexual partners. b. methods to prevent perinatal HIV transmission. c. ways to sterilize needles used by injectable drug users. d. means to prevent transmission through blood transfusions.

A Rationale: Sexual transmission is the most common way that HIV is transmitted. The nurse should also provide education about perinatal transmission, needle sterilization, and blood transfusion, but the rate of HIV infection associated with these situations is lower. Cognitive Level: Application Text Reference: pp. 250, 260-263 Nursing Process: Planning NCLEX: Physiological Integrity

To evaluate the effectiveness of ART, the nurse will schedule the patient for a. viral load testing. b. enzyme immunoassay. c. rapid HIV antibody testing. d. immunofluorescence assay.

A Rationale: The effectiveness of ART is measured by the decrease in the amount of virus detectable in the blood. The other tests are used to detect for HIV antibodies, which remain positive even with effective ART. Cognitive Level: Application Text Reference: p. 265 Nursing Process: Planning NCLEX: Physiological Integrity

When teaching a patient with HIV infection about ART, the nurse explains that these drugs a. work in various ways to decrease viral replication in the blood. b. boost the ability of the immune system to destroy the virus. c. destroy intracellular virus as well as lowering the viral load. d. increase the number of CD4+ cells available to fight the HIV.

A Rationale: The three groups of antiretroviral drugs work in different ways to decrease the ability of the virus to replicate. The drugs do not work by boosting the ability of the immune system or CD4 cells to fight the virus. The viral load detected in the blood is decreased with effective therapy, but intracellular virus is still present. Cognitive Level: Application Text Reference: pp. 256-257 Nursing Process: Implementation NCLEX: Physiological Integrity

Autoimmunity

A breakdown of selftolerance

d. When administering immunotherapy, it is imperative to closely monitor the patient for any signs of an adverse reaction. The high risk and significant consequence of an adverse reaction supersede the need to assess the patient's fluid balance, whereas pain and changes in level of consciousness are not likely events when administering immunotherapy.

A patient has begun immunotherapy for the treatment of intractable environmental allergies. When administering the patient's immunotherapy, which of the following is the nurse's priority action? A. Monitoring the patient's fluid balance B. Assessing the patient's need for analgesia C. Assessing the patient for changes in level of consciousness D. Monitoring for signs and symptoms of an adverse reaction

c

A patient is undergoing plasmapheresis for treatment of systemic lupus erythematosus. The nurse explains that plasmapheresis is used in her treatment to: A. remove T lymphocytes in her blood that are producing antinuclear antibodies B. remove normal particles in her blood that are being damaged by autoantibodies C. exchange her plasma that contains antinuclear antibodies with a substitute fluid D. replace viral-damaged cellular components of her blood with replacement whole blood

When the nurse is admitting a patient who has acute rejection of an organ transplant, which of these already admitted patients will be the most appropriate roommate?

A patient who is recovering from an anaphylactic reaction to a bee sting

A clinical nurse specialist (CNS) is orienting a new licensed practical nurse to an oncology unit where blood product transfusions are frequently administered. In discussing ABO compatibility, the CNS presents several hypothetical scenarios. A well-informed new graduate would know the greatest likelihood of an acute hemolytic reaction would occur when giving:

A-positive blood to an A-negative client.

On initial assessment of an older patient, the nurse knows to look for certain types of diseases because which immunologic response increases with age? A. Autoimmune response B. Cell-mediated immunity C. Hypersensitivity response D. Humoral immune response

A. Autoimmune response With aging, autoantibodies increase, which lead to autoimmune diseases (e.g., systemic lupus erythematosus, acute glomerulonephritis, rheumatoid arthritis, hypothyroidism). Cell-mediated immunity decreases with decreased thymic output of T cells and decreased activation of both T and B cells. There is a decreased or absent delayed hypersensitivity reaction. Immunoglobulin levels decrease and lead to a suppressed humoral immune response in older adults.

On initial assessment of an older patient, the nurse knows to look for certain types of diseases because which immunologic response increases with age? A. Autoimmune response B. Cell-mediated immunity C. Hypersensitivity response D. Humoral immune response

A. Autoimmune response With aging, autoantibodies increase, which lead to autoimmune diseases (e.g., systemic lupus erythematosus, acute glomerulonephritis, rheumatoid arthritis, hypothyroidism). Cell-mediated immunity decreases with decreased thymic output of T cells and decreased activation of both T and B cells. There is a decreased or absent delayed hypersensitivity reaction. Immunoglobulin levels decrease and lead to a suppressed humoral immune response in older adults.

On initial assessment of an older patient, the nurse knows to look for certain types of diseases because which immunologic response increases with age? A. Autoimmune response B. Cell-mediated immunity C. Hypersensitivity response D. Humoral immune response

A. Autoimmune response With aging, autoantibodies increase, which lead to autoimmune diseases (e.g., systemic lupus erythematosus, acute glomerulonephritis, rheumatoid arthritis, hypothyroidism). Cell-mediated immunity decreases with decreased thymic output of T cells and decreased activation of both T and B cells. There is a decreased or absent delayed hypersensitivity reaction. Immunoglobulin levels decrease and lead to a suppressed humoral immune response in older adults.

When caring for a patient with a known latex allergy, the nurse would monitor the patient closely for a cross-sensitivity to which foods (select all that apply)? A. Grapes B. Oranges C. Bananas D. Potatoes E. Tomatoes

A. Grapes C. Bananas D. Potatoes E. Tomatoes Because some proteins in rubber are similar to food proteins, some foods may cause an allergic reaction in people who are allergic to latex. The most common of these foods are bananas, avocados, chestnuts, kiwi fruit, tomatoes, water chestnuts, guava, hazelnuts, potatoes, peaches, grapes, and apricots.

When caring for a patient with a known latex allergy, the nurse would monitor the patient closely for a cross-sensitivity to which foods (select all that apply)? A. Grapes B. Oranges C. Bananas D. Potatoes E. Tomatoes

A. Grapes C. Bananas D. Potatoes E. Tomatoes Because some proteins in rubber are similar to food proteins, some foods may cause an allergic reaction in people who are allergic to latex. The most common of these foods are bananas, avocados, chestnuts, kiwi fruit, tomatoes, water chestnuts, guava, hazelnuts, potatoes, peaches, grapes, and apricots.

When caring for a patient with a known latex allergy, the nurse would monitor the patient closely for a cross-sensitivity to which foods (select all that apply)? A. Grapes B. Oranges C. Bananas D. Potatoes E. Tomatoes

A. Grapes C. Bananas D. Potatoes E. Tomatoes Because some proteins in rubber are similar to food proteins, some foods may cause an allergic reaction in people who are allergic to latex. The most common of these foods are bananas, avocados, chestnuts, kiwi fruit, tomatoes, water chestnuts, guava, hazelnuts, potatoes, peaches, grapes, and apricots.

A 34-year-old female patient who has systemic lupus erythematosus is receiving plasmapheresis to treat an acute attack. What symptoms will the nurse monitor to determine if the patient develops complications related to the procedure? A. Hypotension, paresthesias, and dizziness B. Polyuria, decreased reflexes, and lethargy C. Intense thirst, flushed skin, and weight gain D. Abdominal cramping, diarrhea, and leg weakness

A. Hypotension, paresthesias, and dizziness Common complications associated with plasmapheresis are hypotension and citrate toxicity. Citrate is used as an anticoagulant and may cause hypocalcemia, which may manifest as headache, paresthesias, and dizziness. Polyuria, decreased reflexes, and lethargy are symptoms of hypercalcemia. Abdominal cramping, diarrhea, and leg weakness indicate hyperkalemia. Intense thirst, flushed skin, and weight gain indicate hypernatremia with normal or increased extracellular fluid volume.

A healthy 65-year-old man who lives at home is at the clinic requesting a "flu shot." When assessing the patient, what other vaccinations should the nurse ask the patient about receiving (select all that apply)? A. Shingles B. Pneumonia C. Meningococcal D. Haemophilus influenzae type b (Hib) E. Measles, mumps, and rubella (MMR)

A. Shingles B. Pneumonia The patient should receive the shingles (heres zoster) vaccine, Pneumovax, and influenza. The other options do not apply to this patient. Meningococcal vaccination is recommended for adults at risk (e.g., adults with anatomic or functional asplenia or persistent complement component deficiencies). Adults born before 1957 are generally considered immune to measles and mumps. Haemophilus influenzae type b (Hib) vaccination is only considered for adults with selected conditions (e.g., sickle cell disease, leukemia, HIV infection or for those who have anatomic or functional asplenia) if they have not been previously vaccinated.

A healthy 65-year-old man who lives at home is at the clinic requesting a "flu shot." When assessing the patient, what other vaccinations should the nurse ask the patient about receiving (select all that apply)? A. Shingles B. Pneumonia C. Meningococcal D. Haemophilus influenzae type b (Hib) E. Measles, mumps, and rubella (MMR)

A. Shingles B. Pneumonia The patient should receive the shingles (heres zoster) vaccine, Pneumovax, and influenza. The other options do not apply to this patient. Meningococcal vaccination is recommended for adults at risk (e.g., adults with anatomic or functional asplenia or persistent complement component deficiencies). Adults born before 1957 are generally considered immune to measles and mumps. Haemophilus influenzae type b (Hib) vaccination is only considered for adults with selected conditions (e.g., sickle cell disease, leukemia, HIV infection or for those who have anatomic or functional asplenia) if they have not been previously vaccinated.

the patient with HIV/AIDS tells the nurse that food tastes funny and is difficult to swallow. what is the nurses priority action at this time? a. Check the patients gag reflex b. ask about blood cultures c. examine the patient's mouth and throat d. collaborate with the dietitian to provide a soft diet

c

A 62-year-old patient has acquired immunodeficiency syndrome (AIDS), and the viral load is reported as undetectable. What patient teaching should be provided by the nurse related to this laboratory study result? A. The patient has the virus present and can transmit the infection to others. B. The patient is not able to transmit the virus to others through sexual contact. C. The patient will be prescribed lower doses of antiretroviral medications for 2 months. D. The syndrome has been cured, and the patient will be able to discontinue all medications.

A. The patient has the virus present and can transmit the infection to others. In human immunodeficiency virus (HIV) infections, viral loads are reported as real numbers of copies/μL or as undetectable. "Undetectable" indicates that the viral load is lower than the test is able to report. "Undetectable" does not mean that the virus has been eliminated from the body or that the individual can no longer transmit HIV to others.

The nurse is caring for a 2-year-old child with cancer. The parents have been told that the child will need an allogeneic bone marrow transplant and want to know what this means. What is the best response by the nurse? a) "The donor for this type of transplant must have the same DNA as your child." b) "This type of transplant uses the child's own stem cells for the procedure." c) "A donor is determined after testing for similar human leukocyte antigens." d) "The donor bone marrow can be accepted from anyone who volunteers."

c) "A donor is determined after testing for similar human leukocyte antigens." Explanation: An allogeneic transplant is one in which the donor and the recipient are related or unrelated but share similar human leukocyte antigens (HLA).

The nurse is caring for a client newly diagnosed with human immunodeficiency virus (HIV) obtained from unprotected sex. The nurse is in the room when the client is explaining the disease to another person. Which statement by the client would the nurse clarify? Select all that apply. a) "I will have this for the rest of my life." b) "The disease can also be spread by body fluids." c) "I am afraid that I will give this disease to my nephew." d) "I could pass this on to a baby before I give birth." e) "My sexual practices will have to change." f) "Medications can cure the disease."

c) "I am afraid that I will give this disease to my nephew." f) "Medications can cure the disease." Human immunodeficiency virus (HIV) is a sexually transmitted infection. Casual contact such as that with a family member will not spread the disease.Unfortunately, at this time, there is no cure for the disease. The client is correct in stating that sexual practices will have to change to prevent further spread of the disease, the disease can be spread by body fluids and can also be passed on to a fetus.

Two sons of a father who has Huntington's disease cannot agree on whether or not to be tested for Huntington's disease because of the cost. What assistance should the nurse give when discussing presymptomatic genetic testing with these men? a) "If one brother has the disease, the other brother will as well." b) "A positive genetic mutation increases your risk of the disease." c) "If there is a positive result, the patient will be diagnosed with the disease." d) "You could use a direct-to-consumer genetic test for making future life decisions."

c) "If there is a positive result, the patient will be diagnosed with the disease." With the autosomal dominant Huntington's disease, if the result is positive, the patient will develop the disease manifestations and can plan accordingly for his future. There is a 50% chance for each brother to be affected. Direct-to-consumer genetic testing should not be recommended because results may be misleading, are expensive, and may not be associated with genetic counseling which would be beneficial for these brothers. There is also currently no regulation of the use of the genetic information that these companies obtain.

A pregnant woman who was tested and diagnosed with HIV infection is very upset. What should the nurse teach this patient about her baby's risk of being born with HIV infection? a) "The baby will probably be infected with HIV." b) "Only an abortion will keep your baby from having HIV." c) "Treatment with antiretroviral therapy will decrease the baby's chance of HIV infection." d) "The duration and frequency of contact with the organism will determine if the baby gets HIV infection."

c) "Treatment with antiretroviral therapy will decrease the baby's chance of HIV infection." On average, 25% of infants born to women with untreated HIV will be born with HIV. The risk of transmission is reduced to less than 2% if the infected pregnant woman is treated with antiretroviral therapy. Duration and frequency of contact with the HIV organism is one variable that influences whether transmission of HIV occurs. Volume, virulence, and concentration of the organism as well as host immune status are variables related to transmission via blood, semen, vaginal secretions, or breast milk.

A patient who has vague symptoms of fatigue, headaches, and a positive test for human immunodeficiency virus (HIV) antibodies using an enzyme immunoassay (EIA) test. What instructions should the nurse give to this patient? a. "The EIA test will need to be repeated to verify the results." b. "A viral culture will be done to determine the progression of the disease." c. "It will probably be 10 or more years before you develop acquired immunodeficiency syndrome (AIDS)." d. "The Western blot test will be done to determine whether acquired immunodeficiency syndrome (AIDS) has developed."

ANS: A After an initial positive EIA test, the EIA is repeated before more specific testing such as the Western blot is done. Viral cultures are not usually part of HIV testing. It is not appropriate for the nurse to predict the time frame for AIDS development. The Western blot tests for HIV antibodies, not for AIDS

Which patient exposure by the nurse is most likely to require postexposure prophylaxis when the patient's human immunodeficiency virus (HIV) status is unknown? a. Needle stick with a needle and syringe used to draw blood b. Splash into the eyes when emptying a bedpan containing stool c. Contamination of open skin lesions with patient vaginal secretions d. Needle stick injury with a suture needle during a surgical procedure

ANS: A Puncture wounds are the most common means for workplace transmission of blood-borne diseases, and a needle with a hollow bore that had been contaminated with the patient's blood would be a high-risk situation. The other situations described would be much less likely to result in transmission of the virus

A patient is being evaluated for possible atopic dermatitis. The nurse expects elevation of which laboratory value? a. IgE b. IgA c. Basophils d. Neutrophils

ANS: A Serum IgE is elevated in an allergic response (type 1 hypersensitivity disorders). The eosinophil level will be elevated rather than neutrophil or basophil counts. IgA is located in body secretions and would not be tested when evaluating a patient who has symptoms of atopic dermatitis

An older adult who takes medications for coronary artery disease has just been diagnosed with asymptomatic chronic human immunodeficiency virus (HIV) infection. Which information will the nurse include in patient teaching? a. Many medications have interactions with antiretroviral drugs. b. Less frequent CD4+ level monitoring is needed in older adults. c. Hospice care is available for patients with terminal HIV infection. d. Progression of HIV infection occurs more rapidly in older patients.

ANS: A The nurse will teach the patient about potential interactions between antiretrovirals and the medications that the patient is using for chronic health problems. Treatment and monitoring of HIV infection is not affected by age. A patient with asymptomatic HIV infection is not a candidate for hospice. Progression of HIV is not affected by age, although it may be affected by chronic disease

The nurse cares for a patient infected with human immunodeficiency virus (HIV) who has just been diagnosed with asymptomatic chronic HIV infection. Which prophylactic measures will the nurse include in the plan of care (select all that apply)? a. Hepatitis B vaccine b. Pneumococcal vaccine c. Influenza virus vaccine d. Trimethoprim-sulfamethoxazole e. Varicella zoster immune globulin

ANS: A, B, C Asymptomatic chronic HIV infection is a stage between acute HIV infection and a diagnosis of symptomatic chronic HIV infection. Although called asymptomatic, symptoms (e.g., fatigue, headache, low-grade fever, night sweats) often occur. Prevention of other infections is an important intervention in patients who are HIV positive, and these vaccines are recommended as soon as the HIV infection is diagnosed. Antibiotics and immune globulin are used to prevent and treat infections that occur later in the course of the disease when the CD4+ counts have dropped or when infection has occurred

The nurse plans a presentation for community members about how to decrease the risk for antibiotic-resistant infections. Which information will the nurse include in the teaching plan (select all that apply)? a. Continue taking antibiotics until all the medication is gone. b. Antibiotics may sometimes be prescribed to prevent infection. c. Unused antibiotics that are more than a year old should be discarded. d. Antibiotics are effective in treating influenza associated with high fevers. e. Hand washing is effective in preventing many viral and bacterial infections.

ANS: A, B, E All prescribed doses of antibiotics should be taken. In some situations, such as before surgery, antibiotics are prescribed to prevent infection. There should not be any leftover antibiotics because all prescribed doses should be taken. However, if there are leftover antibiotics, they should be discarded immediately because the number left will not be enough to treat a future infection. Hand washing is generally considered the single most effective action in decreasing infection transmission. Antibiotics are ineffective in treating viral infections such as influenza

The nurse provides discharge instructions to a patient who has an immune deficiency involving the T lymphocytes. Which screening should the nurse include in the teaching plan for this patient? a. Screening for allergies b. Screening for malignancy c. Antibody deficiency screening d. Screening for autoimmune disorders

ANS: B Cell-mediated immunity is responsible for the recognition and destruction of cancer cells. Allergic reactions, autoimmune disorders, and antibody deficiencies are mediated primarily by B lymphocytes and humoral immunity

A pregnant woman with a history of asymptomatic chronic human immunodeficiency virus (HIV) infection is seen at the clinic. The patient states, "I am very nervous about making my baby sick." Which information will the nurse include when teaching the patient? a. The antiretroviral medications used to treat HIV infection are teratogenic. b. Most infants born to HIV-positive mothers are not infected with the virus. c. Because she is at an early stage of HIV infection, the infant will not contract HIV. d. It is likely that her newborn will become infected with HIV unless she uses antiretroviral therapy (ART).

ANS: B Only 25% of infants born to HIV-positive mothers develop HIV infection, even when the mother does not use ART during pregnancy. The percentage drops to 2% when ART is used. Perinatal transmission can occur at any stage of HIV infection (although it is less likely to occur when the viral load is lower). ART can safely be used in pregnancy, although some ART drugs should be avoided

A patient who uses injectable illegal drugs asks the nurse about preventing acquired immunodeficiency syndrome (AIDS). Which response by the nurse is best? a. "Avoid sexual intercourse when using injectable drugs." b. "It is important to participate in a needle-exchange program." c. "You should ask those who share equipment to be tested for HIV." d. "I recommend cleaning drug injection equipment before each use."

ANS: B Participation in needle-exchange programs has been shown to decrease and control the rate of HIV infection. Cleaning drug equipment before use also reduces risk, but it might not be consistently practiced. HIV antibodies do not appear for several weeks to months after exposure, so testing drug users would not be very effective in reducing risk for HIV exposure. It is difficult to make appropriate decisions about sexual activity when under the influence of drugs

The nurse teaches a patient diagnosed with systemic lupus erythematosus (SLE) about plasmapheresis. What instructions about plasmapheresis should the nurse include in the teaching plan? a. Plasmapheresis will eliminate eosinophils and basophils from blood. b. Plasmapheresis will remove antibody-antigen complexes from circulation. c. Plasmapheresis will prevent foreign antibodies from damaging various body tissues. d. Plasmapheresis will decrease the damage to organs caused by attacking T lymphocytes.

ANS: B Plasmapheresis is used in SLE to remove antibodies, antibody-antigen complexes, and complement from blood. T lymphocytes, foreign antibodies, eosinophils, and basophils do not directly contribute to the tissue damage in SLE

Eight years after seroconversion, a human immunodeficiency virus (HIV)-infected patient has a CD4+ cell count of 800/µL and an undetectable viral load. What is the priority nursing intervention at this time? a. Teach about the effects of antiretroviral agents. b. Encourage adequate nutrition, exercise, and sleep. c. Discuss likelihood of increased opportunistic infections. d. Monitor for symptoms of acquired immunodeficiency syndrome (AIDS).

ANS: B The CD4+ level for this patient is in the normal range, indicating that the patient is the stage of asymptomatic chronic infection, when the body is able to produce enough CD4+ cells to maintain a normal CD4+ count. AIDS and increased incidence of opportunistic infections typically develop when the CD4+ count is much lower than normal. Although the initiation of ART is highly individual, it would not be likely that a patient with a normal CD4+ level would receive ART

A patient who is anxious and has difficulty breathing seeks treatment after being stung by a wasp. What is the nurse's priority action? a. Have the patient lie down. b. Assess the patient's airway. c. Administer high-flow oxygen. d. Remove the stinger from the site.

ANS: B The initial action with any patient with difficulty breathing is to assess and maintain the airway. The other actions also are part of the emergency management protocol for anaphylaxis, but the priority is airway maintenance

An adolescent patient seeks care in the emergency department after sharing needles for heroin injection with a friend who has hepatitis B. To provide immediate protection from infection, what medication will the nurse administer? a. Corticosteroids b. Gamma globulin c. Hepatitis B vaccine d. Fresh frozen plasma

ANS: B The patient should first receive antibodies for hepatitis B from injection of gamma globulin. The hepatitis B vaccination series should be started to provide active immunity. Fresh frozen plasma and corticosteroids will not be effective in preventing hepatitis B in the patient

According to the Center for Disease Control (CDC) guidelines, which personal protective equipment will the nurse put on when assessing a patient who is on contact precautions for diarrhea caused by Clostridium difficile (select all that apply)? a. Mask b. Gown c. Gloves d. Shoe covers e. Eye protection

ANS: B, C Because the nurse will have substantial contact with the patient and bedding when doing an assessment, gloves and gowns are needed. Eye protection and masks are needed for patients in contact precautions only when spraying or splashing is anticipated. Shoe covers are not recommended in the CDC guidelines

A patient with a positive rapid antibody test result for human immunodeficiency virus (HIV) is anxious and does not appear to hear what the nurse is saying. What action by the nurse is most important at this time? a. Teach the patient about the medications available for treatment. b. Inform the patient how to protect sexual and needle-sharing partners. c. Remind the patient about the need to return for retesting to verify the results. d. Ask the patient to notify individuals who have had risky contact with the patient.

ANS: C After an initial positive antibody test, the next step is retesting to confirm the results. A patient who is anxious is not likely to be able to take in new information or be willing to disclose information about HIV status of other individuals

Which teaching should the nurse provide about intradermal skin testing to a patient with possible allergies? a. "Do not eat anything for about 6 hours before the testing." b. "Take an oral antihistamine about an hour before the testing." c. "Plan to wait in the clinic for 20 to 30 minutes after the testing." d. "Reaction to the testing will take about 48 to 72 hours to occur."

ANS: C Allergic reactions usually occur within minutes after injection of an allergen, and the patient will be monitored for at least 20 minutes for anaphylactic reactions after the testing. Medications that might modify the response, such as antihistamines, should be avoided before allergy testing. There is no reason to be NPO for skin testing. Results with intradermal testing occur within minutes

The health care provider asks the nurse whether a patient's angioedema has responded to prescribed therapies. Which assessment should the nurse perform? a. Ask the patient about any clear nasal discharge. b. Obtain the patient's blood pressure and heart rate. c. Check for swelling of the patient's lips and tongue. d. Assess the patient's extremities for wheal and flare lesions.

ANS: C Angioedema is characterized by swelling of the eyelids, lips, and tongue. Wheal and flare lesions, clear nasal drainage, and hypotension and tachycardia are characteristic of other allergic reactions.

Immediately after the nurse administers an intracutaneous injection of an allergen on the forearm, a patient complains of itching at the site and of weakness and dizziness. What action should the nurse take first? a. Remind the patient to remain calm. b. Administer subcutaneous epinephrine. c. Apply a tourniquet above the injection site. d. Rub a local antiinflammatory cream on the site.

ANS: C Application of a tourniquet will decrease systemic circulation of the allergen and should be the first reaction. A local antiinflammatory cream may be applied to the site of a cutaneous test if the itching persists. Epinephrine will be needed if the allergic reaction progresses to anaphylaxis. The nurse should assist the patient to remain calm, but this is not an adequate initial nursing action

A patient who collects honey to earn supplemental income has developed a hypersensitivity to bee stings. Which statement, if made by the patient, would indicate a need for additional teaching? a. "I need to find another way to earn extra money." b. "I will get a prescription for epinephrine and learn to self-inject it." c. "I will plan to take oral antihistamines daily before going to work." d. "I should wear a Medic-Alert bracelet indicating my allergy to bee stings."

ANS: C Because the patient is at risk for bee stings and the severity of allergic reactions tends to increase with added exposure to allergen, taking oral antihistamines will not adequately control the patient's hypersensitivity reaction. The other patient statements indicate a good understanding of management of the problem

The nurse, who is reviewing a clinic patient's medical record, notes that the patient missed the previous appointment for weekly immunotherapy. Which action by the nurse is most appropriate? a. Schedule an additional dose that week. b. Administer the usual dosage of the allergen. c. Consult with the health care provider about giving a lower allergen dose. d. Re-evaluate the patient's sensitivity to the allergen with a repeat skin test.

ANS: C Because there is an increased risk for adverse reactions after a patient misses a scheduled dose of allergen, the nurse should check with the health care provider before administration of the injection. A skin test is used to identify the allergen and would not be used at this time. An additional dose for the week may increase the risk for a reaction.

A new mother expresses concern about her baby developing allergies and asks what the health care provider meant by "passive immunity." Which example should the nurse use to explain this type of immunity? a. Early immunization b. Bone marrow donation c. Breastfeeding her infant d. Exposure to communicable diseases

ANS: C Colostrum provides passive immunity through antibodies from the mother. These antibodies protect the infant for a few months. However, memory cells are not retained, so the protection is not permanent. Active immunity is acquired by being immunized with vaccinations or having an infection. It requires that the infant has an immune response after exposure to an antigen. Cell-mediated immunity is acquired through T lymphocytes and is a form of active immunity

A young adult female patient who is human immunodeficiency virus (HIV)-positive has a new prescription for efavirenz (Sustiva). Which information is most important to include in the medication teaching plan? a. Driving is allowed when starting this medication. b. Report any bizarre dreams to the health care provider. c. Continue to use contraception while on this medication. d. Take this medication in the morning on an empty stomach.

ANS: C Efavirenz can cause fetal anomalies and should not be used in patients who may be pregnant. The drug should not be used during pregnancy because large doses could cause fetal anomalies. Once-a-day doses should be taken at bedtime (at least initially) to help patients cope with the side effects that include dizziness and confusion. Patients should be cautioned about driving when starting this drug. Patients should be informed that many people who use the drug have reported vivid and sometimes bizarre dreams

The nurse palpates enlarged cervical lymph nodes on a patient diagnosed with acute human immunodeficiency virus (HIV) infection. Which action would be most appropriate for the nurse to take? a. Instruct the patient to apply ice to the neck. b. Advise the patient that this is probably the flu. c. Explain to the patient that this is an expected finding. d. Request that an antibiotic be prescribed for the patient.

ANS: C Persistent generalized lymphadenopathy is common in the early stages of HIV infection. No antibiotic is needed because the enlarged nodes are probably not caused by bacteria. Applying ice to the neck may provide comfort, but the initial action is to reassure the patient this is an expected finding. Lymphadenopathy is common with acute HIV infection and is therefore not likely the flu

A nurse has obtained donor tissue typing information about a patient who is waiting for a kidney transplant. Which results should be reported to the transplant surgeon? a. Patient is Rh positive and donor is Rh negative b. Six antigen matches are present in HLA typing c. Results of patient-donor cross matching are positive d. Panel of reactive antibodies (PRA) percentage is low

ANS: C Positive crossmatching is an absolute contraindication to kidney transplantation, since a hyperacute rejection will occur after the transplant. The other information indicates that the tissue match between the patient and potential donor is acceptable

The nurse designs a program to decrease the incidence of human immunodeficiency virus (HIV) infection in the adolescent and young adult populations. Which information should the nurse assign as the highest priority? a. Methods to prevent perinatal HIV transmission b. Ways to sterilize needles used by injectable drug users c. Prevention of HIV transmission between sexual partners d. Means to prevent transmission through blood transfusions

ANS: C Sexual transmission is the most common way that HIV is transmitted. The nurse should also provide teaching about perinatal transmission, needle sterilization, and blood transfusion, but the rate of HIV infection associated with these situations is lower

Which patient should the nurse assess first? a. Patient with urticaria after receiving an IV antibiotic b. Patient who has graft-versus-host disease and severe diarrhea c. Patient who is sneezing after having subcutaneous immunotherapy d. Patient with multiple chemical sensitivities who has muscle stiffness

ANS: C Sneezing after subcutaneous immunotherapy may indicate impending anaphylaxis and assessment and emergency measures should be initiated. The other patients also have findings that need assessment and intervention by the nurse, but do not have evidence of life-threatening complications

Which of these patients being seen at the human immunodeficiency virus (HIV) clinic should the nurse assess first? a. Patient whose latest CD4+ count is 250/µL b. Patient whose rapid HIV-antibody test is positive c. Patient who has had 10 liquid stools in the last 24 hours d. Patient who has nausea from prescribed antiretroviral drugs

ANS: C The nurse should assess the patient for dehydration and hypovolemia. The other patients also will require assessment and possible interventions, but do not require immediate action to prevent complications such as hypovolemia and shock

The nurse cares for a patient who is human immunodeficiency virus (HIV) positive and taking antiretroviral therapy (ART). Which information is most important for the nurse to address when planning care? a. The patient's blood glucose level is 142 mg/dL. b. The patient complains of feeling "constantly tired." c. The patient is unable to state the side effects of the medications. d. The patient states, "Sometimes I miss a dose of zidovudine (AZT)."

ANS: D Because missing doses of ART can lead to drug resistance, this patient statement indicates the need for interventions such as teaching or changes in the drug scheduling. Elevated blood glucose and fatigue are common side effects of ART. The nurse should discuss medication side effects with the patient, but this is not as important as addressing the skipped doses of AZT

The nurse will most likely prepare a medication teaching plan about antiretroviral therapy (ART) for which patient? a. Patient who is currently HIV negative but has unprotected sex with multiple partners b. Patient who was infected with HIV 15 years ago and now has a CD4+ count of 840/µL c. HIV-positive patient with a CD4+ count of 160/µL who drinks a fifth of whiskey daily d. Patient who tested positive for HIV 2 years ago and now has cytomegalovirus (CMV) retinitis

ANS: D CMV retinitis is an acquired immunodeficiency syndrome (AIDS)-defining illness and indicates that the patient is appropriate for ART even though the HIV infection period is relatively short. An HIV-negative patient would not be offered ART. A patient with a CD4+ count in the normal range would not typically be started on ART. A patient who drinks alcohol heavily would be unlikely to be able to manage the complex drug regimen and would not be appropriate for ART despite the low CD4+ count

An older adult patient has a prescription for cyclosporine following a kidney transplant. Which information in the patient's health history has the most implications for planning patient teaching about the medication at this time? a. The patient restricts salt to treat prehypertension. b. The patient drinks 3 to 4 quarts of fluids every day. c. The patient has many concerns about the effects of cyclosporine. d. The patient has a glass of grapefruit juice every day for breakfast.

ANS: D Grapefruit juice can increase the toxicity of cyclosporine. The patient should be taught to avoid grapefruit juice. High fluid intake will not affect cyclosporine levels or renal function. Cyclosporine may cause hypertension, and the patient's many concerns should be addressed, but these are not potentially life-threatening problems

A clinic patient is experiencing an allergic reaction to an unknown allergen. Which action is most appropriate for the registered nurse (RN) to delegate to a licensed practical/vocational nurse (LPN/LVN)? a. Perform a focused physical assessment. b. Obtain the health history from the patient. c. Teach the patient about the various diagnostic studies. d. Administer skin testing by the cutaneous scratch method.

ANS: D LPN/LVNs are educated and licensed to administer medications under the supervision of an RN. RN-level education and the scope of practice include assessment of health history, focused physical assessment, and patient teaching

A patient who is diagnosed with acquired immunodeficiency syndrome (AIDS) tells the nurse, "I feel obsessed with thoughts about dying. Do you think I am just being morbid?" Which response by the nurse is best? a. "Thinking about dying will not improve the course of AIDS." b. "It is important to focus on the good things about your life now." c. "Do you think that taking an antidepressant might be helpful to you?" d. "Can you tell me more about the kind of thoughts that you are having?"

ANS: D More assessment of the patient's psychosocial status is needed before taking any other action. The statements, "Thinking about dying will not improve the course of AIDS" and "It is important to focus on the good things in life" discourage the patient from sharing any further information with the nurse and decrease the nurse's ability to develop a trusting relationship with the patient. Although antidepressants may be helpful, the initial action should be further assessment of the patient's feelings

The nurse is providing care for a patient who has been living with HIV for several years. Which assessment finding most clearly indicates an acute exacerbation of the disease? a) A new onset of polycythemia b) Presence of mononucleosis-like symptoms c) A sharp decrease in the patient's CD4+ count d) A sudden increase in the patient's WBC count

c) A sharp decrease in the patient's CD4+ count A decrease in CD4+ count signals an exacerbation of the severity of HIV. Polycythemia is not characteristic of the course of HIV. A patient's WBC count is very unlikely to suddenly increase, with decreases being typical. Mononucleosis-like symptoms such as malaise, headache, and fatigue are typical of early HIV infection and seroconversion.

The nurse is caring for a client being discharged following kidney transplantation. The client is ordered mofetil to prevent organ rejection. Which nursing instruction is essential regarding medication use? a) Administer medication following breakfast daily. b) Sprinkle the contents of the capsule on food. c) Contact the health care provider at first signs of an infection. d) Administer the medication with an antacid to prevent stomach upset.

c) Contact the health care provider at first signs of an infection. Mofetil is an organ rejection medication that diminishes the body's ability to identify and eliminate pathogens (immunosuppressant). Identifying symptoms of infection at an early state is helpful in treating the infection. This medication is administered on an empty stomach. Typically, capsules would not be opened dispensing medication at one time. Antacids may decrease the absorption of the medication

A 2-month-old infant hasn't received any immunizations. Which immunizations should the nurse prepare to administer? a) Polio (IPV), DTaP, MMR b) Varicella, Haemophilus influenzae type b (HIB), IPV, and DTaP c) HIB, DTaP, HepB, IPV, and pneumococcal conjugate vaccine (PCV) d) Measles, mumps, rubella (MMR); diphtheria, tetanus toxoids, and acellular pertussis (DTaP); and hepatitis B (HepB)

c) HIB, DTaP, HepB, IPV, and pneumococcal conjugate vaccine (PCV) The current immunizations recommended for a 2-month-old who hasn't received any immunizations are HIB, DTaP, HepB, PCV, and IPV. The first immunizations for MMR and varicella are recommended when a child is age 12 months.

1. A child is admitted to the pediatric unit with a diagnosis of suspected meningococcal meningitis. Which of the following nursing measures should the nurse do FIRST? a. Institute seizure precautions b. Assess neurologic status c. Place in respiratory isolation d. Assess vital signs

Answer C. The initial therapeutic management of acute bacterial meningitis includes isolation precautions, initiation of antimicrobial therapy and maintenance of optimum hydration. Nurses should take necessary precautions to protect themselves and others from possible infection.

A patient who has been treated for HIV infection for 7 years has developed fat redistribution to the trunk, with wasting of the arms, legs, and face. The nurse will anticipate teaching the patient about a. treatment with antifungal agents. b. a change in antiretroviral therapy. c. foods that are higher in protein. d. the benefits of daily exercise.

B Rationale: A frequent first intervention for metabolic disorders is a change in ART. Treatment with antifungal agents would not be appropriate because there is no indication of fungal infection. Changes in diet or exercise have not proven helpful for this problem. Cognitive Level: Application Text Reference: pp. 266-267 Nursing Process: Planning NCLEX: Physiological Integrity

Which of these patients will the nurse working in an HIV testing and treatment clinic anticipate teaching about ART? a. A patient who is HIV negative but has unprotected sex with multiple partners b. A patient who has been HIV positive for 5 years and has cytomegalovirus (CMV) retinitis c. A patient who was infected with HIV 15 years ago and has a CD4 count of 740/µl d. An HIV-positive patient with a CD4 count of 120/µl who drinks a fifth of whiskey daily

B Rationale: CMV retinitis is an AIDS-defining illness and indicates that the patient is appropriate for ART even though the HIV infection period is relatively short. An HIV-negative patient would not be offered ART. A patient with a CD4+ count in the normal range would not require ART. A patient who drinks alcohol heavily would be unlikely to be able to manage the complex drug regimen and would not be appropriate for ART despite the low CD4+ count. Cognitive Level: Application Text Reference: p. 253 Nursing Process: Planning NCLEX: Physiological Integrity

A patient with HIV infection has developed Mycobacterium avium complex infection. An appropriate outcome for the patient is that the patient will a. be free from injury. b. maintain intact perineal skin. c. have adequate oxygenation. d. receive immunizations.

B Rationale: The major manifestation of M. avium infection is loose, watery stools, which would increase the risk for perineal skin breakdown. The other outcomes would be appropriate for other complications (pneumonia, dementia, influenza, etc) associated with HIV infection. Cognitive Level: Analysis Text Reference: p. 255 Nursing Process: Planning NCLEX: Physiological Integrity

A 58-year-old man who is waiting for a kidney transplant asks the nurse to explain the difference between a negative and positive cross match. Which statement by the nurse would be the most accurate response? A. "A negative cross match means that both the donor and recipient are Rh negative, and the transplant is safe." B. "A negative cross match means that no preformed antibodies are present and the transplant would be safe." C. "A positive cross match means the blood type is the same between donor and recipient, and the transplant is safe." D. "A positive cross match means that both the donor and the recipient have antigens that are similar, and the transplant would be safe."

B. "A negative cross match means that no preformed antibodies are present and the transplant would be safe." A cross match uses serum from the recipient mixed with donor lymphocytes to test for any preformed antibodies to the potential donor organ. A positive cross match indicates that the recipient has cytotoxic antibodies to the donor and is an absolute contraindication to transplantation. A negative cross match indicates that no preformed antibodies are present and it is safe to proceed with transplantation.

A 58-year-old man who is waiting for a kidney transplant asks the nurse to explain the difference between a negative and positive cross match. Which statement by the nurse would be the most accurate response? A. "A negative cross match means that both the donor and recipient are Rh negative, and the transplant is safe." B. "A negative cross match means that no preformed antibodies are present and the transplant would be safe." C. "A positive cross match means the blood type is the same between donor and recipient, and the transplant is safe." D. "A positive cross match means that both the donor and the recipient have antigens that are similar, and the transplant would be safe."

B. "A negative cross match means that no preformed antibodies are present and the transplant would be safe." A cross match uses serum from the recipient mixed with donor lymphocytes to test for any preformed antibodies to the potential donor organ. A positive cross match indicates that the recipient has cytotoxic antibodies to the donor and is an absolute contraindication to transplantation. A negative cross match indicates that no preformed antibodies are present and it is safe to proceed with transplantation.

A 22-year-old man who smokes 2 packs of cigarettes per day tells the nurse, "It does not matter what I do because every man in my family dies before age 50 of a heart attack." What information should the nurse provide? a) If there is a family history of heart disease, tobacco smoking adds nothing to the level of risk. b) Gene therapy can decrease the risk of future heart disease for the patient and his other relatives. c) Heart disease usually results from a combination of factors, including behaviors that can be changed. d) There is no point in being screened for heart disease if you already have a strong family history of the disease.

c) Heart disease usually results from a combination of factors, including behaviors that can be changed. People with a family history of disease may have the most to gain from lifestyle changes and screening tests. People cannot change their genes, but they can change unhealthy behaviors (e.g., smoking, poor eating habits) and have screening tests to detect risk factors (e.g., elevated cholesterol, hypertension) that can be treated.

A 58-year-old man who is waiting for a kidney transplant asks the nurse to explain the difference between a negative and positive cross match. Which statement by the nurse would be the most accurate response? A. "A negative cross match means that both the donor and recipient are Rh negative, and the transplant is safe." B. "A negative cross match means that no preformed antibodies are present and the transplant would be safe." C. "A positive cross match means the blood type is the same between donor and recipient, and the transplant is safe." D. "A positive cross match means that both the donor and the recipient have antigens that are similar, and the transplant would be safe."

B. "A negative cross match means that no preformed antibodies are present and the transplant would be safe." A cross match uses serum from the recipient mixed with donor lymphocytes to test for any preformed antibodies to the potential donor organ. A positive cross match indicates that the recipient has cytotoxic antibodies to the donor and is an absolute contraindication to transplantation. A negative cross match indicates that no preformed antibodies are present and it is safe to proceed with transplantation.

Which statement made by the nurse is most appropriate in teaching patient interventions to minimize the effects of seasonal allergic rhinitis? A. "You will need to get rid of your pets." B. "You should sleep in an air-conditioned room." C. "You would do best to stay indoors during the winter months." D. "You will need to dust your house with a dry feather duster twice a week."

B. "You should sleep in an air-conditioned room." Seasonal allergic rhinitis is most commonly caused by pollens from trees, weeds, and grasses. Airborne allergies can be controlled by sleeping in an air-conditioned room, daily damp dusting, covering the mattress and pillows with hypoallergenic covers, and wearing a mask outdoors.

Which statement made by the nurse is most appropriate in teaching patient interventions to minimize the effects of seasonal allergic rhinitis? A. "You will need to get rid of your pets." B. "You should sleep in an air-conditioned room." C. "You would do best to stay indoors during the winter months." D. "You will need to dust your house with a dry feather duster twice a week."

B. "You should sleep in an air-conditioned room." Seasonal allergic rhinitis is most commonly caused by pollens from trees, weeds, and grasses. Airborne allergies can be controlled by sleeping in an air-conditioned room, daily damp dusting, covering the mattress and pillows with hypoallergenic covers, and wearing a mask outdoors.

Which statement made by the nurse is most appropriate in teaching patient interventions to minimize the effects of seasonal allergic rhinitis? A. "You will need to get rid of your pets." B. "You should sleep in an air-conditioned room." C. "You would do best to stay indoors during the winter months." D. "You will need to dust your house with a dry feather duster twice a week."

B. "You should sleep in an air-conditioned room." Seasonal allergic rhinitis is most commonly caused by pollens from trees, weeds, and grasses. Airborne allergies can be controlled by sleeping in an air-conditioned room, daily damp dusting, covering the mattress and pillows with hypoallergenic covers, and wearing a mask outdoors.

The parents of a child diagnosed with cystic fibrosis ask the nurse what happened to cause this disease. What is the best response by the nurse? a) It is X-linked so it was passed to the child from the mother. b) It is a chromosome disorder that usually skips a generation. c) It is autosomal recessive so both copies of the gene are abnormal. d) It is autosomal dominant so the abnormal gene allele is expressed instead of the normal allele.

c) It is autosomal recessive so both copies of the gene are abnormal. Cystic fibrosis is an autosomal recessive disorder, which means both of the genes in the pair on the chromosome are abnormal. Cystic fibrosis is not X-linked, from a chromosome disorder, or autosomal dominant.

In a client infected with human immunodeficiency virus (HIV) has a low CD4 level. What interventions should the nurse implement as a result of this finding? a) Request human granulocyte colony-stimulating factor to improve WBC production. b) Provide antibiotics as per order. c) Place the client in reverse isolation. d) Increase nutritional protein with each meal.

c) Place the client in reverse isolation. CD4+ levels in the blood of an individual with HIV infection determine the extent of damage to the individual's immune system. The test indicates the individual's risk of an opportunistic infection, but does not identify specific infections. Viral loads and resistance to specific antigens are determined using other diagnostic tests. Because of the client's risk, isolation is recommended

The nurse is teaching a group of young adults who live in a dormitory about the prevention of antibiotic-resistant infections. What should be included in the teaching plan? A. Save leftover antibiotics for future uses. B. Hand washing can prevent many infections. C. Antibiotics are indicated for preventing most colds. D. Stop taking prescribed antibiotics when symptoms improve.

B. Hand washing can prevent many infections. Hand washing is the single most important action to prevent infections. Antibiotics are used to treat bacterial infections, not viral colds and flu. Patients should complete the entire prescription of antibiotics to prevent the development of resistant bacteria. Antibiotics should not be taken to prevent infections unless they are given prophylactically before undergoing certain surgeries and dental work.

The nurse is teaching a group of young adults who live in a dormitory about the prevention of antibiotic-resistant infections. What should be included in the teaching plan? A. Save leftover antibiotics for future uses. B. Hand washing can prevent many infections. C. Antibiotics are indicated for preventing most colds. D. Stop taking prescribed antibiotics when symptoms improve.

B. Hand washing can prevent many infections. Hand washing is the single most important action to prevent infections. Antibiotics are used to treat bacterial infections, not viral colds and flu. Patients should complete the entire prescription of antibiotics to prevent the development of resistant bacteria. Antibiotics should not be taken to prevent infections unless they are given prophylactically before undergoing certain surgeries and dental work.

The woman with ovarian cancer would like to know which kind of genetic testing could help prevent her daughters from getting ovarian cancer. What should the nurse tell this patient? a) Forensic testing b) Carrier screening c) Predictive testing d) Prenatal diagnostic testing

c) Predictive testing Predictive genetic testing can be done to find mutated BRCA1 or BRCA2 genes. People who have these genetic mutations can elect to have a prophylactic oophorectomy to prevent the development of the cancer. Diagnostic testing can also identify genetic conditions. Forensic testing is done to identify an individual for legal purposes. Carrier screening identifies an unaffected individual who carries one copy of a specific gene and could pass it to next generations. Prenatal diagnostic genetic testing is done to detect changes in genes or chromosomes of a fetus before birth.

A patient's low hemoglobin and hematocrit have necessitated a transfusion of packed red blood cells (RBCs). Shortly after the first unit of RBCs starts to infuse, the patient develops signs and symptoms of a transfusion reaction. Which type of hypersensitivity reaction has the patient experienced? A. Type I B. Type II C. Type III D. Type IV

B. Type II Transfusion reactions are characterized as a type II (cytotoxic) reaction in which agglutination and cytolysis occur. Type I hypersensitivity reactions are IgE-mediated reactions to specific allergens (e.g., exogenous pollen, food, drugs, or dust). Type III reactions are immune-complex reactions that occur secondary to antigen-antibody complexes. Type IV reactions are delayed cell-mediated immune response reactions.

A patient's low hemoglobin and hematocrit have necessitated a transfusion of packed red blood cells (RBCs). Shortly after the first unit of RBCs starts to infuse, the patient develops signs and symptoms of a transfusion reaction. Which type of hypersensitivity reaction has the patient experienced? A. Type I B. Type II C. Type III D. Type IV

B. Type II Transfusion reactions are characterized as a type II (cytotoxic) reaction in which agglutination and cytolysis occur. Type I hypersensitivity reactions are IgE-mediated reactions to specific allergens (e.g., exogenous pollen, food, drugs, or dust). Type III reactions are immune-complex reactions that occur secondary to antigen-antibody complexes. Type IV reactions are delayed cell-mediated immune response reactions.

A patient has recently tested positive for HIV and asks the nurse about drug therapy for HIV infection. The nurse informs the patient that a. drug therapy for HIV is indicated only for patients whose CD4+ cell counts indicate that AIDS has developed. b. medication therapy is delayed as long as possible to prevent development of viral resistance to the drugs. c. treatment is individualized based on CD4+ counts, the amount of virus in the blood, and the patient's wishes. d. ART is typically started soon after HIV diagnosis to prevent progression of the disease.

C Rationale: ART is typically considered when the CD4+ count drops below normal levels or the viral load is high in patients who are appropriate for ART and desire ART. ART is used to prevent the progression to AIDS and is used in patients who have AIDS. ART is not delayed as long as possible but can be started when the CD4+ counts are relatively high in some patients. ART is not started soon after HIV diagnosis; rather, it is started when CD4+ count, viral load, or patient symptoms indicate that it will be beneficial. Cognitive Level: Application Text Reference: pp. 256-257 Nursing Process: Implementation NCLEX: Physiological Integrity

A patient who tested positive for HIV 3 years ago is admitted to the hospital with Pneumocystis jiroveci pneumonia (PCP). Based on diagnostic criteria established by the Centers for Disease Control and Prevention (CDC), the patient is diagnosed as having a. early chronic infection. b. HIV infection. c. AIDS. d. intermediate chronic infection.

C Rationale: Development of PCP pneumonia meets the diagnostic criterion for AIDS. The other responses indicate an earlier stage of HIV infection than is indicated by the PCP infection. Cognitive Level: Comprehension Text Reference: p. 253 Nursing Process: Assessment NCLEX: Physiological Integrity

The occupational health nurse will teach the nursing staff that the highest risk of acquiring HIV from an HIV-infected patient is a. a needlestick with a suture needle during a surgical procedure. b. contamination of open skin lesions with vaginal secretions. c. a needlestick with a needle and syringe used to draw blood. d. splashing the eyes when emptying a bedpan containing stool.

C Rationale: Puncture wounds are the most common means for workplace transmission of blood-borne diseases, and a needle with a hollow bore that had been contaminated with the patient's blood would be a high-risk situation. The other situations described would be much less likely to result in transmission of the virus. Cognitive Level: Comprehension Text Reference: p. 250 Nursing Process: Assessment NCLEX: Safe and Effective Care Environment

Interventions such as promotion of nutrition, exercise, and stress reduction should be promoted by the nurse for patients who have HIV infection, primarily because these interventions will a. promote a feeling of well-being in the patient. b. prevent transmission of the virus to others. c. improve the patient's immune function. d. increase the patient's strength and self-care ability.

C Rationale: The primary goal for the patient with HIV infection is to increase immune function, and these interventions will promote a healthy immune system. They may also promote a feeling of well-being and increase strength, but these are not the priority goals for HIV-positive patients. These activities will not prevent the risk for transmission to others because the patient will still be HIV positive. Cognitive Level: Comprehension Text Reference: p. 265 Nursing Process: Planning NCLEX: Physiological Integrity

A 34-year-old multigravida at 36 weeks' gestation in active labor has been diagnosed with Rh sensitization. The fetus is in a frank breech presentation. The client's membranes rupture spontaneously, and the nurse documents the color of the fluid as yellowish. This color indicates: a) amniotic fluid embolism. b) oligohydramnios. c) Rh sensitization. d) abnormal presentation.

c) Rh sensitization. Amniotic fluid is normally clear. Yellowish fluid indicates Rh sensitization. The yellowish color is related to fetal anemia and bilirubin in the amniotic fluid. In an abnormal presentation, in this case a breech presentation, it is not uncommon for the amniotic fluid to be green in color owing to meconium expelled by the fetus. Amniotic fluid embolism is not related to the fluid color. This condition, a medical emergency, may occur naturally after a difficult labor or from hyperstimulation of the uterus. Oligohydramnios refers to a markedly decreased volume of amniotic fluid. It has no association with the color of the fluid.

The nurse is monitoring the effectiveness of antiretroviral therapy (ART) for a 56-year-old man with acquired immunodeficiency syndrome (AIDS). What laboratory study result indicates the medications have been effective? A. Increased viral load B. Decreased neutrophil count C. Increased CD4+ T cell count D. Decreased white blood cell count

C. Increased CD4+ T cell count Antiretroviral therapy is effective if there are decreased viral loads and increased CD4+ T cell counts.

The nurse is monitoring the effectiveness of antiretroviral therapy (ART) for a 56-year-old man with acquired immunodeficiency syndrome (AIDS). What laboratory study result indicates the medications have been effective? A. Increased viral load B. Decreased neutrophil count C. Increased CD4+ T cell count D. Decreased white blood cell count

C. Increased CD4+ T cell count Antiretroviral therapy is effective if there are decreased viral loads and increased CD4+ T cell counts.

The patient with an autoimmune disease will be treated with plasmapheresis. What should the nurse teach the patient about this treatment? A. It will gather platelets for use later when needed. B. It will cause anemia because it removes whole blood and RBCs are damaged. C. It will remove the IgG autoantibodies and antigen complexes from the plasma. D. It will remove the peripheral stem cells in order to cure the autoimmune disease.

C. It will remove the IgG autoantibodies and antigen complexes from the plasma. Plasmapheresis removes plasma that contains autoantibodies (usually IgG class) and antigen-antibody complexes to remove the pathologic substances in the plasma without causing anemia. Plateletpheresis removes platelets from normal individuals for use by patients with low platelet counts. Apheresis is used to collect stem cells from peripheral blood that does not cure autoimmune disease.

The patient with an autoimmune disease will be treated with plasmapheresis. What should the nurse teach the patient about this treatment? A. It will gather platelets for use later when needed. B. It will cause anemia because it removes whole blood and RBCs are damaged. C. It will remove the IgG autoantibodies and antigen complexes from the plasma. D. It will remove the peripheral stem cells in order to cure the autoimmune disease.

C. It will remove the IgG autoantibodies and antigen complexes from the plasma. Plasmapheresis removes plasma that contains autoantibodies (usually IgG class) and antigen-antibody complexes to remove the pathologic substances in the plasma without causing anemia. Plateletpheresis removes platelets from normal individuals for use by patients with low platelet counts. Apheresis is used to collect stem cells from peripheral blood that does not cure autoimmune disease.

The patient with an autoimmune disease will be treated with plasmapheresis. What should the nurse teach the patient about this treatment? A. It will gather platelets for use later when needed. B. It will cause anemia because it removes whole blood and RBCs are damaged. C. It will remove the IgG autoantibodies and antigen complexes from the plasma. D. It will remove the peripheral stem cells in order to cure the autoimmune disease

C. It will remove the IgG autoantibodies and antigen complexes from the plasma. Plasmapheresis removes plasma that contains autoantibodies (usually IgG class) and antigen-antibody complexes to remove the pathologic substances in the plasma without causing anemia. Plateletpheresis removes platelets from normal individuals for use by patients with low platelet counts. Apheresis is used to collect stem cells from peripheral blood that does not cure autoimmune disease.

A 21-year-old student had taken amoxicillin once as a child for an ear infection. She is given an injection of Penicillin V and develops a systemic anaphylactic reaction. What manifestations would be seen first? A. Dyspnea B. Dilated pupils C. Itching and edema D. Wheal-and-flare reaction

C. Itching and edema A systemic anaphylactic reaction starts with edema and itching at the site of exposure to the antigen. Shock can rapidly develop with rapid, weak pulse; hypotension; dilated pupils; dyspnea, and possible cyanosis. The wheal-and-flare reaction occurs with a localized anaphylactic reaction such as a mosquito bite.

A 21-year-old student had taken amoxicillin once as a child for an ear infection. She is given an injection of Penicillin V and develops a systemic anaphylactic reaction. What manifestations would be seen first? A. Dyspnea B. Dilated pupils C. Itching and edema D. Wheal-and-flare reaction

C. Itching and edema A systemic anaphylactic reaction starts with edema and itching at the site of exposure to the antigen. Shock can rapidly develop with rapid, weak pulse; hypotension; dilated pupils; dyspnea, and possible cyanosis. The wheal-and-flare reaction occurs with a localized anaphylactic reaction such as a mosquito bite.

A patient has begun immunotherapy for the treatment of intractable environmental allergies. When administering the patient's immunotherapy, what is the nurse's priority action? A. Monitor the patient's fluid balance. B. Assess the patient's need for analgesia. C. Monitor for signs and symptoms of an adverse reaction. D. Assess the patient for changes in level of consciousness.

C. Monitor for signs and symptoms of an adverse reaction. When administering immunotherapy, it is imperative to closely monitor the patient for any signs of an adverse reaction. The high risk and significant consequence of an adverse reaction supersede the need to assess the patient's fluid balance. Pain and changes in level of consciousness are not likely events when administering immunotherapy.

A patient has begun immunotherapy for the treatment of intractable environmental allergies. When administering the patient's immunotherapy, what is the nurse's priority action? A. Monitor the patient's fluid balance. B. Assess the patient's need for analgesia. C. Monitor for signs and symptoms of an adverse reaction. D. Assess the patient for changes in level of consciousness.

C. Monitor for signs and symptoms of an adverse reaction. When administering immunotherapy, it is imperative to closely monitor the patient for any signs of an adverse reaction. The high risk and significant consequence of an adverse reaction supersede the need to assess the patient's fluid balance. Pain and changes in level of consciousness are not likely events when administering immunotherapy.

A 19-year-old male being tested for multiple allergies develops localized redness and swelling in reaction to a patch skin test. Which intervention by the nurse would have the highest priority? A. Notify the primary care provider B. Apply a topical anti-inflammatory cream C. Remove the patch and extract from the skin D. Administer oral diphenhydramine (Benadryl)

C. Remove the patch and extract from the skin If a severe reaction to a patch skin test occurs, the nurse should immediately remove the patch and the extract from the skin. Next the nurse should apply a topical anti-inflammatory cream to the site. A subcutaneous injection of epinephrine may also be necessary but would need a health care provider's order

Antenatal laboratory testing revealed a negative rubella antibody for a client admitted to the postpartum unit. Which action takes priority for this client during early puerperium? a) Rubella counseling and immunization with adult measles-mumps-rubella (MMR) vaccine b) Rubella counseling and immunization with Rho(D) immune gobulin vaccine c) Rubella counseling and immunization with live rubella virus vaccine d) Rubella counseling and instruction to obtain live rubella virus vaccine during her first postpartum examination

c) Rubella counseling and immunization with live rubella virus vaccine Explanation: A client who contracts rubella during pregnancy is at risk for delivering a fetus with severe congenital defects. Therefore, the client should receive rubella counseling and immunization with the live rubella virus vaccine before discharge. Immunization should take place as soon as possible before the client becomes pregnant again. The client should be advised to avoid pregnancy for 3 months after immunization. Rho(D) immune gobulin is administered to clients who are Rh-negative and come in contact with Rh-positive blood from an Rh-positive fetus.

The patient with diabetes mellitus has been ill for some time with a severe lung infection needing corticosteroids and antibiotics. The patient does not feel like eating. The nurse understands that this patient is likely to develop A. major histoincompatibility. B. primary immunodeficiency. C. secondary immunodeficiency. D. acute hypersensitivity reaction.

C. secondary immunodeficiency. Secondary immunodeficiency is most commonly caused by immunosuppressive drugs, such as corticosteroids. It can also be caused by diabetes mellitus, severe infection, malnutrition, and chronic stress, all of which are present in this patient. The other options are not possible for this patient. Histoincompatibility occurs when the human leukocyte antigen (HLA) system of the donor is not compatible with the recipient's HLA genes. Primary immunodeficiency is rare and includes phagocytic defects, B cell deficiency, T cell deficiency, or a combination of B cell and T cell deficiency. Acute hypersensitivity reaction is an anaphylactic-type allergic reaction to an antigen.

The client with idiopathic thrombocytopenic purpura (ITP) asks the nurse why it is necessary to take steroids. The nurse should base the response on which information? a) Steroids increase phagocytosis and increase the life of platelets. b) Steroids neutralize the antigens and prolong the life of platelets. c) Steroids alter the spleen's recognition of platelets and increase the life of platelets. d) Steroids destroy the antibodies and prolong the life of platelets.

c) Steroids alter the spleen's recognition of platelets and increase the life of platelets. ITP is treated with steroids to suppress the splenic macrophages from phagocytizing the antibody-coated platelets, which are recognized as foreign bodies, so that the platelets live longer. The steroids also suppress the binding of the autoimmune antibody to the platelet surface. Steroids do not destroy the antibodies on the platelets, neutralize antigens, or increase phagocytosis.

Which nursing action will be most useful in assisting a 21-year-old college student to adhere to a newly prescribed antiretroviral therapy (ART) regimen?

Check the patient's class schedule to help decide when the ART should be taken.

The patient has late stage non-small cell lung cancer. The physician is considering using crizotinib (Xalkori) for this patient. What should be done before it is prescribed for the patient? a) Give chemotherapy first. b) Test for hypersensitivity to this drug. c) Test for the abnormal anaplastic lymphoma kinase (ALK) gene. d) Test for gene abnormalities that will affect the appropriate dose.

c) Test for the abnormal anaplastic lymphoma kinase (ALK) gene. Pharmacogenetics shows that an abnormal ALK gene in the patient with late-stage non-small cell lung cancers causes the cancer to develop and grow. Crizotinib (Xalkori) works by blocking certain proteins called kinases, including the protein produced by the abnormal ALK gene. This drug interacts with many other drugs, so the patient's medications should be reviewed as well.

When explaining the long-term toxic effects of cancer treatments on the immune system, what should the nurse tell the client? a) Long-term immunologic effects have been studied only in clients with breast and lung cancer. b) Clients with persistent immunologic abnormalities after treatment are at a much greater risk for infection than clients with a history of splenectomy. c) The use of radiation and combination chemotherapy can result in more frequent and more severe immune system impairment. d) The helper T cells recover more rapidly than the suppressor T cells, which results in positive helper cell balance that can last 5 years.

c) The use of radiation and combination chemotherapy can result in more frequent and more severe immune system impairment. Explanation: Studies of long-term immunologic effects in clients treated for leukemia, Hodgkin's disease, and breast cancer reveal that combination treatments of chemotherapy and radiation can cause overall bone marrow suppression, decreased leukocyte counts, and profound immunosuppression. Persistent and severe immunologic impairment may follow radiation and chemotherapy (especially multiagent therapy). There is no evidence of greater risk of infection in clients with persistent immunologic abnormalities. Suppressor T cells recover more rapidly than the helper T cells.

The woman is afraid she may get HIV from her bisexual husband. What should the nurse include when teaching her about preexposure prophylaxis (select all that apply)? a) Take fluconazole (Diflucan). b) Take amphotericin B (Fungizone). c) Use condoms for risk-reducing sexual relations. d) Take emtricitabine and tenofovir (Truvada) regularly. e) Have regular HIV testing for herself and her husband.

c) Use condoms for risk-reducing sexual relations. d) Take emtricitabine and tenofovir (Truvada) regularly. e) Have regular HIV testing for herself and her husband. Using male or female condoms, having monthly HIV testing for the patient and her husband, and the woman taking emtricitabine and tenofovir regularly has shown to decrease the infection of heterosexual women having sex with a partner who participates in high-risk behavior. Fluconazole and amphotericin B are taken for Candida albicans, Coccidioides immitis, and Cryptococcosus neoformans, which are all opportunistic diseases associate with HIV infection.

At the health promotion level of care for HIV infection, which question is most appropriate for the nurse to ask? a. "Are you having any symptoms such as severe weight loss or confusion?" b. "Are you experiencing any side effects from the antiretroviral medications? c. "Do you need any assistance to obtain antiretroviral drugs or other treatments?" d. "Do you use any injectable drugs or have sexual activity with multiple partners?"

D Rationale: At the health-promotion level, the nurse screens for behaviors that might increase the risk for HIV infection and implements interventions to prevent infection (or, in the case of an already infected patient, implement interventions to prevent progression of the disease to AIDS). The other questions would be appropriate at the acute intervention level, when the patient already has significant immune compromise. Cognitive Level: Application Text Reference: pp. 260-261 Nursing Process: Assessment NCLEX: Health Promotion and Maintenance

Drug therapy is being considered for an HIV-infected patient who has a CD4+ cell count of 400/µl. The nursing assessment that is most important in determining whether therapy will be used is the patient's a. social support system offered by significant others and family. b. socioeconomic status and availability of medical insurance. c. understanding of the multiple side effects that the drugs may cause. d. willingness and ability to comply with stringent medication schedules.

D Rationale: Drug resistance develops quickly unless the patient takes multiple drugs on a stringent schedule, and this endangers both the patient and the community. The other information is also important to consider, but patients who are unable to manage and follow a complex drug treatment regimen should not be considered for ART. Cognitive Level: Comprehension Text Reference: pp. 264-265 Nursing Process: Assessment NCLEX: Health Promotion and Maintenance

The nurse is preparing to give the following medications to an HIV-positive patient who is hospitalized with PCP. Which is most important to administer at the right time? a. Nystatin (Mycostatin) tablet for vaginal candidiasis b. Aerosolized pentamadine (NebuPent) for PCP infection c. Oral acyclovir ((Zovirax to treat systemic herpes simplex d. Oral saquinavir (Inverase) to suppress HIV infection

D Rationale: It is important that antiretrovirals be taken at the prescribed time every day to avoid developing drug-resistant HIV. The other medications should also be given as close as possible to the correct time, but they are not as essential to receive at the same time every day. Cognitive Level: Application Text Reference: pp. 258, 264-265 Nursing Process: Implementation NCLEX: Physiological Integrity

A patient who is diagnosed with AIDS and has developed Kaposi's sarcoma tells the nurse, "I have lots of thoughts about dying. Do you think I am just being morbid?" Which response by the nurse is most appropriate? a. "Thinking about dying will not improve the course of AIDS." b. "Although your diagnosis is serious, there are more treatments available now." c. "Try to focus on the good things in life because stress impairs the immune system." d. "Tell me what kind of thoughts you have about dying."

D Rationale: More assessment of the patient's psychosocial status is needed before taking any other action. The statements, "Thinking about dying will not improve the course of AIDS" and "Try to focus on the good things in life ..." discourage the patient from sharing any further information with the nurse and decrease the nurse's ability to develop a trusting relationship with the patient. The statement, "Although your diagnosis is serious, there are more treatments available now" is correct, but without further assessment, it is impossible to know whether this responds to the patient's concerns. Cognitive Level: Application Text Reference: pp. 260, 265, 267 Nursing Process: Implementation NCLEX: Psychosocial Integrity

While teaching community groups about AIDS, the nurse informs people that the most common method of transmission of the HIV virus currently is a. perinatal transmission to the fetus. b. sharing equipment to inject illegal drugs. c. transfusions with HIV-contaminated blood. d. sexual contact with an infected partner.

D Rationale: Sexual contact with an infected partner is currently the most common mode of transmission, although HIV is also spread through perinatal transmission, through sharing drug injection equipment, and through transfusions with HIV-infected blood. Cognitive Level: Comprehension Text Reference: p. 250 Nursing Process: Assessment NCLEX: Health Promotion and Maintenance

Four years after seroconversion, an HIV-infected patient has a CD4+ cell count of 800/µl and a low viral load. The nurse teaches the patient that a. the patient is at risk for development of opportunistic infections because of CD4+ cell destruction. b. the patient is in a clinical and biologic latent period, during which very few viruses are being replicated. c. anti-HIV antibodies produced by B cells enter CD4+ cells infected with HIV to stop replication of viruses in the cells. d. the body currently is able to produce an adequate number of CD4+ cells to replace those destroyed by viral activity.

D Rationale: The patient is the early chronic stage of infection, when the body is able to produce enough CD4+ cells to maintain the CD4+ count at a normal level. The risk for opportunistic infection is low because of the normal CD4+ count. Although the viral load in the blood is low, intracellular reproduction of virus still occurs. Anti-HIV antibodies produced by B cells attack the viruses in the blood, but not intracellular viruses. Cognitive Level: Application Text Reference: pp. 252, 257 Nursing Process: Implementation NCLEX: Physiological Integrity

The nurse is teaching a 24-year-old female patient who has a latex allergy about preventing and treating allergic reactions. Which statement, if made by the patient, indicates a need for further teaching? A. "My dentist should be told about my latex allergy." B. "I should avoid foods such as bananas, avocados, and kiwi." C. "I will use vinyl gloves for activities such as housekeeping." D. "Because my reactions are not severe, I will not need an EpiPen."

D. "Because my reactions are not severe, I will not need an EpiPen." An individual with latex allergies should carry an injectable epinephrine pen. The proteins in latex are similar to the proteins in certain foods and may cause an allergic reaction in people who are allergic to latex. Foods to avoid include banana, avocado, chestnut, kiwi, tomato, water chestnuts, guava, hazelnuts, potatoes, peaches, grapes, and apricots. Vinyl gloves are not latex and are safe to use. Individuals with latex allergies need to share this information with all health care providers and wear a medical alert bracelet.

The nurse is teaching a 24-year-old female patient who has a latex allergy about preventing and treating allergic reactions. Which statement, if made by the patient, indicates a need for further teaching? A. "My dentist should be told about my latex allergy." B. "I should avoid foods such as bananas, avocados, and kiwi." C. "I will use vinyl gloves for activities such as housekeeping." D. "Because my reactions are not severe, I will not need an EpiPen."

D. "Because my reactions are not severe, I will not need an EpiPen." An individual with latex allergies should carry an injectable epinephrine pen. The proteins in latex are similar to the proteins in certain foods and may cause an allergic reaction in people who are allergic to latex. Foods to avoid include banana, avocado, chestnut, kiwi, tomato, water chestnuts, guava, hazelnuts, potatoes, peaches, grapes, and apricots. Vinyl gloves are not latex and are safe to use. Individuals with latex allergies need to share this information with all health care providers and wear a medical alert bracelet.

The nurse is teaching a 24-year-old female patient who has a latex allergy about preventing and treating allergic reactions. Which statement, if made by the patient, indicates a need for further teaching? A. "My dentist should be told about my latex allergy." B. "I should avoid foods such as bananas, avocados, and kiwi." C. "I will use vinyl gloves for activities such as housekeeping." D. "Because my reactions are not severe, I will not need an EpiPen."

D. "Because my reactions are not severe, I will not need an EpiPen." An individual with latex allergies should carry an injectable epinephrine pen. The proteins in latex are similar to the proteins in certain foods and may cause an allergic reaction in people who are allergic to latex. Foods to avoid include banana, avocado, chestnut, kiwi, tomato, water chestnuts, guava, hazelnuts, potatoes, peaches, grapes, and apricots. Vinyl gloves are not latex and are safe to use. Individuals with latex allergies need to share this information with all health care providers and wear a medical alert bracelet.

Which statement by the patient who has had an organ transplant would indicate that the patient understands the teaching about the immunosuppressive medications? A. "My drug dosages will be lower because the medications enhance each other." B. "Taking more than one medication will put me at risk for developing allergies." C. "I will be more prone to malignancies because I will be taking more than one drug." D. "The lower doses of my medications can prevent rejection and minimize the side effects."

D. "The lower doses of my medications can prevent rejection and minimize the side effects." Because immunosuppressants work at different phases of the immune response, lower doses of each drug can be used to produce effective immunosuppression while minimizing side effects. The use of several medications is not because they enhance each other and does not increase the risk of allergies or of malignancies

Which statement by the patient who has had an organ transplant would indicate that the patient understands the teaching about the immunosuppressive medications? A. "My drug dosages will be lower because the medications enhance each other." B. "Taking more than one medication will put me at risk for developing allergies." C. "I will be more prone to malignancies because I will be taking more than one drug." D. "The lower doses of my medications can prevent rejection and minimize the side effects."

D. "The lower doses of my medications can prevent rejection and minimize the side effects." Because immunosuppressants work at different phases of the immune response, lower doses of each drug can be used to produce effective immunosuppression while minimizing side effects. The use of several medications is not because they enhance each other and does not increase the risk of allergies or of malignancies.

Which statement by the patient who has had an organ transplant would indicate that the patient understands the teaching about the immunosuppressive medications? A. "My drug dosages will be lower because the medications enhance each other." B. "Taking more than one medication will put me at risk for developing allergies." C. "I will be more prone to malignancies because I will be taking more than one drug." D. "The lower doses of my medications can prevent rejection and minimize the side effects."

D. "The lower doses of my medications can prevent rejection and minimize the side effects." Because immunosuppressants work at different phases of the immune response, lower doses of each drug can be used to produce effective immunosuppression while minimizing side effects. The use of several medications is not because they enhance each other and does not increase the risk of allergies or of malignancies

Which patient is at highest risk for developing graft-versus-host disease? A. A 65-year-old man who received an autologous blood transfusion B. A 40-year-old man who received a kidney transplant from a living donor C. A 65-year-old woman who received a pancreas and kidney from a deceased donor D. A 40-year-old woman who received a bone marrow transplant from a close relative

D. A 40-year-old woman who received a bone marrow transplant from a close relative Graft-versus-host disease occurs when an immunoincompetent patient is transfused or transplanted with immunocompetent cells. Examples include blood transfusions or the transplantation of bone marrow, fetal thymus, or fetal liver. An autologous blood transfusion is the collection and reinfusion of the individual's own blood or blood components. There is no risk for graft-versus-host disease in this situation.

Which patient is at highest risk for developing graft-versus-host disease? A. A 65-year-old man who received an autologous blood transfusion B. A 40-year-old man who received a kidney transplant from a living donor C. A 65-year-old woman who received a pancreas and kidney from a deceased donor D. A 40-year-old woman who received a bone marrow transplant from a close relative

D. A 40-year-old woman who received a bone marrow transplant from a close relative Graft-versus-host disease occurs when an immunoincompetent patient is transfused or transplanted with immunocompetent cells. Examples include blood transfusions or the transplantation of bone marrow, fetal thymus, or fetal liver. An autologous blood transfusion is the collection and reinfusion of the individual's own blood or blood components. There is no risk for graft-versus-host disease in this situation.

Ten days after receiving a bone marrow transplant, a patient develops a skin rash on his palms and soles, jaundice, and diarrhea. What is the most likely etiology of these clinical manifestations? A. The patient is experiencing a type I allergic reaction. B. An atopic reaction is causing the patient's symptoms. C. The patient is experiencing rejection of the bone marrow. D. Cells in the transplanted bone marrow are attacking the host tissue.

D. Cells in the transplanted bone marrow are attacking the host tissue. The patient's symptoms are characteristic of graft-versus-host-disease (GVHD) in which transplanted cells mount an immune response to the host's tissue. GVHD is not a type I allergic response or an atopic reaction, and it differs from transplant rejection in that the graft rejects the host rather than the host rejecting the graft.

Ten days after receiving a bone marrow transplant, a patient develops a skin rash on his palms and soles, jaundice, and diarrhea. What is the most likely etiology of these clinical manifestations? A. The patient is experiencing a type I allergic reaction. B. An atopic reaction is causing the patient's symptoms. C. The patient is experiencing rejection of the bone marrow. D. Cells in the transplanted bone marrow are attacking the host tissue.

D. Cells in the transplanted bone marrow are attacking the host tissue. The patient's symptoms are characteristic of graft-versus-host-disease (GVHD) in which transplanted cells mount an immune response to the host's tissue. GVHD is not a type I allergic response or an atopic reaction, and it differs from transplant rejection in that the graft rejects the host rather than the host rejecting the graft.

A 52-year-old female patient was exposed to human immunodeficiency virus (HIV) 2 weeks ago through sharing needles with other substance users. What symptoms will the nurse teach the patient to report that would indicate the patient has developed an acute HIV infection? A. Cough, diarrhea, headaches, blurred vision, muscle fatigue B. Night sweats, fatigue, fever, and persistent generalized lymphadenopathy C. Oropharyngeal candidiasis or thrush, vaginal candidal infection, or oral or genital herpes D. Flu-like symptoms such as fever, sore throat, swollen lymph glands, nausea, or diarrhea

D. Flu-like symptoms such as fever, sore throat, swollen lymph glands, nausea, or diarrhea Clinical manifestations of an acute infection with HIV include flu-like symptoms between 2 to 4 weeks after exposure. Early chronic HIV infection clinical manifestations are either asymptomatic or include fatigue, headache, low-grade fever, night sweats, and persistent generalized lympadenopathy. Intermediate chronic HIV infection clinical manifestations include candidal infections, shingles, oral or genital herpes, bacterial infections, Kaposi sarcoma, or oral hairy leukoplakia. Late chronic HIV infection or acquired immunodeficiency syndrome (AIDS) includes opportunistic diseases (infections and cancer).

The patient with an allergy to bee stings was just stung by a bee. After administering oxygen, removing the stinger, and administering epinephrine, the nurse notices the patient is hypotensive. What should be the nurse's first action? A. Administer IV diphenhydramine (Benadryl). B. Administer nitroprusside as soon as possible. C. Anticipate tracheostomy with laryngeal edema. D. Place the patient recumbent and elevate the legs.

D. Place the patient recumbent and elevate the legs. In this emergency situation, the ABCs (airway, breathing, circulation) are being followed. For hypotension the patient should be placed in a recumbent position with the legs elevated, epinephrine will continue to be administered every 2-5 minutes, and fluids will be administered with vasopressors. Diphenhydramine is an antihistamine used to treat allergy symptoms. Anticipating a tracheostomy may occur with ongoing patient monitoring. Nitroprusside is a vasodilator and would not be used now.

The patient with an allergy to bee stings was just stung by a bee. After administering oxygen, removing the stinger, and administering epinephrine, the nurse notices the patient is hypotensive. What should be the nurse's first action? A. Administer IV diphenhydramine (Benadryl). B. Administer nitroprusside as soon as possible. C. Anticipate tracheostomy with laryngeal edema. D. Place the patient recumbent and elevate the legs.

D. Place the patient recumbent and elevate the legs. In this emergency situation, the ABCs (airway, breathing, circulation) are being followed. For hypotension the patient should be placed in a recumbent position with the legs elevated, epinephrine will continue to be administered every 2-5 minutes, and fluids will be administered with vasopressors. Diphenhydramine is an antihistamine used to treat allergy symptoms. Anticipating a tracheostomy may occur with ongoing patient monitoring. Nitroprusside is a vasodilator and would not be used now.

The patient with an allergy to bee stings was just stung by a bee. After administering oxygen, removing the stinger, and administering epinephrine, the nurse notices the patient is hypotensive. What should be the nurse's first action? A. Administer IV diphenhydramine (Benadryl). B. Administer nitroprusside as soon as possible. C. Anticipate tracheostomy with laryngeal edema. D. Place the patient recumbent and elevate the legs.

D. Place the patient recumbent and elevate the legs. In this emergency situation, the ABCs (airway, breathing, circulation) are being followed. For hypotension the patient should be placed in a recumbent position with the legs elevated, epinephrine will continue to be administered every 2-5 minutes, and fluids will be administered with vasopressors. Diphenhydramine is an antihistamine used to treat allergy symptoms. Anticipating a tracheostomy may occur with ongoing patient monitoring. Nitroprusside is a vasodilator and would not be used now.

The nurse is providing postoperative care for a 30-year-old female patient after an appendectomy. The patient has tested positive for human immunodeficiency virus (HIV). What type of precautions should the nurse observe to prevent the transmission of this disease? A. Droplet precautions B. Contact precautions C. Airborne precautions D. Standard precautions

D. Standard precautions Standard precautions are indicated for prevention of transmission of HIV to the health care worker. HIV is not transmitted by casual contact or respiratory droplets. HIV may be transmitted through sexual intercourse with an infected partner, exposure to HIV-infected blood or blood products, and perinatal transmission during pregnancy, at delivery, or though breastfeeding.

b. Genetic carrier screening should be done in families with a history of sickle cell disease. Diabetes and iron-deficiency anemia are not amenable to any form of genetic testing, whereas a family history of breast cancer suggests the need for presymptomatic testing for estimating the patient's risk of developing breast cancer.

For which of the following individuals is genetic carrier screening indicated? A. A patient with a history of type 1 diabetes B. A patient with a family history of sickle cell disease C. A patient whose mother and sister died of breast cancer D. A patient who has a long-standing history of iron-deficiency anemia

HIV and AIDs medications 1) the standard treatment for HIV/AIDs is known as ______ 2) NRTI's - name a drug (2) - Action - common adverse for each drug 3) NNRTI's - name a drug - Action - common adverse for each drug 4) Protease Inhibitor - name a drug - Action 5) Integrase inhibitor - name the drug - Action 6) Fusion Inhibitor - name the drug - Action 7) CCR5 Antagonist - name the drug - Action

HIV and AIDs medications 1) HAART (highly active antiretroviral therapy) 2) Nucleoside Reverse Transcriptase Inhibitor (NRTI) - Lamivudine & Zidovudine - Action: acts as a faulty building block that HIV needs to make copies - Adverse: - Lamivudine: nausea, vomiting - Zidovudine: leukopenia, myopathy 3) Non nucleoside Reverse Transcriptase Inhibitor - Efavirenz & Nevirapine - Action: disables a protein that HIV needs to make copies - Adverse - Efavirenz: encephalopathy - Nevirapine: hepatitis, Steven Johnson syndrome 4) Protease Inhibitor - Fosamprenavir (Lexiva) - Action: blocks protease which is a protein HIV needs to make copies 5) Integrase Inhibitor - Raltegravir (Isentress) - Action: blocks integrate which prevents HIV from injecting its material into CD4 cells 6) Fusion Inhibitor - Enfuvirtide - Action: prevents HIV from entering CD4 cells 7) CCR5 Antagonist - Maraviroc - Action: blocks viral entry

A patient is being evaluated for possible atopic dermatitis. The nurse will review the patient's laboratory values for the level of ______________

IgE.

Which findings should cause the nurse to postpone administration of an immunization and do further data collection? Select all that apply.

Immune deficiency disease Familial history of severe allergic response to immunization

Innate immunity

Immunity that is always present

A nurse is caring for a client who is experiencing the end-stage of acquired immunodeficiency syndrome (AIDS) . What is the goal of treatment for this client?

Promoting client comfort

Which action can the nurse delegate to nursing assistive personnel (NAP) who help with the care of a patient admitted with tuberculosis and placed on airborne precautions?

Stock the patient's room with all the necessary personal protective equipment.

d. The patient's symptoms are characteristic of graft-versus-host-disease (GVHD) in which transplanted cells mount an immune response to the host's tissue. GVHD is not a type I allergic response or an atopic reaction, and it differs from transplant rejection in that the graft rejects the host rather than the host rejecting the graft.

Ten days after receiving a bone marrow transplant, a patient has developed a skin rash on his palms and soles, jaundice, and diarrhea. What is the most likely etiology of these clinical manifestations? A. The patient is experiencing a type I allergic reaction. B. An atopic reaction is causing the patient's symptoms. C. The patient is experiencing rejection of the bone marrow. D. Cells in the transplanted bone marrow are rejecting the host tissue.

b. Seasonal allergic rhinitis is most commonly caused by pollens from trees, weeds, and grasses. Airborne allergies can be controlled by sleeping in an air-conditioned room, daily damp dusting, covering the mattress and pillows with hypoallergenic covers, and wearing a mask outdoors.

Which of the following statements made by the nurse is most appropriate in teaching patient interventions to minimize the effects of seasonal allergic rhinitis? A. "You will need to get rid of your pets." B. "You should sleep in an air-conditioned room." C. "You would do best to stay indoors during the winter months." D. "You will need to dust your house with a dry feather duster twice a week."

a patient asks why it is essential that HAART meds be taken everyday at the same time. what is the nurses best response? a. missing or delaying doses of these drugs decreases blood conenctrations needed to inhibit viral replication b. missing or delayed doses of these drugs decreases the risk of developing infections c. missing or delaying doses of these drugs decreases the effectiveness missing or delaying doses can decrease the risk of developing HIV resistant mutations

a

which point are you sure to include when teaching a new RN to prevent HIV transmission from patients? A. wear gloves when in contact with patients mucous membrane or non-intact skin B. be sure to wear protective gear when providing any care to HIV positive patients C. always war a mask D. use PEP whether a patient is positive or not

a

A patient who has been treated for HIV infection for 7 years has developed fat redistribution to the trunk, with wasting of the arms, legs, and face. The nurse will anticipate teaching the patient about ____________

a change in antiretroviral therapy.

The nurse is discussing treatment options with the parents of a child who has been newly diagnosed with leukemia. Which of the following is the most appropriate information for the nurse to give the parents about treatment options? a) "Chemotherapy has the greatest potential to cure the cancer." b) "Bone marrow transplant is the quickest treatment option." c) "There are numerous alternative therapies that are highly effective." d) "Immunotherapy has fewer side effects than traditional therapies."

a) "Chemotherapy has the greatest potential to cure the cancer." Chemotherapy is the treatment of choice for leukemia. Other options may eventually be considered, but initially a course of chemotherapy is the expected treatment modality.

Which is the best nursing response to make when a client asks why their blood glucose is higher on days when they sleep less? a) "Cortisol levels remain high when you sleep less, since cortisol is inhibited during sleep." b) "You are probably eating more when you sleep less. This is making your blood glucose higher than usual." c) "You are more active when you sleep less, so your blood glucose will be higher." d) "Your body is producing cortisol for the extra energy you need when you sleep less."

a) "Cortisol levels remain high when you sleep less, since cortisol is inhibited during sleep." Explanation: Cortisol is decreased during sleep, and also decreases blood glucose levels. Less sleep means that cortisol continues to be produced, and blood glucose levels are high. The other statements are incorrect.

When teaching a patient infected with HIV regarding transmission of the virus to others, which statement made by the patient would indicate a need for further teaching? a) "I will need to isolate any tissues I use so as not to infect my family." b) "I will notify all of my sexual partners so they can get tested for HIV." c) "Unprotected sexual contact is the most common mode of transmission." d) "I do not need to worry about spreading this virus to others by sweating at the gym."

a) "I will need to isolate any tissues I use so as not to infect my family." HIV is not spread casually. The virus cannot be transmitted through hugging, dry kissing, shaking hands, sharing eating utensils, using toilet seats, or attending school with an HIV-infected person. It is not transmitted through tears, saliva, urine, emesis, sputum, feces, or sweat.

Choice Multiple question - Select all answer choices that apply. A 4-year-old child with suspected leukemia is admitted to the hospital for diagnosis and treatment. Which tests aid in diagnosing leukemia? Select all that apply. a) Bone marrow aspiration and analysis b) Complete blood count c) Quantitative hemoglobin studies d) Lumbar puncture e) Chest radiography

a) Bone marrow aspiration and analysis b) Complete blood count d) Lumbar puncture Bone marrow aspiration and analysis are necessary to confirm leukemia. The bone marrow of a child with leukemia is characterized by hypercellularity, lack of fat globules, and blast cells (immature white cells). Complete blood counts show thrombocytopenia and neutropenia in clients with leukemia. Lumbar puncture is performed to detect meningeal involvement. Quantitative hemoglobin studies are used to diagnose thalassemia. Chest radiography verifies the presence of a mediastinal mass in those with suspected Hodgkin's disease.

A mother brings her 4-month-old infant to the clinic for a wellness checkup. Which immunizations should the infant receive? a) DTaP, IPV, Hib, hepatitis B, and pneumococcal conjugate vaccine (PCV) b) Haemophilus influenzae type B (Hib), rotavirus, DTaP, and IPV c) DTaP, hepatitis B, Hib, and varicella d) Diphtheria, tetanus toxoids, and acellular pertussis (DTaP), inactivated polio virus (IPV), rotavirus, and measles-mumps-rubella (MMR)

a) DTaP, IPV, Hib, hepatitis B, and pneumococcal conjugate vaccine (PCV) Explanation: DTaP, IPV, Hib, hepatitis B, and PCV are administered at ages 2 and 4 months. The MMR vaccine is typically administered at age 12 to 15 months. Rotavirus vaccine is no longer recommended because of the associated risk of intussusception. The varicella vaccine is commonly administered between ages 12 and 18 months

The HIV-infected patient is taught health promotion activities including good nutrition; avoiding alcohol, tobacco, drug use, and exposure to infectious agents; keeping up to date with vaccines; getting adequate rest; and stress management. What is the rationale behind these interventions that the nurse knows? a) Delaying disease progression b) Preventing disease transmission c) Helping to cure the HIV infection d) Enabling an increase in self-care activities

a) Delaying disease progression These health promotion activities along with mental health counseling, support groups, and a therapeutic relationship with health care providers will promote a healthy immune system, which may delay disease progression. These measures will not cure HIV infection, prevent disease transmission, or increase self-care activities.

A patient with a father with polycystic kidney disease does not want to have genetic testing done for polycystic kidney disease because she is worried that she might lose her health insurance if genetic abnormalities are found. Based on the nurse's knowledge of the Genetic Information Nondiscrimination Act (GINA), what should the nurse teach this patient? a) GINA should protect her from this happening. b) GINA does not extend to cover preexisting conditions. c) GINA will only protect her after she is diagnosed with polycystic kidney disease. d) GINA health coverage nondiscrimination protection also extends to life insurance.

a) GINA should protect her from this happening. The Genetic Information Nondiscrimination Act (GINA) prohibits discrimination in health care coverage and employment based on genetic information, but does not extend to life insurance, disability insurance, or long-term care insurance.

A healthy 2-month-old infant is being seen in the local clinic for a well-child checkup and initial immunizations. When analyzing the pediatric record, which immunizations would the nurse anticipate administering at this appointment? Select all that apply. a) IPV (inactivated polio vaccine) b) Hib (Haemophilus influenzae vaccine) c) Varicella (chickenpox) vaccine d) PCV (pneumococcal vaccine) e) DTaP (diphtheria, tetanus, and acellular pertussis) f) MMR (measles, mumps, and rubella)

a) IPV (inactivated polio vaccine) b) Hib (Haemophilus influenzae vaccine) d) PCV (pneumococcal vaccine) e) DTaP (diphtheria, tetanus, and acellular pertussis) At age 2 months, the American Academy of Pediatrics and Public Health Agency in Canada recommends the administration of DTaP, IPV, (Hep B in the United States), Hib, Rotavirus vaccine, and PCV. The MMR and varicella immunizations would be administered at 12 to 15 months.

When developing a teaching plan for the parent of an asthmatic child concerning measures to reduce allergic triggers, which suggestion should the nurse include? a) Keep the humidity in the home between 50% and 60%. b) Have the child sleep in the bottom bunk bed. c) Use a scented room deodorizer to keep the room fresh. d) Vacuum the carpet once or twice a week.

a) Keep the humidity in the home between 50% and 60%. Explanation: To help reduce allergic triggers in the home, the nurse should recommend that the humidity level be kept between 50% and 60%. Doing so keeps the air moist and comfortable for breathing. When air is dry, the risk for respiratory infections increases. Too high a level of humidity increases the risk for mold growth. Typically, the child with asthma should sleep in the top bunk bed to minimize the risk of exposure to dust mites. The risk of exposure to dust mites increases when the child sleeps in the bottom bunk bed because dust mites fall from the top bed, settling in the bottom bed. Scented sprays should be avoided because they may trigger an asthmatic episode. Ideally, carpeting should be avoided in the home if the child has asthma. However, if it is present, carpeting in the child's room should be vacuumed often, possibly daily, to remove dust mites and dust particles.

The nurse explains to the client that a biopsy of the enlarged lymph node is important because, if Hodgkin's disease is present, the histologic examination will reveal which of the following? a) Reed-Sternberg cells. b) Duchenne's cells. c) Tay-Sachs cells. d) Sarcoidosis cells.

a) Reed-Sternberg cells. A definitive diagnosis of Hodgkin's disease is made if Reed-Sternberg cells are found in the histologic examination of the excisional lymph node biopsy. Tay-Sachs disease is an inherited disease carried by an autosomal recessive gene. Sarcoidosis is an inflammatory granulomatous disease. Duchenne's disease is a type of muscular disorder.

Choice Multiple question - Select all answer choices that apply. A nurse is assigned a client with an acute exacerbation of rheumatoid arthritis (RA). Which medical facts about RA are essential in developing a plan of care? Select all that apply. a) The client experiences stiff, swollen joints bilaterally. b) The client may not exercise once the disease is diagnosed. c) Onset is acute and usually occurs between ages 20 and 40. d) Erythrocyte sedimentation rate (ESR) is elevated, and x-rays show erosions and decalcification of involved joints. e) The first-line treatment is gold salts and methotrexate. f) Inflamed cartilage triggers complement activation, which stimulates the release of additional inflammatory mediators.

a) The client experiences stiff, swollen joints bilaterally. d) Erythrocyte sedimentation rate (ESR) is elevated, and x-rays show erosions and decalcification of involved joints. f) Inflamed cartilage triggers complement activation, which stimulates the release of additional inflammatory mediators. Explanation: RA is a chronic disorder where individuals experience stiff, swollen joints due to a severe inflammatory reaction. Elevated ESR and x-ray evidence of bony destruction are indicative of severe involvement. RA starts insidiously, with fatigue, persistent low-grade fever, anorexia, and vague skeletal symptoms, usually in middle age between the ages 35 and 50 years. Maintaining the ROM by a prescribed exercise program is essential, but clients must rest between activities. Salicylates and nonsteroidal anti-inflammatory drugs are considered the first-line treatments.

An 8-year-old child has been admitted to the oncology unit with a suspected diagnosis of acute lymphoblastic leukemia. The nurse is obtaining a health history from the parents. During the interview, the parents ask the nurse if any of the factors discussed would make their child more at risk for this type of leukemia. What information about potential risk factors is correct for the nurse to share with the parents? a) The diagnosis of Down's syndrome at birth b) A diet that includes a large proportion of dairy products c) A weight that is above the limit for the child's age d) The X-rays that the child had at age 6 for a broken leg

a) The diagnosis of Down's syndrome at birth Explanation: Children with Down's syndrome and other genetic conditions have an increased risk of developing acute lymphoblastic leukemia. Prenatal exposure to X-rays is actually a higher concern than postnatal exposure with respect to increasing the risk of developing ALL. The exception would be postnatal exposure to high doses of therapeutic radiation used as a treatment modality, which was not indicated here. Diet would have little impact on risk factors at this stage in the child's life.

A patient received a result of "negative" on a test for a specific genetic mutation. Which interpretation by the patient would the nurse reinforce as being accurate? a) The patient is not at high risk for the tested disease. b) The patient is not at high risk for any inherited disorder. c) The patient's relatives are not at risk for any inherited disorder. d) The patient's relatives are not at risk for the specific tested disease.

a) The patient is not at high risk for the tested disease. If a specific genetic test reveals a strongly negative result, the patient probably will not develop that disease. However, that does not eliminate the risk of other inherited disorders. The absence of a specific genetic mutation in the person tested does not eliminate the risk of other genetic diseases.

An 82-year-old woman is brought to her physician by her daughter with complaints of some confusion. What testing should the nurse suggest for this patient? a) Urinalysis b) Sputum culture c) Red blood cell count d) White blood cell count

a) Urinalysis The developments of urinary tract infections commonly contribute to atypical manifestations such as cognitive and behavior changes in older adults. Sputum culture, red blood cell count, and white blood cell count may be done, but the first step would be to assess for a possible urinary tract infection.

Choice Multiple question - Select all answer choices that apply. A school nurse is gathering registration data for a child entering first grade. Which immunizations would the school nurse verify that the child has had? Select all that apply. a) Varicella vaccine. b) Diphtheria-tetanus-pertussis series. c) H. influenzae type b series. d) Oral polio series. e) Pneumonia vaccine. f) Influenza vaccine.

a) Varicella vaccine. b) Diphtheria-tetanus-pertussis series. c) H. influenzae type b series. Explanation: The exact immunization schedule differs between the United States and Canadian provinces but there are many similarities. Diphtheria-tetanus-pertussis series, H. influenzae type b series, varicella, and inactivated (not oral) polio series are the immunizations that the child would receive before entering first grade. The oral polio vaccine was discontinued; the safer IPV is now used. Pneumonia vaccine is not required or routinely given to children.

What should the nurse teach the patients in the assisted living facility to decrease their risk for antibiotic-resistant infection (select all that apply)? a) Wash hands frequently. b) Take antibiotics as prescribed. c) Take the antibiotic until it is gone. d) Take antibiotics to prevent illnesses like colds. e) Save leftover antibiotics to take if needed later.

a) Wash hands frequently. b) Take antibiotics as prescribed. c) Take the antibiotic until it is gone. To decrease the risk for antibiotic-resistant infections, people should wash their hands frequently, follow the directions when taking the antibiotics, finish the antibiotic, do not request antibiotics for colds or flu, do not save leftover antibiotics, or take antibiotics to prevent an illness without them being prescribed by a health care provider

After birth, a direct Coombs test is performed on the umbilical cord blood of a neonate with Rh-positive blood born to a mother with Rh-negative blood. The nurse explains to the client that this test is done to detect which information? a) antibodies coating the neonate's red blood cells b) degree of anemia in the neonate c) initial bilirubin level d) antigens coating the neonate's red blood cells

a) antibodies coating the neonate's red blood cells A direct Coombs test is done on umbilical cord blood to detect antibodies coating the neonate's red blood cells. Hematocrit is used to detect anemia. A direct Coombs does not measure bilirubin but may help explain the underlying cause of increased bilirubin levels. Antigens on the neonate's red blood cells are proteins that help determine the neonate's blood type.

Choice Multiple question - Select all answer choices that apply. The nurse is teaching the parents of a 5-year-old child about commonly expected adverse effects of the diphtheria, tetanus, and pertussis; inactivated polio; and measles, mumps, and rubella vaccines. What possible effects should be included? Select all that apply. a) rash b) redness at the injection site c) anorexia d) diarrhea e) prolonged crying f) fever of 103° F (39.5° C)

a) rash b) redness at the injection site c) anorexia Common adverse effects associated with these vaccines include redness, swelling, soreness at the injection site, low-grade fever, anorexia, malaise, and fussiness; rash may also occur 7 to 10 days after the measles, mumps, and rubella vaccine. A fever of 103° F (39.5° C) is a high-grade fever and not usually seen following immunization administration. Prolonged crying and diarrhea are not associated with these immunizations.

Propylthiouracil (PTU) is prescribed for a client with Graves' disease. The nurse should teach the client to immediately report: a) sore throat. b) increased urine output. c) painful, excessive menstruation. d) constipation.

a) sore throat. The most serious adverse effects of PTU are leukopenia and agranulocytosis, which usually occur within the first 3 months of treatment. The client should be taught to promptly report to the health care provider (HCP) signs and symptoms of infection, such as a sore throat and fever. Clients having a sore throat and fever should have an immediate white blood cell count and differential performed, and the drug must be withheld until the results are obtained. Painful menstruation, constipation, and increased urine output are not associated with PTU therapy.

an HIV positive women who is pregnant asks if her baby is at risk for HIV. which points must the nurse be sure to include when teaching? (Select all that apply) A. HIV can cross the placenta B. infant can contract HIV with exposure to blood and vaginal secretions during birth C. once your baby is born, you should be able to breastfeed D. there is a risk for perinatal transmission of HIV from you to your child. because you are on drug therapy, that risk is about 8% E. consider oral contraceptives o protect yourself from other STDs

a, b, d

which descriptions are characteristic of a non progressor? (Select all that apply) A. has been infected for 10 years B. is asymptomatic C. has no CD4+ or t-lymphocytes D. is immunocompetent E. are functional antibodies

a, b, d

which methods or items are means of transmitting HIV (Select all that apply) a. sex b. household utensils c. breast milk d. toilet facilities e. mosquitoes

a, c

corticosteroids perform which actions (Select all that apply) a. block movement of neutrophils and monoctyes through cell membrane b. increase cell production in the bone marrow c. reduce number of circulating t cells, resulting in suppressed cell mediated immunity d. decrease ICP e. contrict blood vessels

a, c, d

where in the body can cytomegalovirus present with symptoms? (Select all that apply) a. eyes, causing visual impairment b. kidneys as glomerulonephritis c. respiratory tract causing pneumonia d. GI tract, causing diarrhea e. heart as cardiomyopathy

a, c, d

which conditions may be the first signs of HIV in women? (Select all that apply) A. vaginal candidiasis B. bladder infections C. cervical caner D. PID E. mononucleosis

a, c, d

a patient presenting with toxicoplasmosis may have with s/s? (Select all that apply) A. speech difficulty B. Shortness of breath C. visual changes D. impaired gait E. mental status changes

a, c, d, e

A nurse is providing discharge teaching to a client who is immunosuppressed. Which statement by the client indicates the need for additional teaching? a) "I won't go to see my nephew right after he gets his vaccines." b) "I stopped smoking last year; this year I'll quit drinking alcohol." c) "I won't go to see my sister while she has a cold." d) "I can eat whatever I want as long as it's low in fat."

d) "I can eat whatever I want as long as it's low in fat." The client requires additional teaching if he states that he can eat whatever he wants. Immunosuppressed clients should avoid raw fruit and vegetables because they may contain bacteria that could increase the risk of infection; foods must be thoroughly cooked. Avoiding people who are sick, products containing alcohol, and people who have just received vaccines are appropriate actions for an immunosuppressed client.

A preschooler with a history of repaired lumbar myelomeningocele is in the emergency department with wheezing and skin rash. Which question should the nurse ask the parent first? a) "What are you doing to treat your child's skin rash?" b) "Who brought your child to the emergency department?" c) "Is your child taking any medications?" d) "Is your child allergic to bananas or any other food?"

d) "Is your child allergic to bananas or any other food?" Children with myelomeningocele are at high risk for development of latex allergy because of repeated exposure to latex products during surgery and bladder catheterizations. Cross-reactions to food items such as bananas, kiwi, chestnuts, and avocados also occur. These allergic reactions vary in severity ranging from mild (such as sneezing) to severe anaphylaxis. While the child could have allergies to medications that caused the wheezing, latex and food allergies are more common. Asking about the skin rash is not a priority when a child is wheezing. Who brought the child to the emergency department is irrelevant at this time.

A 12-month-old child is seen in the neighborhood clinic for a regular checkup. Which statement by the child's mother about the influenza vaccine reflects the need for more teaching? a) "The first time a child receives the influenza vaccine, a second dose is recommended in 1 month." b) "Yearly influenza vaccinations are recommended to begin as early as 6 months of age." c) "My child is too young to receive the live attenuated intranasal vaccine." d) "The Haemophilus influenzae vaccine my child has already received helps protect against some forms of the flu."

d) "The Haemophilus influenzae vaccine my child has already received helps protect against some forms of the flu." Haemophilus influenzae is a bacteria that can cause severe disease in children younger than age 5 years, but it does not cause influenza. Yearly vaccination for influenza is recommended to begin at 6 months. The live vaccine is not recommended for children younger than 2 years or with respiratory disease. A second vaccine 4 weeks after the first is recommended the first time a child younger than 9 years receives the flu vaccine.

A nurse is evaluating the parents' comprehension of their child's diagnosis of celiac disease. Which of the following statements indicates a need for further teaching? a) "Celiac disease is an autoimmune disease with symptoms that vary by person." b) "Possible complications of the disease include vitamin deficiency and various neurological manifestations." c) "Our child must maintain these dietary restrictions for life." d) "We will need to bring our child in for regular blood work to monitor the disease."

d) "We will need to bring our child in for regular blood work to monitor the disease." Teaching is effective if the parents say that their child must maintain the dietary restrictions for life, because the child needs to avoid recurrence of the disease's clinical manifestations. Celiac disease is an autoimmune disorder that can cause vitamin deficiencies and neurological manifestations. The symptoms can vary from person to person. However, regular lab work is not needed.

The nurse is assigning a room for a client admitted with hepatitis A. Which of the following diagnoses would be an appropriate roommate for this client? a) Pneumonia b) Varicella c) Postoperative hip arthroplasty d) Congestive heart failure

d) Congestive heart failure The nurse is assigning rooms for the new admissions of the following clients. The nurse needs to determine the need for specific standard and transmission-based precautions. It would be appropriate for a hepatitis A and congestive heart failure client to share a room, as neither requires isolation. The varicella is airborne isolation and must be in a private negative airflow room. Postoperative clients should not be in a room with a medical client with a communicable infection.

A young mother is worried that her female baby will have hemophilia because the baby's father has it. How should the nurse explain this genetic disorder to the young mother? a) Nearly all affected people are male. b) Daughters of affected males will be carriers. c) The daughter has a 50% chance of being affected. d) If the mother is a carrier, the patient could have hemophilia.

d) If the mother is a carrier, the patient could have hemophilia. Because hemophilia is an X-linked recessive genetic disorder, hemophilia results from a mutated gene on an X chromosome. Mothers always contribute an X chromosome to their offspring, whereas fathers contribute X to their female children and Y to their male children. If the mother is a carrier, there is a 25% chance that the daughter could have hemophilia and a 25% chance that she will be a carrier. If the mother is not a carrier, the daughter in this case will have a 50% chance of being a carrier from her father's affected X chromosome.

When preparing a teaching plan for a client who is to receive a rubella vaccine during the postpartum period, the nurse should include which information? a) The vaccine prevents a future fetus from developing congenital anomalies. b) The client should avoid contact with children diagnosed with rubella. c) The injection will provide immunity against the chickenpox. d) Pregnancy should be avoided for 4 weeks after the immunization.

d) Pregnancy should be avoided for 4 weeks after the immunization. After administration of rubella vaccine, the client should be instructed to avoid pregnancy for at least 4 weeks to prevent the possibility of the vaccine's teratogenic effects to the fetus. The vaccine does not protect a future fetus from infection. Rather it protects the woman from developing the infection if exposed during pregnancy and subsequently causing harm to the fetus. The vaccine will provide immunity to rubella, also known as German measles. The injection immunizes the client against the 3-day or German measles, not chickenpox.

When discussing plans for genetic counseling with the parents of a fetus with Down syndrome, which function should the nurse include as the primary role of the genetic team when working with a family? a) Prepare the parents psychologically for the birth of a child with disabilities. b) Prescribe birth control or abortion measures for the parents as needed. c) Report the findings of chromosome analysis of the amniotic cells. d) Provide parents with information about the risks of birth defects.

d) Provide parents with information about the risks of birth defects. Explanation: The primary aim of genetic counseling is to inform clients of birth defect risks and the nature of the disorder to help the family understand and adjust to the disorder. Reporting results of chromosome analysis of amniotic cells is a secondary role of the team. Preparing a couple psychologically for the birth of a child with a disability is a secondary role. Although suggestions may be offered, the decision about birth control methods should be left to the couple.

A nurse is taking a health history from a healthy 30-year-old man. Which information about the patient's family history increases his risk of a genetic-related disease? a) Relatives with unhealthy diet and exercise behaviors b) Disease onset at a later age than is usual in the population c) Family disease incidence similar to the general population d) Same disease present in more than one close relative

d) Same disease present in more than one close relative The key features of a family history that may increase a person's risk for genetic-related diseases are disease in more than one close relative, disease that does not usually affect a certain gender (e.g., breast cancer in a male), diseases that occur at an earlier age than expected (myocardial infarction 10 to 20 years before most people have one), and certain combinations of diseases within a family (e.g., breast and ovarian cancer, heart disease, and diabetes). Relatives' later health behavior choices do not affect a person's genetic risk. Reduced incidence or later onset of disease indicates less risk.

The couple is adopting a baby girl. What health information related to the baby's biologic parents will be most useful to the parents and the baby as she grows up? a) The grandmother had breast cancer. b) The family has a history of Alzheimer's disease. c) The family has an individual with Down syndrome. d) The family has familial adenomatous polyposis (FAP).

d) The family has familial adenomatous polyposis (FAP). Because familial adenomatous polyposis occurs in those with the gene, being able to screen, monitor, and treat this baby will save her life in the long run. Breast cancer may or may not occur if the BRCA 1 or 2 are mutated. Many people who are positive for Apo E-4 do not develop Alzheimer's disease. Down syndrome results from a chromosomal alteration and not a mutated gene.

When teaching the mother of a child diagnosed with phenylketonuria (PKU) about its transmission, the nurse should use knowledge of which factor as the basis for the discussion? a) chromosome translocation b) x-linked recessive gene c) chromosome deletion d) autosomal recessive gene

d) autosomal recessive gene PKU is caused by an inborn error of metabolism. It is an autosomal recessive disorder that inhibits the conversion of phenylalanine to tyrosine. A form of Down syndrome, trisomy 21, is an example of a disorder caused by chromosomal translocation. Cri du chat is an example of a disorder caused by chromosomal deletion. Hemophilia A is an example of a disorder caused by an X-linked recessive gene

which groups are experiencing increased numbers of HIV infection? (Select all that apply) A. men having sex with other men B. IV drug users C. women having sex with men D. african americans E. hispanics

d, e

While obtaining a health history from the patient who works as a laboratory technician, the nurse learns that the patient has a history of allergic rhinitis, asthma, and multiple food allergies. It is important that the nurse ______________

document the patient's allergy history and be alert for any clinical manifestations of a type I latex allergy.

A patient who received a bone marrow transplant for treatment of leukemia develops a skin rash 10 days after the transplant. The nurse recognizes this reaction as an indication that the ______________

donor T cells are attacking the patient's skin cells.

Following a splenectomy, a client has a hemoglobin (Hb) level of 7.5 g/dl and has vertigo when getting out of bed. The nurse suspects abnormal orthostatic changes. The vital sign values that would most support the nurse's suspicions are:

drop in blood pressure and rise in heart rate.

Choice Multiple question - Select all answer choices that apply. The nurse is planning care for a client with human immunodeficiency virus (HIV). Which statement by the nurse indicates understanding of HIV transmission? Select all that apply. a) "I will wear a gown, mask, and gloves with all client contact." b) "I don't need to wear any personal protective equipment due to decreased risk of occupational exposure." c) "I will wear a mask if the client has a cough caused by an upper respiratory infection." d) "I will wear a mask, gown, and gloves when splashing bodily fluids is likely." e) "I will wash my hands after client care."

e) "I will wash my hands after client care." d) "I will wear a mask, gown, and gloves when splashing bodily fluids is likely." Standard precautions include wearing gloves for any known or anticipated contact with blood, body fluids, tissue, mucous membranes, and nonintact skin. If the task or procedure may result in splashing or splattering of blood or body fluids to the face, the nurse should wear a mask and goggles or face shield. If the task or procedure may result in splashing or splattering of blood or body fluids, the nurse should wear a fluid-resistant gown or apron. The nurse should wash hands before and after client care and after removing gloves. A gown, mask, and gloves are not necessary for all client care unless contact with bodily fluids, tissue, mucous membranes, and nonintact skin is expected. Nurses have an increased, not decreased, risk of occupational exposure to blood-borne pathogens. HIV is not transmitted in sputum unless blood is present.

When designing a program to decrease the incidence of HIV infection in the community, the nurse will prioritize teaching about ________________

how to prevent transmission between sexual partners.

A patient is admitted to the hospital with Pneumocystis jiroveci pneumonia (PCP) and HIV testing is positive. Based on diagnostic criteria established by the Centers for Disease Control and Prevention (CDC), the patient is diagnosed as having ____________

late chronic infection or AIDS.

myelomeningocele

latex allergy Cross-reactions to food items such as bananas, kiwi, chestnuts, and avocados also occur.

A client with allergic rhinitis is prescribed loratadine (Claritin). On a follow-up visit, the client tells the nurse, "I take one 10-mg tablet of Claritin with a glass of water two times daily." The nurse concludes that the client requires additional teaching about this medication because:

loratadine should be taken once daily for allergic rhinitis.

A patient seen in the outpatient clinic has an immune deficiency involving the T-lymphocytes. The nurse should teach the patient about the need for more frequent screening for ________________

malignancy.

variable region arrangement

occurs in the bone marrow generates antigenic specificity independent of T cell help independent of the antigen

Complement function

opsonization - enhancing phagocytosis of antigens chemotaxis- attracting macrophages and neutrophils cell lysis - rupturing membranes of foregin cells aggulination - clustering and binding of forgein cells

When assessing an individual who has been diagnosed with early chronic HIV infection and has a normal CD4+ count, the nurse will _______________

palpate the regional lymph nodes.

A 24-year-old woman who uses injectable illegal drugs asks the nurse about preventing AIDS. The nurse informs the patient that the best way to reduce the risk of HIV infection from drug use is to ______________

participate in a needle-exchange program.

The nurse encourages a new mother to breastfeed her infant, even for a short time, because colostrum will provide the infant with ________________

passive immunity.

A pregnant patient with a family history of cystic fibrosis (CF) asks for information about genetic testing. The most appropriate action by the nurse is to ______________

refer the patient to a qualified genetic counselor.

After having a positive rapid-antibody test for HIV, a patient is anxious and does not appear to hear what the nurse is saying. At this time, it is most important that the nurse ____________

remind the patient about the need to return for retesting to verify the results.

A patient diagnosed with systemic lupus erythematosus (SLE) is scheduled for plasmapheresis. The nurse plans to teach the patient that plasmapheresis will _______________

remove antibody-antigen complexes from circulation.

Type 4 hypersensitivity

soluble cell associated effector mechanisms - macrophages, eosinophils, CTL's Causes chronic asthma and contact dermatitis

IgA

the antibidy you produce when your immune to something neutalises the antigen and fixes compliment can be secreted onto mucosal membranes as it is coated in sugar residues

The nurse is caring for a client with possible immune deficiency. Which subjective data would be most indicative?

this answer is incorrect but was shown as the correct answer on prep U "I sneeze and have watery eyes throughout the spring and summer." Explanation: Immune deficiencies make it harder for the body to fight infection. With a low resistance, the client is susceptible to obtaining more circulating viruses. Having morning stuffiness and a sore throat is indicative of sinus congestion. Having a leg sore is indicative of cardiovascular insufficiency or diabetes. Sneezing with watery eyes is indicates seasonal allergies.

IgE

triggers the release of histidine and mast cell degradation

The nurse is caring for a client with acquired immunodeficiency syndrome (AIDS). To adhere to standard precautions, the nurse should:

wear gloves when providing mouth care.

A child with Down syndrome has an upper respiratory infection (URI). Which of the following is the nurse's best action? Select all that apply. a) Restricting visitation of sick siblings b) Providing fluids that the child likes c) Administering oxygen d) Consulting a speech therapist e) Ensuring that child is as active as possible

• Providing fluids that the child likes • Restricting visitation of sick siblings • Ensuring that child is as active as possible Explanation: A child with Down syndrome has deficits in the immune system and increased mucus viscosity, which contribute to URI. Providing fluids the child likes will increase the chance the child will drink the fluid and help with hydration. Sick siblings should not visit, as the child has deficits in the immune system. Increasing activity as much as possible will help the URI to resolve. Speech therapy and oxygen are not routinely needed for a child with Down syndrome who has URI.


Related study sets

Marriage & Family: ch. 3 -7 Test

View Set

EBP Practice Questions (Chapter 1-19)

View Set

Pre-Work Cancer Module 2 Resources/Questions

View Set